首页 11道经典逻辑推理题

11道经典逻辑推理题

举报
开通vip

11道经典逻辑推理题11道经典的逻辑推理题(申请加精) 猜帽子1 有三顶红帽子和两顶蓝帽子。将五顶中的三顶帽子分别戴在A、B、C三人头上。这三人每人都只能看见其他两人头上的帽子,但看不见自己头上的帽子,并且也不知道剩余的两顶帽子的颜色。 问A:"你戴的是什么颜色的帽子?" A说:"不知道。" 问B:"你戴的是什么颜色的帽子?" B想了想之后,也说:"不知道。" 最后问C。C回答说:"我知道我戴的帽子是什么颜色了。" 当然,C是在听了A、B的回答之后而作出推断的。试问:C戴的是什么颜色的帽子? 猜帽子...

11道经典逻辑推理题
11道经典的逻辑推理题(申请加精) 猜帽子1 有三顶红帽子和两顶蓝帽子。将五顶中的三顶帽子分别戴在A、B、C三人头上。这三人每人都只能看见其他两人头上的帽子,但看不见自己头上的帽子,并且也不知道剩余的两顶帽子的颜色。 问A:"你戴的是什么颜色的帽子?" A说:"不知道。" 问B:"你戴的是什么颜色的帽子?" B想了想之后,也说:"不知道。" 最后问C。C回答说:"我知道我戴的帽子是什么颜色了。" 当然,C是在听了A、B的回答之后而作出推断的。试问:C戴的是什么颜色的帽子? 猜帽子2 一群人开舞会,每人头上都戴着一顶帽子。帽子只有黑白两种,黑的至少有一顶。每个人都能看到其它人帽子的颜色,却看不到自己的。主持人先让大家看看别人头上戴的是什幺帽子,然后关灯,如果有人认为自己戴的是黑帽子,就拍手。第一次关灯,没有声音。于是再开灯,大家再看一遍,关灯时仍然鸦雀无声。一直到第三次关灯,才有劈劈啪啪打耳光的声音响起。问有多少人戴着黑帽子? 猜帽子3 小明、小丰、小兰三位学生这学期在侦探推理竞赛中并列第一,但学校每年只会颁给一个人奖状,于是老师请他们放学后到办公室,决定谁拿这个奖状。 放学后,在办公室里老师让他们闭上眼,给他们每人戴了一顶帽子,再让他们挣开眼,然后说要看看他们的逻辑推理能力,并告诉他们帽子只有绿黄两种,请看到绿帽子的举手,谁先说出自己戴的帽子的颜色,就把奖状颁给谁。 三个人听后都举手了。过了一会,小兰说:“我知道自己戴的是什么颜色的帽子了。” 请问小兰戴的是什么颜色的帽子? 猜帽子4 有3顶橙帽子,4顶青帽子,5顶紫帽子。让10个人从矮到高站成一队,给他们每个人头上戴一顶帽子。每个人都看不见自己戴的帽子颜色,只能看见站在前面比自己矮的人的帽子颜色。所以最后一个人可以看见前面9个人头上帽子的颜色,而最前面那个人谁的帽子都看不见。现在从最后那个人开始,问他是不是知道自己戴的帽子颜色,如果他回答说不知道,就继续问他前面那个人。假设最前面那个人戴的是青帽子,他一定会知道自己的帽子颜色,为什么? 扑克牌(我改编的,与原版的解题思路稍有不同) 1位老师有2个推理能力很强的学生,他告诉学生他手里有以下的牌 黑桃:4,5,6,7,Q,K 红心:4,6,7,8,Q 梅花:3,8,J,Q 方块:2,3,9 然后从中拿出一张牌,告诉了A这张牌的大小,告诉了B这张牌的花色 A:我不知道这张是什么牌 B:我也不知道这张是什么牌 A:现在我们可以知道了 请问这张是什么牌? 扑克牌(升级版)(原版) 1位老师有2个推理能力很强的学生,他告诉学生他手里有以下的牌 黑桃:2,5,7,9,J,K 红心:3,4,9,J,K 梅花:5,8,9,Q 方块:2,7,8 然后从中拿出一张牌,告诉了A这张牌的大小,告诉了B这张牌的花色 A:我不知道这张是什么牌 B:我知道你不知道这张是什么牌 A:现在我知道了 B:现在我也知道了 请问这张是什么牌? 海盗分赃1 5个很聪明的海盗抢到100个金币,他们决定依次由A,B,C,D,E五个海盗来分 当由A分时,剩下的海盗表决,如果B,C,D,E四人中有一半以上反对就把A扔下海,再由B分……以此类推;如果一半及以上的人同意,就按A的分法 请问A要依次分给B,C,D,E多少才能不被扔下海并且让自己拿到最多? 海盗分赃2 5个很聪明的海盗抢到100个金币,他们决定依次由A,B,C,D,E五个海盗来分 当由A分时,如果A,B,C,D,E五人中有一半以上反对就把A扔下海,再由B分……以此类推;如果一半及以上的人同意,就按A的分法 请问A要依次分给B,C,D,E多少才能不被扔下海并且让自己拿到最多? 海盗分赃3 5个很聪明的海盗抢到100个金币,他们决定依次由A,B,C,D,E五个海盗来分 当由A分时,剩下的海盗表决,如果B,C,D,E四人中有一半及以上反对就把A扔下海,再由B分……以此类推;如果一半以上的人同意,就按A的分法 请问A要依次分给B,C,D,E多少才能不被扔下海并且让自己拿到最多? 阿凡提九死一生   古时候有个残酷的国王,十分嫉妒阿凡提的聪明才智。有一次他抓住了阿凡提,一心想整死他,但又顾及到体面,就故意想了一个自认为天衣无缝的办法。他对阿凡提说:你现在可以说一句陈述的话,但是如果你说的是真话,我将用绞刑架吊死你,如果你说的是假话,我将用油锅炸死你。结果阿凡提说出一句话,国王意拿他一点招也没有。问:阿凡提说的是一句什么话? 神仙指路   有个智者去找神仙,走到一个三岔路口,不知道往左走还是往右。路口边站着两个天使,他俩一个永远说真话,另一个永远说假话,现在要求这个智者只能向其中一位天使问一句话,就确定神仙的方位。请问:这个智者怎么问才能有结果? 答案见下: 猜帽子1 C戴红帽子 猜帽子2 我认为是3个人戴黑帽子   分析:假设戴黑帽子的是A、B、C三人,以A的角度思考,A看到B、C戴黑帽子,A认为:第一次关灯时B看到C戴黑帽子,已满足“黑的至少有一顶”,所以B不能确定自己是否黑帽子,不会拍手,并且如果只有C戴黑帽子,第一次关灯时C就会拍手。但第一次关灯时C没拍手,这代表C也在等别人拍手,B就知道自己也戴了黑帽子,第二次关灯时B、C就都会拍手。但第二次关灯时也没拍手,这代表B、C也各自看到2顶黑帽子,A由此推出自己带了黑帽子。B、C逻辑推理也是如此,其他戴白帽子的人都是如此推理,在第三次关灯时会等着A、B、C拍手,于是第三次关灯时有且仅有三个人会拍手 猜帽子3 小兰戴绿帽子   分析:首先,由“三个人听后都举手”,推出小兰至少看到一顶绿帽子并且不会有2人戴黄帽子。 情况一:小兰、小丰戴绿帽子,小明戴黄帽子。小兰认为:如果自己戴黄帽子,小丰不会举手,所以自己戴绿帽子。之后小丰也能推理出自己戴绿帽子,但小明推理不出自己戴什么颜色的帽子,原因不说明了。 情况二:小兰、小丰、小明戴绿帽子。小兰认为:小丰看到小明戴绿帽子会举手,但小丰看不到自己帽子颜色的情况下却没有因为小明举手而推理出自己是戴绿帽子,这代表不光小丰和小明两人戴绿帽子(即代表不是情况一),所以小兰戴绿帽子。但小丰和小明推理不出自己戴什么颜色的帽子 猜帽子4 不知道 扑克牌(我改编的) 梅花3 扑克牌(原版) 方块8 海盗分赃1 A-97 B-0 C-1 D-2 E-0或A-97 B-0 C-1 D-0 E-2   提示:当扔下ABC后,D就算分D-0,E-100,E也可能不同意再扔下D,因此就算C分C-100,D-0,E-0,D也会同意 海盗分赃2 A-98 B-0 C-1 D-0 E-1   提示:当扔下ABC后,D分D-100,E-0,D就能拿到全部,因此C分C-99,D-0,E-1就行 海盗分赃3 A-97 B-0 C-1 D-1 E-1 阿凡提九死一生   答:国王要炸死我。   解释:如果这句话是真的,那么应当执行吊刑,但如果执行吊刑,就反过来证明这句话是假的,是假的就不应当执行吊刑;如果当这句话是假的,那么应当执行炸刑,但如果执行炸刑,就反过来证明这句话是真的,是真的就不应当执行炸刑。所以吊也不行,炸也不行,国王一言九鼎,只好放了他。 神仙指路   答:这个智者随便对其中一位天使说——如果我问那位天使神仙在哪边,他会说哪边?   解释:假设之一、神仙在左边——如果这位天使是说真话的,那么另一位天使将回答在右边,而这位天使也将转告右边;如果这位天使是说假话的,那么另一位天使将回答在左边,而这位天使却将转告右边。假设之二、神仙在右边——如果这位天使是说真话的,那么另一位天使将回答在左边,而这位天使也将转告左边;如果这位天使是说假话的,那么另一位天使将回答在右边,而这位天使却将转告左边。   结论:不管天使说哪边,神仙肯定在相反的方向,虽然我们并不知道哪位天使说真话。 启示:此题其实是一道二元方程式,天使说真说假代表X,神仙在左在右代表Y,回答的两个解代表Z。我们逆向求解的思路应当是问一句同时牵涉两位天使的话,使X、Y合作起来推导Z。 8道经典逻辑推理题 一、 Q先生和S先生、 P先生在一起做游戏。 Q先生用两张小纸片,各写一个数。这两个数都 是正整数,差数是1。他把一张纸片贴在S先生额头上,另一张贴在P先生额头上。于是, 两个人只能看见对方额头上的数。 Q先生不断地问:你们谁能猜到自己头上的数吗? S先生说:“我猜不到。” P先生说:“我也猜不到。” S先生又说:“我还是猜不到。” P先生又说:“我也猜不到。” S先生仍然猜不到; P先生也猜不到。 S先生和P先生都已经三次猜不到了。 可是,到了第四次, S先生喊起来:“我知道了!” P先生也喊道:“我也知道了!” 问: S先生和P先生头上各是什么数? 二、 有一个牢房,有3个犯人关在其中。因为玻璃很厚,所以3个人只能互相看见,不能听到 对方说话的声音。” 有一天,国王想了一个办法,给他们每个人头上都戴了一顶帽子,只叫他们知道帽 子的颜色不是白的就是黑的,不叫他们知道自己所戴帽子的是什么颜色的。在这种情况 下,国王宣布两条如下: 1.谁能看到其他两个犯人戴的都是白帽子,就可以释放谁; 2.谁知道自己戴的是黑帽子,就释放谁。 其实,国王给他们戴的都是黑帽子。他们因为被绑,看不见自己罢了。于是他们3个 人互相盯着不说话。可是不久,心眼灵的A用推理的方法,认定自己戴的是黑帽子。您想 ,他是怎样推断的? 三、 有一个很古老的村子,这个村子的人分两种,红眼睛和蓝眼睛,这两种人并没有什 么不同,小孩在没生出来之前,没人知道他是什么颜色的眼睛,这个村子中间有一个广 场,是村民们聚集的地方,现在这个村子只有三个人,分 住三处。在这个村子,有一个规定,就是如果一个人能知道自己眼睛的颜色并且在晚上 自杀的话,他就会升入天堂,这三个人不能够用语言告诉对方眼睛的颜色,也不能用任 何方式提示对方的眼睛是什么颜色,而且也不能用镜子, 水等一切有反光的物质来看到自己眼睛的颜色,当然,他们不是瞎子,他们能看到对方 的眼睛,但就是不能告诉他!他们只能用思想来思考,于是他们每天就一大早来到广场 上,面对面的傻坐着,想自己眼睛的颜色,一天天过去了 ,一点进展也没有,直到有一天,来了一个外地人,他到广场上说了一句话,改变了他 们的命运,他说,你们之中至少有一个人的眼睛是红色的。说完就走了。这三个人听了 之后,又面对面的坐到晚上才回去睡觉,第二天,他们又 来到广场,又坐了一天。当天晚上,就有两个人成功的自杀了!第三天,当最后一个人 来到广场,看到那两个人没来,知道他们成功的自杀了,于是他也回去,当天晚上,也 成功的自杀了!   根据以上,请说出三个人的眼睛的颜色,并能够说出推理过程! 四、 两个房子互为隔壁,一个房子中的三个开关控制另一个房子的三盏灯。 你只能各进入这二个房子一次,怎么来判断哪个开关控制哪盏灯? 五、 有9个点排列如下: . . . . . . . . . 如何用四条直线把这9个点连起来,(要求这四条直线是连续的) 六、 注:美国货币中的硬币有1美分、5美分、10美分、25美分、50美分和1美元这几种面值 。请接着看正文吧,挑战你逻辑推理的极限。 一家小店刚开始营业,店堂中只有三位男顾客和一位女店主。当这三位男士同时站 起来付帐的时候,出现了以下的情况: (1)这四个人每人都至少有一枚硬币,但都不是面值为1美分或1美元的硬币。 (2)这四人中没有一人能够兑开任何一枚硬币。 (3)一个叫卢的男士要付的帐单款额最大,一位叫莫的男士要付的帐单款额其次, 一个叫内德的男士要付的帐单款额最小。 (4)每个男士无论怎样用手中所持的硬币付帐,女店主都无法找清零钱。 (5)如果这三位男士相互之间等值调换一下手中的硬币,则每个人都可以付清自己 的帐单而无需找零。 (6)当这三位男士进行了两次等值调换以后,他们发现手中的硬币与各人自己原先 所持的硬币没有一枚面值相同。 随着事情的进一步发展,又出现如下的情况: (7)在付清了帐单而且有两位男士离开以后,留下的男士又买了一些糖果。这位男 士本来可以用他手中剩下的硬币付款,可是女店主却无法用她现在所持的硬币找清零钱。 (8)于是,这位男士用1美元的纸币付了糖果钱,但是现在女店主不得不把她的全部 硬币都找给了他。 现在,请你不要管那天女店主怎么会在找零上屡屡遇到麻烦,这三位男士中谁用1美 元的纸币付了糖果钱? ——摘自G.J.Summers的“逻辑推理新题”(50) 七、 有一条河,河岸边有猎人,狼,还有一个男人,带两个小孩.还有一个女人,带两个小孩, 如果猎人离开,狼就把所有的人全部吃掉,如果男人离开,女人就把她的两个小孩掐死, 如果女人离开同上.河里有一条船,船上只能做两个人(附加条件:只有猎人,男人,女人 会划船).问:这八个人如何过河(都在河一边,狼也算一个) 八、 p先生、q先生都具有足够的推理能力。这天,他们正在接受推理面试。 他们知道桌子的抽屉里有如下16张扑克牌: 红桃 a、q、4 黑桃 j、8、4、2、7、3 草花 k、q、5、4、6 方块 a、5 约翰教授从这16张牌中挑出一张牌来,并把这张牌的点数告诉p先生,把这张牌 的花色告诉q先生。 这时,约翰教授问p先生和q先生:你们能从已知的点数或花色中推知这张牌是什么牌吗? p先生:“我不知道这张牌。” q先生:“我知道你不知道这张牌。” p先生:“现在我知道这张牌了。” q先生:“我也知道了。” 请问:这张牌是什么牌? 答案:第一题,7和8 如果P头上是1,S当然知道自己头上就是2。S第一次说 "猜不到",就等于告诉P先生,你头上的不是1。 这时,如果S头上是2,P先生当然知道自己头上应当是3(1已经排除了), 可是,P说 "猜不到",就等于说S头上不是2。 第二次S又说猜不到,就等于说P先生头上不是3,如果 是,我头上一定是4,我就能猜到了。 P又说猜不到,说明S头上不是4。 S又说猜不到,说明P头上不是5。 P又说猜不到,说明S头上不是6。(理由同前) 以此类推,第四次是说猜到了就说明P头上是7,S头上是8 第二题: 其中一人,看到两人都是黑的,就会假设,若自己是白的帽子,那另外两个人中的一个(假设B)就能判断出自己带了黑帽子,因为如果不是第三个人就看到了两顶白帽子而会被释放(而他并没说自己看到了两顶白帽),由此第一个人可以推出自己带了黑帽子 第三题,前面自杀的两个是红眼睛,最后那个是蓝的 如果有两个人是蓝的的话,那第一天晚上,那个红眼睛的就会自杀了(她看到的是两个蓝眼睛的),但第一晚大家都没自杀 如果两个是红的,那其中一个红的就会假设如果自己的眼睛是蓝的,那自己看到的那个红眼的就会在第一晚自杀,而他没有,因此在第二天判断自己是红眼的;同样另一个红眼的也由此得出结论;这样最后一个根据前面的推理就能知道自己是蓝眼睛了 如果三个都是红眼睛的,就该在第三天同时自杀了。因为大家都在等对方的反应。 第四题 先进开关房间开2盏灯等几小时关掉一个, 再到另一个灯的房间有个亮,2个不亮但其中有个是热的 第五题 好难讲清楚啊 任选一个顶点,以左上角那个为例,先向右下方划,到右下角的顶点。 然后竖直向上划过右上角的顶点再往上两个点间隔的长度 接着往左下方向划过第一行中间的点和第一列中间的点,再延长至与第三行同行的位置 最后往右划过第三行剩下的两个点即可 第六题 美分的那道题目: 卢开始有3×10+25,账单为50 莫开始有50,账单为25 内德开始有5+25,账单为10 店主开始有10 此时满足1,2,3,4 第一次调换:卢拿3×10换内德的5+25 卢5+2×25内德3×10 第二次调换:卢拿2×25换莫的50 此时: 卢有50+5账单为50付完走人 莫有2×25账单为25付完走人 内德有3×10账单为10付完剩20,要买5分的糖 付账后,店主有50+25+2×10,无法找开10,但硬币和为95,能找开纸币1元。 第七题 猎人带狼过去,自己回来(对岸:狼) 猎人带男孩过去,带狼回来(对岸:男孩) 男人带男孩过去,自己回来(对岸:两个男孩) 男人和女人过去,女人回来(对岸:男人和两个男孩) 猎人带狼过去,男人回来(对岸:猎人,狼和两个男孩) 男人和女人过去,女人回来(对岸:猎人,狼,男人和两个男孩) 女人带女孩过去,和男人回来(对岸:猎人,狼,女孩和两个男孩) 女人带女孩过去,自己回来(对岸:猎人,狼,两个女孩和两个男孩) 男人和女人过去(对岸:猎人,狼,女人,两个女孩,男人和两个男孩) 第八题 方块 5 逻辑推理题大全-- 1 1.粮食可以在收割前在期货市场进行交易。如果预测谷物产量不足,谷物期货价格就会上升;如果预测谷物丰收,谷物期货价格就会下降。今天早上,气象学家们预测从明天开始谷物产区里会有非常需要的降雨。因为充分的潮湿对目前谷物的存活非常重要,所以今天的谷物期货价格会大幅下降。   下面哪个,如果正确,最严重地削弱了以上的观点?   A.在关键的授粉阶段没有接受足够潮湿的谷物不会取得丰收。   B.本季度谷物期货价格的波动比上季度更加剧烈。   C.气象学家们预测的明天的降雨估计很可能会延伸到谷物产区以外。   D.农业专家们今天宣布,一种已经毁坏一些谷物作物的病菌在生长季节结束前会更广泛地传播。   E.许多在谷物期货市场交易的人很少实际拥有他们所交易的谷物。   02.   在一次选举中,统计显示,有人投了所有候选人的赞成票。   如果统计是真实的,那么下列哪项也必定是真实的?   A.对每个候选人来说,都有选民投了他的赞成票。   B.对所有候选人都投赞成票的不止一人。   C.有人没有投所有候选人的赞成票。   D.不可能所有的候选人都当选。   E.所有的候选人都可以当选。   03.   针对某种溃疡最常用的一种疗法可在6个月内将44%的患者的溃疡完全治愈。针对这种溃疡的一种新疗法在6个月的试验中使治疗的80%的溃疡取得了明显改善,61%的溃疡得到了痊愈。由于该试验只治疗了那些病情比较严重的溃疡,因此这种新法显然在疗效方面比最常用的疗法更显著。   对下列哪一项的回答最能有效地对上文论述做出评价?   (A)这两种疗法使用的方法有何不同?   (B)这两种疗法的使用成本是否存在很大差别?   (C)在6个月中以最常用疗法治疗的该种溃疡的患者中,有多大比例取得了明显康复?   (D)这种溃疡如果不进行治疗的话,病情显著恶化的速度有多快?   (E)在参加6个月的新疗法试验的患者中,有多大比例的人对康复的比例不满意?   04.   由风险资本家融资的初创公司比通过其他渠道融资的公司失败率要低。所以,与诸如企业家个人素质、战略规划质量或公司管理结构等因素相比,融资渠道在初创公司的成功上是更为重要的原因。   下面哪个,如果正确,最严重地削弱了以上的结论?   A.风险资本家对初创公司财务需要的变化比其他融资渠道更为敏感。   B.在公司的长期成功方面,初创公司的战略规划比起企业家个人素质来说是一个相对不重要的因素。   C.所有的新公司中有超过一半在5年内倒闭了。   D.一般来讲,初创公司的管理结构不如发展中的公司正式。   E.风险资本家在决定为初创公司提供资金时依据以下因素,如企业家素质和公司的战略规划质量。   05.   高塔公司是一家占用几栋办公楼的公司,它现在考虑在它所有的建筑内都安装节能灯泡,这种新灯泡与目前正在使用的传统灯泡发出同样多的光,而所需的电量仅是传统灯泡的一半。这种新灯泡的寿命也大大加长,因此通过在旧灯泡坏掉的时候换上这种新灯泡,高塔公司可以大大地降低其总体照明的成本。   下列哪一项,如果正确,最能支持上面论述?   (A)如果广泛地采用这种新灯泡,这是非常可能的,那么新灯泡的产量就会大大增加,从而使其价格与那些传统灯泡相当。 折扣。   (C)高塔公司最近签订了一份 合同 劳动合同范本免费下载装修合同范本免费下载租赁合同免费下载房屋买卖合同下载劳务合同范本下载 ,要再占用一栋小的办公楼。   (D)高塔公司发起了一项运动,鼓励其员工每次在离开房间时关灯。   (E)生产这种新灯泡的公司对灯泡中使用的革新技术取得了专利,因此它享有生产新灯泡的独家权利。   06.   社会撰写人:因为青少年缺乏基本的开车技巧,所以应给予青少年的驾驶执照附加限制。尽管19岁和再小一点的司机只占注册司机的7%,但是他们却是超过14%的交通死亡事故的肇事者。   下面每一项,如果正确,都能削弱青少年缺乏基本的开车技巧的论述,除了:   A.与其他人开的车相比,青少年开的车较旧,且稳定性也差。   B.青少年司机和他们的乘客使用座带和肩带的可能性不如其他人的大。C.青少年司机平均每年开车的距离超过其他司机的两倍。   D.青少年引起的交通事故比其他人引起的交通事故严重。   E.青少年开车时的乘客人数很有可能比一般的司机多。   07.   某一市长曾建议向进城的私人车辆每天收取五美元的费用,宣称这种费用的征收将缓解该城市的交通拥挤状况。该市长解释说,由于该费用比许多附近站点乘坐环线公共汽车的费用要高,许多人会由自己驾驶汽车转为乘坐公共汽车。   以下哪一项陈述,如果是正确的,为证明该市长的推理是有缺陷的提供了最好的证据?   (A)汽油价格的预期上升将提高进城的私人车辆的成本。   (B)停车费用已经使大多数开私人车辆进城的人觉得比乘坐公共汽车昂贵很多了。   (C)目前乘坐公共汽车的人多数没有自己的私家车。   (D)许多反对该市长 计划 项目进度计划表范例计划下载计划下载计划下载课程教学计划下载 的通勤者指出他们宁愿忍受交通拥挤,也不愿付每天五美元的费用。   (E)在一般的工作日,居住在城区里的人拥有和驾驶的私人车辆占到了该城整个交通流量的20%。   08.   国会削减社会福利费看来会损害穷人的利益,其实不会。因为社会福利预   算削减的同时,税收也削减。因此,每个人手中的钱将变多,而不是变少.以下哪项能动摇上述论证?   (l)穷人正在敦促国会提高社会福利预算。   (2)穷人本来几乎不纳税或者纳很少的税,因此,税收削减对他们来说没有多大好处。   (3)穷人因税收削减所得到的好处,补偿不了因福利费削减带来的损失。   A.仅仅(1)。   B.仅仅(2)。   C.仅仅(3)。   D.仅仅(2)和(3)。   E.(1)、(2)和(3)。   09.   反核活动家:关闭这个核电站是反核的胜利,它同时也体现了核工业很迟才肯承认它们不能安全运作这样的发电站的事实。   核电站经理:它并不体现这样的事情。从非核资源可得到便宜的电力,加   上强制性的安全检查和安全维修,使继续运作变得不经济。因此,不是出于安全方面的考虑,而是出于经济方面的考虑,才下令关闭了这个核电站。   经理的论证的推论是有缺陷的,因为该论证   A.没有承认即使这家核电站不是出于安全方面的原因则被关闭,电力公司现在也可能会认为核电站是不安全的。   B.忽略了那些可以利用的便宜电力资源本身也可能存在安全问题的可能   性。   C.把关闭这个核电站对公众来说体现了什么的问题错认为是经理的关闭理由是什么的问题。   D.把电力工业对待核安全的态度与反核活动家的观点相抵触的态度的观点作为它的一个前提。   E.把由于需要采取安全预防措施而引起的一些费用的上升看作是纯粹的经济上的因素。   10.   赵明、钱红、孙杰三人被北京大学、清华大学和北京师范大学录取。他们分别被哪个学校录取的,同学们作了如下的猜测:   同学甲猜:赵明被清华大学录取,孙杰被北京师范大学录取。   同学乙猜:赵明被北京师范大学录取,钱红被清华大学录取。   同学丙猜:赵明被北京大学录取,孙杰被清华大学录取。   结果,同学们的猜测各对了一半。   那么,他们的录取情况是   A.赵明、钱红、孙杰分别被北京大学、清华大学和北京师范大学录取。   B.赵明、钱红、孙杰分别被清华大学、北京师范大学和北京大学录取。   C.赵明、钱红、孙杰分别被北京师范大学、清华大学和北京大学录取。   D.赵明、钱红、孙杰分别被北京大学、北京师范大学和清华大学录取。   E.赵明、钱红、孙杰分别被清华大学、北京大学和北京师范大学录取。   11.   电学工程师已反复重申,最好的晶体管扩音机与最好的电子管扩音机在通常测量评价扩音机的音乐再现质量方面的性能是一样的。因此那些坚持认为录制的音乐在最好的电子管扩音机里播放时要比在最好的晶体管扩音机里播放时听起来好的音乐爱好者,一定是在想象他们声称的听到的质量上的差异。   下面哪一点,如果正确,最能严重削弱上述辩论?   A.许多人仅凭耳听不能区分正在播放的音乐是在好的晶体管扩音机里播放还是在好的电子管扩音机里播放。   B.电子管扩音机的音乐再现质量的变化范围要比晶体管扩音机的大。   C.有些重要的决定音乐听起来怎么样的特性不能被测量出来。   D.当放出相同的音量时,晶体管扩音机比电子管扩音机的体积小,用电少且产生的热量少。   E.在试验室里通常测定的用以评价扩音机的音乐再现质量的特性方面,有些电子管扩音机明显地比晶体管扩音机好。   12.   室外音乐会的组织者宣布,明天的音乐会将如期举行,除非预报了坏天气或预售票卖得太少了。如果音乐会被取消,将给已买了票的人退款。尽管预售票已卖得足够多,但仍有一些已买了票的人已经得到了退款,这一定是因为预报了坏天气的缘故。   下列哪一项是该论述中含有的推理错误?   (A)该推理认为如果一个原因自身足以导致其一结果,那么导致这个结果的原因只能是它。   (B)该推理将已知需要两个前提条件才能成立的结论建立在仅与这两个条件中的一个有关系的论据基础之上。   (C)该推理仍解释说其中一事件是由另一事件引起的,即使这两件事都是由第三件未知的事件引起的。   (D)该推理把缺少某一事件会发生的一项条件的证据当作了该事件不会发生的结论性证据。   (E)试图证明该结论的证据实际上削弱了该结论。   13.   如果小赵去旅游,那么小钱、小孙和小李将一起去。   如果上述断定是真的,那么,以下哪项也是真的?   A.如果小赵没去旅游,那么小钱、小孙、小李三人中至少有一人没去。   B.如果小赵没去旅游,那么小钱、小孙、小李三人都没去。   C.如果小钱、小孙、小李都去旅游,那么小赵也去   D.如果小李没去旅游,那么小钱和小孙不会都去。 br>   14.   甲:最近,我被一家航空公司的某一航班拒绝了一个我已经确认过的预定座位,因为这家航空公司超额预定了那个航班。因此,我被迫乘下一班可乘的航班,该航班两个小时后才起飞,我错过了一个非常重要的商业会议。即使我预定的那个航班在最后一分钟因为天气原因而被取消,航空公司也应该因没能让我乘坐那个航班而给我赔偿。   乙:从道义上来说,航空公司没有给你赔偿的责任,即使你没被拒绝乘坐早一点的航班,无论如何你都会错过你的商业会议。   下面哪一条原则,如果正确,能证明乙对甲的反应,即从道义上讲航空公司有责任赔偿那些在某一航班上确认了预定座位而又被拒绝乘坐该航班的乘客是合理的?   A.如果迫使乘坐晚一点航班的惟一原因是航空公司已超额预定了那次航班。   B.只有当乘客被迫乘坐晚一点的航班的原因不是因为天气恶劣而取消了该航班。   C.只有当航空公司没有超额预定最初的那次航班,乘客也没有被迫乘坐晚一点的航班。   D.即使乘客被迫乘坐晚一点的航班的惟一原因是航空公司因为天气不好而取消了最初的那次航班。   E.即使在航空公司没有超额预定最初的那次航班的情况下,乘客仍被迫乘坐晚一点的航班。   15.   软饮料制造商:我们的新型儿童软饮料力比咖增加了钙的含量。由于钙对形成健康的骨骼非常重要,所以经常饮用力比咖会使孩子更加健康。   消费者代表:但力比咖中同时含有大量的糖份,经常食用大量的糖是不利于健康的,尤其是对孩子。   在对软饮料制造商的回应中,消费者代表做了下列哪一项?   (A)对制造商宣称的钙元素在儿童饮食中的营养价值提出质疑。   (B)争论说如果对制造商引用的证据加以正确地考虑,会得出完全相反的结论。   (C)暗示产品制造商通常对该产品的营养价值毫不关心。   (D)怀疑某种物质是否在适度食用时有利于健康,而过度食用时则对健康有害。   (E)举出其他事实以向制造商所做的结论提出质疑。   16.   有确凿的证据显示,偏头痛(严重的周期性头痛)不是由于心理上的原因引起的,而是完全由生理上的原因所致。然而,数项研究结果表明那些因为偏头痛受到专业化治疗的人患有 标准 excel标准偏差excel标准偏差函数exl标准差函数国标检验抽样标准表免费下载红头文件格式标准下载 心理尺度的焦虑症的比率比那些没经专业治疗的偏头痛患者的高。   下面哪一点,如果正确,最能有助于解决上面论述中的明显矛盾?   A.那些患有偏头痛的人,倾向于有患偏头痛的亲戚。   B.那些患偏头痛的人,在情绪紧张时经常头痛。   C.那些患有标准心理尺度的焦虑症且发作率较高的人追求专业治疗的可能性要比那些在同样尺度上发作率较低的人大。   D.在许多有关偏头痛起因的研究中,大多数认为偏头痛是由像焦虑这样的心理因素引起的研究已被广泛宣传。   E.不管他们的医生认为偏头痛的起因是心理方面的,还是生理方面的,大多数患有偏头痛且追求专业治疗的人在他们停止患有偏头痛后仍坚持治疗。   17.   小王、小李、小张准备去爬山。天气预报说,今天可能下雨。围绕天气预报,三个人争论起来。   小王。“今天可能下雨,那并不排斥今天也可能不下雨,我们还是去爬山吧。   小李:“今天可能下雨,那就表明今天要下雨,我们还是不去爬山了吧。”   小张:“今天可能下雨,只是表明今天不下雨不具有必然性去不去爬山由你们决定。”   对天气预报的理解,三个人中   A.小王和小张正确,小李不正确。   B.小王正确,小李和小张不正确。 <   E.如果小孙没去旅游,那么小赵和小李不会都去。 < br>  C.小李正确,小王和小张不正确。   D.小张正确,小王和小李不正确。   E.小李和小张正确,小王不正确。   18.   如果一项投资不能产生利润,那么以投资为基础的减轻赋税就是毫无用处的。任何一位担心新资产不会赚钱的公司经理都不会因减轻公司本来就不欠的税款的允诺而得到安慰。   下面哪项是从上文得出的最可靠的推论?   (A)阻止效益不佳的投资的最有效的方法是对可以产生利润的投资减轻税赋。   (B)公司经理在决定他们认为可以盈利的投资时,可能会不考虑税款问题。   (C)对新投资减轻税款的承诺本身不会刺激新投资。   (D)公司经理把税款问题的重要性看得越小,他就越可能正确地预测投资的有利性。   (E)公司投资决策的一个关键因素可能是公司经理对感知到的商业状况的心理反应。   19.   一块石头被石匠修整后,曝露于自然环境中时,一层泥土和其他的矿物便逐渐地开始在刚修整过的石头的表面聚集。这层泥土和矿物被称作岩石覆盖层。在一安迪斯纪念碑的石头的覆盖层下面,发现了被埋藏一千多年的有机物质。因为那些有机物质肯定是在石头被修理后不久就生长到它上面的,也就是说,那个纪念碑是在1492年欧洲人到达美洲之前很早建造的。   下面哪一点,如果正确,能最严重地削弱上述论述?   A.岩石覆盖层自身就含有有机物质。   B.在安迪斯,1492年前后重新使用古人修理过的石头的现象非常普遍。   C.安迪斯纪念碑与在西亚古代遗址发现的纪念碑极为相似。   D.最早的关于安迪斯纪念碑的书面资料始于1778年。   E.贮存在干燥和封闭地方的修理过的石头表现,倘若能形成岩石覆盖层的话,形成的速度也会非常地慢。   20.   根据医学资料记载,全球癌症的发病率20世纪下半叶比上半叶增长了近10倍,成为威胁人类生命的第一杀手。这说明,20世纪下半叶以高科技为标志的经济迅猛发展所造成的全球性生态失衡是诱发癌症的重要原因。   以下各项,如果是真的,都能削弱上述论证,除了   A.人类的平均寿命,20世纪初约为30岁,20世纪中叶约为4O岁,目前约为65岁,癌症发病率高的发达国家的人均寿命普遍超过70岁。   B.20世纪上半叶,人类经历了两次世界大战,大量的青壮年人口死于战争;而20世纪下半叶,世界基本处于和平发展时期。   C.高科技极大地提高了医疗诊断的准确率和这种准确的医疗诊断在世界范围的覆盖率。   D.高科技极大地提高了人类预防、早期发现和诊治癌症的能力,有效地延长着癌症病人的生命时间。   E.从世界范围来看,医学资料的覆盖面和保存完好率,20世纪上半叶大约分别只有20世纪下半叶的50%和70%。   21.   一份关于酸雨的报告总结道,“加拿大的大多数森林没有被酸雨损害。”这份报告的批评者坚持认为这一结论必须改变为,“加拿大的大多数森林没有显示出明显的被酸雨损害的症状,如不正常的落叶、生长速度的减慢或者更高的死亡率。”   下面哪个,如果正确,为批评者坚持要改变报告结论提供了逻辑上最强有力的正当理由?   A.加拿大的一些森林正在被酸雨损害。   B.酸雨可能正在造成症状尚未明显的损害。   C.报告没有把酸雨对加拿大森林的损害与酸雨对其他国家森林的损害进行比较。   D.过去的15年内,加拿大所有森林都下过酸雨。   E.酸雨造成的损害程度在不同森林之间具有差异   (B)向高塔公司提供电力的公共事业公司向其最大的客户们提供>   22.   近期土地价格的下跌已经使许多在房地产上大量投资的机构受到了损害。去年,在这次价格下跌尚未开始的时候,一所地方大学为其资产增加了200英亩的土地。当然,这所大学并未购买这块土地,而是作为馈赠接受下来的。所以价格下降并没有影响到该大学。   下面哪个,如果正确,对以上的结论提出了最严重的质疑?   A.去年给予这所大学的200亩土地与该所大学处于同一社区。   B.与房地产馈赠相比,这所大学经常接受更多的资金损赠。   C.这所大学所处地区目前的土地价格要高于全国的平均水平。   D.去年,这所大学预算用来进行翻修的资金包括今年出售一些土地的预期收入。   E.去年,这所大学没有交纳学校建筑物所占土地的地产税,相反却付费补偿地方政府所提供的服务。   23.   在某餐馆中,所有的菜或者属于川菜系或者属于粤菜系,张先生点的菜中有川菜,因此,张先生点的菜中没有粤菜。   以下哪项最能增强上述论证?   A.餐馆规定,点粤菜就不能点川菜,反之亦然。   B.餐馆规定,如果点了川菜,可以不点粤菜,但点了粤菜,一定也要点川菜。   C.张先生是四川人,只喜欢川菜。   D.张先生是广东人,但不喜欢粤菜。   E.张先生是四川人,最不喜欢粤菜。   24.   飞机制造商:我反对你把我们的X-387型喷气机描述为危险的。商业使用的X-387飞机从未坠毁,也未曾有过严重的功能失调。   航空调度员:X-387飞机的问题并不在于其自身,而在于发动起来时会引起空气湍流,给附近的飞行器造成危险的环境。   航空调度员通过下面哪一个对制造者做出了回答?   A.把制造商的论断特征描述为来自主观兴趣,而不是来自于对事实的客观评价。   B.把注意力集中于这个事实:制造商对“危险”的阐释太狭隘了。   C.引用一些制造商把它们当作与争论问题无关而明显忽略的证据。   D.引用统计证据以反驳制造商的断言。   E.向制造商对最近空难数量的了解程度提出质疑。   25.   某些媒分子通过使环绕肺气管的骨肉细胞收缩来抵御有毒气体对肺部的损害。这使得肺部部分封闭起来。当这些媒分子被不必要的激活时,对某些无害的像花粉或家庭粉尘作出反应,就出现了哮喘病。   有一项计划是开发一种药物通过阻碍接收由上文所说的媒分子发出的信息来防止哮喘病的发生。以下哪一项,如果是正确的,将指出这项计划的最严重的缺陷?   (A)研究人员仍不知身体是如何产生这种引发哮喘病的媒分子的。   (B)研究人员仍不知是什么使一个人的媒分子比其他人的更易激活。   (C)很多年内无法获得这样的药物,因为开发和生产这种药物都需要很长的时间。   (D)这样的药物无法区分由花粉和家庭粉尘引发的信息与由有毒气体引发的信息。   (E)这样的药物只能是预防性的,一旦得上哮喘,它无法减轻哮喘的程度。 参考答案 01.D 02.A 03.C 04.E 05.A 06.D 07.B 08.D 09.E 10.A 11.C 12.A 13.E 14.C 15.E 16.C 17.A 8.C19.B20.B21.B22.D23.A24.B25.D 逻辑推理题大全-- 2 01.环境科学家:在过去的10年中,政府对保护湿地的投资确实增加了6倍,而同时需要这样保护的土地面积只增加了两倍(尽管这些区域在10年前已经很大了)。即使把通货膨胀考虑进去,今天的资金数额也至少是10年前的3倍。虽然如此,目前政府对保护湿地的投资仍是不够的,政府的投资应该进一步增加。   下面哪一点,如果正确,最有助于使环境科学家的结论与引用的证据相一致?   A.负责管理湿地保护资金的政府机构在过去的10年中一直管理不当且运行效率较低。   B.在过去的10年中,那些被政府雇来保护湿地的科学家的薪水的增长比率高于通货膨胀的比率。   C.过去10年的研究使今天的科学有在潮湿土地遭到严重破坏的危险之前就把它们定为需要保护的对象。   D.今天,有更多的像科学家和非科学家的人在为保护包括湿地在内的自然资源而工作。   E.不像今天,10年以前对保护湿地的投资几乎是不存在的。   02.   某餐馆发生一起谋杀案,经调查:   第一,谋杀或者用的是叉,或者用的是刀,二者必居其一。   第二,谋杀时间或者在午夜12点,或者在凌晨4点。   第三,谋杀者或者是甲,或者是乙,二者必居其一。   如果以上断定是真的,那么以下哪项也一定是真的?   (1)死者不是甲用叉在午夜12点谋杀的,因此,死者是乙用刀子在凌晨4点谋杀的。   (2)死者是甲用叉在凌晨4点谋杀的,因此,死者不是乙用叉在凌晨4点谋   杀的。   (3)谋杀的时间是午夜12点,但不是甲用叉子谋杀的,因此,一定是乙用刀子谋杀的。   A.仅(l)。   B.仅(2)。   C.仅(3)。   D.(1)、(2)、(3)。   E.(2)和(3)。   03.   引自一篇报纸社论:许多有海洛因瘾的人最终都会试图戒毒,这主要基于两个原因:维持吸毒的开支和害怕被捕。如果海洛因被合法化且可以廉价取得,正像一些人所鼓吹的,那么这两个原因都不适用了。   以上考虑可最好地用于下列哪一论述?   A.使海洛因销售合法化可能导致其价格下降。   B.使用海洛因瘾的人容易获得戒毒治疗可能鼓励许多有海洛因瘾的人努力戒毒。   C.使海洛因销售合法化可能增加有海洛因瘾的人为买毒品而造成的犯罪。   D.使海洛因能够合法且廉价地获得,将使那些对海洛因上瘾的人不大可能试图戒掉他们的毒瘾。   E.降低对吸海洛因的个人惩罚的力度,不会增加新的染海洛因瘾的人数。   04.   有人向某制衣厂经理提出一项建议:在机器上换上大号的缝纫线团,这样就可不必经常停机换线团,有利于减少劳动力成本。   这一建议预设了以下哪项?   A.大号缝纫线团不如小号的结实。   B.该厂实行的是计时工资制,不是计件工资制。   C.缝纫机器不必定期停机保养 检修 外浮顶储罐检修方案皮带检修培训教材1变电设备检修规程sf6断路器检修维护检修规程柴油发电机 。   D.操作工人在工作期间不允许离开机器。   E.加快生产速度有利于提高该厂生产的衣服的质量。   05.   根据古代记录,S市政府对基本商品征收的第一种税是对在S市出售的每一罐食用油征税两个生丁。税务纪录显示,尽管人口数量保持稳定且税法执行有力,食用油的税收额在税法生效的头两年中还是显著下降了。下列哪一项,如果正确,最有助于解释在S市油税收入的下降?   A.在税法实施后的10年,S市的平均家庭收入稳定增加。   B.在食用油税实行后的两年,S市政府开始在许多其他基本商品上征税。   C.在S市,食用油罐传统上被用作结婚礼物,在税法实施后,食用油的礼物   增多了。   D.S市的商品,在税法实施后开始用比以前更大的罐子售油。   E.很少S市的家庭在加税后开始生产他们自己的食用油。   06.   每年,一个消费者机构将所有国内航空公司按照在过去一年里飞机起降准时的表现排名,它所使用的惟一标准是每个航空公司晚点不超过15分钟的航班的比率。该机构不将因机械故障造成的延误计算在内,但去年因机械故障造成延误的航班的比率在所有国内航空公司中大致相同,这一事实说明   下列哪项能最合逻辑地完成以上论述?   A.将因机构故障造成的延误包括在航空公司起降准时表现排名的计算之内,如果有影响的话,那么它对去年的排名只有极小的影响。   B.如果把机构故障造成的延误包括在准时表现的排名中,航空公司可能努力工作来减少延误。   C.该机构的排名没有给予消费者关于一个航空公司与另一家航空公司相比较关于其晚点的比率的准确信息。   D.去年有最好的准时表现记录的航空公司同时也有最大数目的机械原因造成的晚点。   E.去年准时表现对于所有国内航空公司大约相同。   07.   某银行被窃,甲、乙、丙、了四人涉嫌被拘审。侦破结果表明,罪犯就是其中的某一个人。   甲说:“是丙渝的。”   乙说:“我没偷。”   内说:‘我也没偷。”   丁说;‘如果乙没有偷,那么就是我偷的。”   现已查明,其中只有一个说假话。从上述条件可以确定以下哪项成立?   A.甲份。   B.乙份。   C.丙偷。   D.丁偷。   E.推不出何人偷。   08.   永久型赛马场的休闲用骑乘每年都要拆卸一次,供独立顾问们进行安全检查。流动型赛马场每个月迁移一次,所以可以在长达几年的时间里逃过安全检查网及独立检查,因此,在流动型赛马场骑马比在永久型赛马场骑马更加危险。   下列哪一项,如果对于流动型赛马场而言是正确的,最能削弱上面论述?   A.在每次迁移前,管理员们都拆卸其骑乘,检查并修复潜在的危险源,如磨损的滚珠轴承。   B.它们的经理们拥有的用于安全方面及维护骑乘的资金要少于永久型赛马场的经理们。   C.由于它们可用迁徙以寻找新的顾客,建立安全方面的良好信誉对于他们而言不是特别重要。   D.在它们迁移时,赛马场无法接收到来自它们的骑乘生产商的设备回收通知。   E.骑乘的管理员们经常忽视骑乘管理的操作指南。   09.   每次核聚变都会发射出中子。为了检验一项关于太阳内部核聚变频繁程度的假设,物理学家们计算了在核假设正确的条件下,每年可能产生的中子数。他们再从这一点出发,计算出在地球某一特定地点应该经过的中子数。事实上,点数到的经过该地点的中子数要比预计的少得多,看起来这一事实证明了该假设是错误的,除了。   下列哪一项,如果正确,最能从逻辑上将上段补充完整?   A.物理学家们应用了另一种方法来估计可能到达该地点的中子数,结果验证了他们最初的估计。r>   B.关于太阳核聚变反应频率还存在着其他几种竞争性假设。   C.太阳内部没有足够的能量来破坏它释放出的中子。   D.用来点数中子的方法仅发现了约不足10%的通过该地区的中子。   E.其他星球核聚变反应所出发的中子也到达了地球。   10.   小张约小李第二天去商场,小李说:“如果明天不下雨,我去爬山。”第二天,天下起了毛毛细雨,小张以为小李不会去爬山了,就去小李的宿舍找他,谁知小李仍然去爬山了。待两人又见面时,小张责怪小李食言,既然天下雨了,为什么还去爬山;小李却说,他并没有食言,是小张的推论不合逻辑。   对于两人的争论,下面哪项论断是合适的?   A.小张和小李的这个争论是没有意义的。   B.小张的推论不合逻辑。   C.两个人对毛毛细雨的理解不同。   D.由于小李食言,引起了这场争论。   E.由于小李的表达不够明确,引起了这场争论   11.   欧洲的舞蹈批评家:美国芭蕾水平的提高是由于现在有更多的欧洲人在美国教授芭蕾。在美国的芭蕾教师中,在欧洲出生并接受训练的教师比例上升了,我知道这一点是因为在我去年去纽约时,我所遇见的欧洲来的芭蕾教师——在欧洲出生并接受训练——比从前要多。   下列哪一项指出了该舞蹈批评家在推理中所使用的一项有疑问的假设?   A.该论述忽视了一种可能,即美国的一些芭蕾教师可能出生在欧洲但却是在美国接受的训练。   B.该论述假设该批评家在其去年去纽约时遇见的教师群在这类教师中具有典型代表性。   C.该论述假设美国的芭蕾教学水平比欧洲的要高。   D.没有考虑导致美国舞蹈家思想状态水平上升的其他可能原因。   E.该论述假设在欧洲出生并受识破的舞蹈家一般比在美国出生并受识破的舞蹈家天赋更高。   12.   为保护海边建筑免遭海洋风暴的袭击,海洋度假地在海滩和建筑之间建起了巨大的防海墙。这些防海墙不仅遮住了一些建筑物的海景,而且使海岸本身也变窄了。这是因为在风暴从水的一边对沙子进行侵蚀的时候,沙子不再向内陆扩展。   如果上述信息正确,那些从其出发,下列哪一项得到了最有力的支持?   A.由于海洋风暴的猛烈程度不断加深,必须在海洋和海边财产之间建立起更多的高大防海墙。   B.即使是在海滩被人类滥用着的时候,它们对于许多使用它们的野生物种的生存来说依然是必不可少的。   C.用来保护海边建筑的防海墙如果要保护那些建筑,它们自己最终不会被风暴破坏,也不需要昂贵的维修和更新。   D.为以后的世代保留下海滩应该是海岸管理的首要目标。   E.对于一个想要维护自己海滩疗养地功能的海边社区来说,通过建筑防海墙来保护海边建筑的努力,从长远来看,其作用是适得其反的。   13.   甲、乙、丙三人讨论“不劳动者不得食”这一原则所包含的意义。   甲说:“不劳动者不得食,意味着得食者可以不劳动。”   乙说:‘“不劳动者不得食,意味着得食者必须是劳动者。”   丙说:“不劳动者不得食,意味着得食者可能是劳动者。”   以下哪项结论是正确的?   A.甲的意见正确,乙和丙的意见不正确。   B.乙和丙的意见正确,甲的意见不正确。   C.甲和丙的意见正确,乙的意见不正确。 r>  E.丙的意见正确,甲和己的意见不正确。   14.   加拿大人现在越来越多地加入“境外购物”,即越过国界到价格较低的地方购物。加拿大以外的价格要低很多,很大一部分原因是支付给加拿大社会服务体系的商品和服务税不再适用。   根据以上的信息,下面哪个说法最可以被支持?   A.如果境外购物的上升趋势继续保持在较高的水平,并且政府支付给加拿大社会服务体系的金额不变,估计加拿大的商品和服务税税率就会上升。   B.如果加拿大对从境外购买的商品征收较多的关税,另一个方向上别国也会相应地对从加拿大购买的商品征收关税,从而损害加拿大的商业。   C.加拿大政府支付给为加拿大人提供社会服务的人的资金数额一直在增加。   D.同样品牌的商品,加拿大顾客在境外和在国内都可以买到。   E.境外购物所购商品在购物者越过边界进入加拿大境内时,要交纳加拿大规定的税收。   15.   某国政府决策者面临的一个头痛的问题就是所谓的“别在我家门口”综合症。例如,尽管民意测验一次又一次地显示公众大多数都赞成建造新的监狱,但是,当决策者正式宣布计划要在某地建造一新的监狱时,总遭到附近居民的抗议,并且抗议者往往总有办法使计划搁浅。   以下哪项也属于上面所说的“别在我家门口”综合症?   A.某家长主张,感染了艾滋病毒的孩子不能允许入公共学校。当知道一个感染了艾滋病毒的孩子进入了他孩子的学校,他立即办理了自己孩子的退学手续。   B.某政客主张所有政府官员必须履行个人财产公开登记,他自己递交了一份虚假的财产登记表。   C.某教授主张宗教团体有义务从事慈善事业,但自己拒绝捐款资助索马里饥民。   D.某汽车商主张国际汽车自由贸易,以有利于各国经济,但要求本国政府限制外国制造的汽车进口。   E.某军事战略家认为核战争足以毁灭人类,但主张本国保持足够的核能力以抵御外部可能的核袭击。   16.   左撇子的人比右撇子的人更经常患有免疫功能失调症,比如过敏。但是左撇子往往在完成由大脑右半球控制的任务上比右撇子具有优势,并且大多数人的数学推理能力都受到大脑右半球的强烈影响。   如果以上的信息正确,它最能支持下面哪个假设?   A.大多数患有过敏或其他免疫功能失调症的人是左撇子而非右撇子。   B.大多数左撇子的数学家患有某种过敏症。   C.数学推理能力强于平均水平的人中,左撇子的人的比例,要高于数学推理能力弱于平均水平的人中的左撇子比例。   D.如果一位左撇子患有过敏症,他很可能擅长数学。   E.比起左撇子的人或者数学推理能力不寻常地好的人所占的比例来讲,患有过敏等免疫功能失调症的人的比例要高一些。   17.   M公司的最新产品成本是如此之低,以至于公司不大可能在出售产品时不增加公司通常允许赚取的成本加价:潜在的客户可能完全不能相信这么便宜的东西会真好使。但M公司的信誉是建立在仅包括合理的边际利润的公平价格基础上的。   以上的论述,如果正确,最强有力地支持了下面哪个?   A.M公司在试图为其最新产品定价、使价格能在不损害公司信誉的前提下促进销售时会遇到困难。   B.尽管售出的每件产品利润很小,但通过大规模的销售,M公司仍取得了巨大的年利润。   C.M公司在为其最新产品计算生产成本时犯了计算错误。   D.M公司的最新产品将要执行的任务是其他制造成本更低的设备也能胜任的。   E.M公司的生产程序的设计和该公司制造的产品一样具有新颖之处18.   报上登出了国内20家大医院的名单,名单按它们在近3年中病人死亡率的高低排序。专家指出不能把名单排列的倾序作为评价这些医院的医疗水平的一个标准。   以下各项,如果是真的,都能作论据支持专家的结论,除了   A.这20家医院中,有5家依靠国家资助从国外进口了多项先进、大型和配套的医疗设备,其余的都没有。   B.有些医院,留病人住院的时间长,病人死亡率因此就较高;有些医院,往往较早地动员患绝症而救治无望的病人出院,病人死亡率因此就较低。   C.这20家医院中,有2家老人医院和3家儿童医院。   D.在20家医院中,有2家是肿瘤医院。   E.有些医院不具备特种手术和特别护理条件,碰到相关的病人就转院了事。   19.   大学图书管理员:三年以前,非学生读者使用本图书馆是免费的。后来,因为我们的预算减少了,所以我们要求他们每年支付100美元的费用。然而,仍然约有150名非学生读者使用了图书馆而没有缴费。因此,如果我们雇用一名警卫来辨认非学生读者并令其缴费,那么我们就可以获得经济收益。   下列哪一项对于评价图书管理员的结论是最为重要的?   (A)每年使用图书馆的学生人数。   (B)今年图书馆的预算。   (C)图书馆是否安装了昂贵的计算机分类系统。   (D)三年前图书馆的预算降低了多少。   (E)图书馆雇用一名警卫每年的成本是多少。   20-21题基于以下题干:   某仓库被窃。经过侦破,查明作某的人部是甲、乙、丙、丁四个人中的一个人。审讯中,四个人的口供如下:   甲:“仓库被窃的那一天,我在别的城市,因此我是不可能作案的。”   乙:“丁就是罪犯。”   丙:“乙是盗窃仓库的罪犯,因为我亲眼看见他那一天进过仓库。”   丁:“乙是有意陷害我。”   20.   现假定这四个人的口供中,只有一个人讲的是真话。那么   A.甲是盗窃仓库的罪犯。   B.乙是盗窃仓库的罪犯。   C.丙是盗窃仓库的罪犯。   D.丁是盗窃仓库的罪犯。   E.甲、乙、丙、丁都不是盗窃仓库的罪犯。   21.   现假定这四个人的口供中,只有一个人讲的是假话。那么   A.甲是盗窃仓库的罪犯。   B.乙是盗窃仓库的罪犯。   C.丙是盗窃仓库的罪犯。   D.丁是盗窃仓库的罪犯。   E.甲、乙、丙、丁都不是盗窃仓库的罪犯。   22.   要成为一名成功的商业经理需具备一定的天赋,商业课程可以帮助人们解决管理问题,但这种课程仅能帮助那些具有管理天赋的人,这些人应该通过商业课程来获得一些方法,如果这些管理问题恰巧发生时,他们就可以很好地应用这些方法。   如果上文论述正确,从其出发,下列哪一项也一定正确?   A.那些在解决管理问题方面受益于商业课程的人也具有管理天赋。   B.那些在解决管理问题方面已经很有办法的人不可能从商业课程中受益。   C.大多数成功地解决了管理问题的方法是从商业课程中学到的。 E.那些没有学过商业课程的人在管理问题发生时无法解决这些问题。   23.   需要一种特殊棉纱带的惟一目的是把某些外科伤口包扎10天——这些伤口需要包扎的最多天数。纽太是这种棉纱带的一个新品牌。纽太的销售人员声称纽太会有助于伤口愈合,因为纽太的粘附能力是现在使用的棉纱带的2倍。   下面哪种说法,如果正确,会对纽太的销售人员的声明提出最严重的疑问?   A.大多数外科伤口大约需要10天愈合。   B.大多数外科棉纱带是由医院和诊所而非私人外科医生购买的。   C.现在使用的棉纱带的粘附能力远超过包扎伤口10天所需的足够的粘附能力。   D.纽太和现在使用的棉纱带都不能很好地粘附未经洁净的皮肤。   E.纽太对已涂上一种特别化学药剂的皮肤的粘附能力仅为现在使用的棉纱带对那种皮肤的粘附能力的一半好。   24.   一项研究发现,1970年调查的孩子中有70%曾经有过牙洞,而在1985年的调查中,仅有50%的孩子子曾经有过牙洞。研究者们得出结论,在1970~1985年这段时间内,孩子们中的牙病比率降低了。   下列哪一项,如果为真,最能削减弱研究者们上面得出的结论?   A.牙洞是孩子们可能得的最普通的一种牙病。   B.被调查的孩子来自不同收入背景的家庭。   C.被调查的孩子是从那些与这些研究者们进行合作的老师的学生中选取的。   D.1970年以来,发现牙洞的技术水平得到了突飞猛进的提高。E.平均来说,1985年调查的孩子要比1970年调查的孩子的年龄要小。   25.   通过检查甲虫化石,一研究小组对英国在过去2.2万年内的气温提出了到目前为止最为详尽的描述。该研究小组对现存的生物化石进行挑选,并确定了它们的日期。当发现在同一地方发现的几种生物的个体属于同一时间段时,现存的甲虫类生物的已知忍受温度就可以被用来决定那个地方在那段时间内的夏季的最高温度。   研究者的论述过程依赖于下面哪一条假设?   A.甲虫忍耐温暖天气的能力比忍耐寒冷天气的能力强。   B.在同一地方发现的不同物种的化石属于不同的时期。   C.确定甲虫早期的方法比确定其他生物日期的方法准确。   D.一个地方某个时期的实际最高夏季气温与在那个地方那段时间发现的每种甲虫类生物的平均最高可忍受气温相等。   E.在过去的2.2万年的时间内,甲虫类生物的可忍受气温没有明显变化。   参考答案 01.E02.B03.D04.B05.D06.A 07.D08.A09.D10.B11.B 12.E13.D14.A15.D16.C17.A 18.A19.E20.A21.B22.A23.C24.E25.E 水平思考法 希望大家踊跃发言,将自己的思考过程写下来,供其他人参考。 1、水平思考法 有一家人决定搬进城里,于是去找房子。 全家三口,夫妻两个和一个5岁的孩子。他们跑了一天,直到傍晚,才好不容易看到一张公寓出租的广告。 他们赶紧跑去,房子出乎意料的好。于是,就前去敲门询问。 这时,温和的房东出来,对这三位客人从上到下地打量了一番。 丈夫豉起勇气问道:"这房屋出租吗?" 房东遗憾地说:"啊,实在对不起,我们公寓不招有孩子的住户。" 丈夫和妻子听了,一时不知如何是好,于是,他们默默地走开 了。 那5岁的孩子,把事情的经过从头至尾都看在眼里。那可爱的心灵在想:真的就没办法了? 他那红叶般的小手,又去敲房东的大门。这时,丈夫和妻子已走出5米来远,都回头望着。 门开了,房东又出来了。这孩子精神抖擞地说:...... 房东听了之后,高声笑了起来,决定把房子租给他们住。 问:这位5岁的小孩子说了什么话,终于说服了房东? 我的想法(首先我保证自己事先没有看过任何答案,朋奕是比较诚实的,但错了也希望大家能礼貌指出)是:小孩以自己身份去租,那么就符合房东条件了。2、篮球赛 在某次篮球比赛中,A组的甲队与乙队正在进行一场关键性比赛。对甲队来说,需要嬴乙队6分,才能在小组出线。现在离终场只有6秒钟了,但甲队只蠃了2分。要想在6秒钟内再赢乙队4分,显然是不可能的了。 这时,如果你是教练,你肯定不会甘心认输,如果允许你有一次叫停机会,你将给场上的队员出个什么主意,才有可能蠃乙队6分? 我的想法:让对方进球,然后加时再打。3、分油问题 有24斤油,今只有盛5斤、11斤和13斤的容器各一个,如何才能将油分成三等份? 我的想法:先把13斤的倒满,然后用13斤的倒满5斤,这时13斤中就有8斤,也就是1/3了,将这些到如11斤容器中。再用5斤和剩余的倒满13斤的,重新来一次,就完成了。4、第十三号大街 史密斯住在第十三号大街,这条大街上的房子的编号是从13号 到1300号。琼斯想知道史密斯所住的房子的号码。 琼斯问道:它小于500吗? 史密斯作了答复,但他讲了谎话。 琼斯问道:它是个平方数吗? 史密斯作了答复,但没有说真话。 琼斯问道:它是个立方数吗? 史密斯回答了并讲了真话。 琼斯说道:如果我知道第二位数是否是1,我就能告诉你那所房子的号码。 史密斯告诉了他第二位数是否是1,琼斯也讲了他所认为的号码。 但是,琼斯说错了。 史密斯住的房子是几号? 我的想法是:64号,首先想最简单的处理办法,这里一共有5个条件,能作为初步判断的只有前三个,那么前三个中最简单的就是第三个立方数的条件,假设为真,得出1~10的立方数,其中既符合平方数的也符合立方数的只有64和512,若大于500则只有512,小于500则64,但512中有1,若通过这个判断是512,那么就不会说错,所以初步判断是64。我判断既符合平方数又符合立方数的原因是如果只符合立方数或平方数其中一项,则会因为符合条件的选项太多而推测不出来,因此估计为两项同时符合,就没有考虑太多了。5.不同部落间的通婚 故事讲的是许多年前欠完美岛上的一件婚事。一个普卡部落人 (总讲真话的)同一个沃汰沃巴部落人(从不讲真话的)结婚。婚后,他们生了一个儿子。这个孩子长大后当然具有西利撤拉部落的性格(真话、假话或假话、真话交替着讲)。 这个婚姻是那么美满,以致夫妻双方在许多年中都受到了对方性格的影响。讲这个故事的时候,普卡部落的人已习惯于每讲三句真话 就讲一句假话,而沃汰沃巴部落的人,则己习惯于每讲三句假话就要 讲一句真话。 这一对家长同他们的儿子每人都有个部落号,号码各不相同。他们的名字分别叫塞西尔、伊夫琳、西德尼 (这些名字在这个岛上男女 通用)。 三个人各说了四句话,但这是不记名的谈话,还有待我们来推断 各组话是由谁讲的 (我们想,前普卡当然是讲一句假话、三句真话,而前沃汰沃巴则是讲一句真话、三句假话)。 他们讲的话如下: A:(1)塞西尔的号码是三人中最大的。(2)我过去是个普卡。(3)B是我的妻子。(4)我的号码比B的大22。 B:(1)A是我的儿子。(2)我的名字是塞西尔。(3)C的号码是54或78或81。(4)C过去是个沃汰沃巴。 C:(1)伊夫琳的号码比西德尼的大10。(2)A是我的父亲。(3)A的号码是66或68或103。(4)B过去是个普卡。 找出A、B、C三个人中谁是父亲、谁是母亲、谁是儿子,他们各自的名字以及他们的部落号。 我的想法啊:题目太长了,有点困,不想看,但好像看过很多类似的。明天再看。6、环球旅行 有人开始环球旅行了。可是,在地球上怎样才算"环球"呢?我很茫然,主要是弄不清 "环球旅行"的定义。后来我就假设:"只要是跨过地球上所有的经度线和纬度线,就可以算环球旅行。" 那么请问,在这样的假设下,环球旅行的最短路程大概是多少公里?不过,解这个题时,为了简化,可以把地球看做是一个正圆球,周长是4万公里。 我的想法:太简单了,也许是我想的太简单了,考虑一下南北极所有经线相交的特殊性,然后顺着南北极随便找一条经线走一圈就OK了,这样就能把所有的纬线跨过,然后在两个极点的时候把所有经线跨过。4万公里就是答案了。7、"15点"游戏 乡村庙会开始了。 今年搞了一种叫做 "15点"的游戏。艺人卡尼先生说:"来吧,老乡们。规则很简单,我们只要把硬 币轮流放在1到9这个数字上,谁先放都一样。你们放镍币,我放银元,谁首先把加起来为15的三个不同数字盖住,那么桌上的钱就全数归他。" 我们先看一下游戏的过程:某妇人先放,她把镍币放在7上,因为将7盖住,他人就不可再放了。其他一些数字也是如此。 卡尼把一块银元放在8上。 妇人第二次把镍币放在2上,这样她以为下一轮再用一枚镍币放在6上就可加为8,于是她以为就可蠃了。但艺人第二次把银元放 在6上,堵住了夫人的路。现在,他只要在下一轮把银元放在1上就可获胜了。 妇人看到这一威胁,便把镍币放在1上。 卡尼先生下一轮笑嘻嘻地把银元放到了4上。妇人看到他下次放到5上便可蠃了,就不得不再次堵住他的路,她把一枚镍币放在5上。 但是卡尼先生却把银元放在3上,因为8+4+3=15,所以他蠃 了。可怜的妇人输掉了这4枚镍币。 该镇的镇长先生被这种游戏所迷住,他断定是卡尼先生用了一种 秘密的方法,使他比赛时怎么也不会输掉,除非他不想蠃。 镇长彻夜末眠,想研究出这一秘密的方法。 突然他从床上跳了下来,"啊哈!我早知道那人有个秘密方法,我现在晓得他是怎么干的了。真的,顾客是没有办法蠃的。" 这位镇长找到了什么窍门?你或许能发现怎么同朋友们玩这种 "15点"游戏而不会输一盘。 我的想法:用最简单的思路,肯定是跟能组成15的任选三个无重复的组合有关,那么我们看:从9开始:9+1+5=15 9+2+4=158: 8+1+6=15 8+2+5=15 8+3+4=157: 7+2+6=15 7+3+5=15下面开始就是重复的了,也就是说能组成15的组合只有7对,只要对方选了一个数字后,可供的选择组合成15的选项仅有3组,那么只要记住这些组合,简单就可以取胜了。如果到这里还要解释你的智商就……9、尤克利地区的电话线路 直到去年,尤克利地区才消除了对电话的抵制情绪。虽然现在己 着手在安装电话,但是由于计划不周,进展比较缓慢。 直到今天,该地区的六个小镇之间的电话线路还很不完备。A镇同其他五个小镇之间都有电话线路;而B镇、C镇却只与其他四个小 镇有电话线路;D、E、F三个镇则只同其他三个小镇有电话线路。如果有完备的电话交换系统,上述现象是不难克服的。因为,如果在 A镇装个电话交换系统,A、B、C、D、E、F六个小镇都可以互相通话。但是,电话交换系统要等半年之后才能建成。在此之前,两个小镇之间必须装上直通线路才能互相通话。 现在,我们还知道D镇可以打电话到F镇。 请问:E镇可以打电话给哪三个小镇呢? ABC三个10,猜字母S先生:让我来猜你心中所想的字母,好吗? P先生:怎么猜? S先生:你先想好一个拼音字母,藏在心里。p先生:嗯,想好了。 S先生:现在我要问你几个问题。P先生:好,请问吧。 S先生:你所想的字母在CARTHORSE这个词中有吗? P先生:有的。 S先生:在SENATORIAL这个词中有吗?P先生:没有。 S先生:在INDETERMINABLES这个词中有吗? P先生:有的。 S先生:在REALISATON这个词中有吗? P先生:有的。 S先生:在ORCHESTRA这个词中有吗? P先生:没有。 S先生:在DISESTABLISHMENTARIANISM中有吗? P先生:有的。 S先生:我知道,你的回答有些是谎话,不过没关系,但你得告诉我,你上面的六个回答,有几个是真实的? P先生:三个。 S先生:行了,我已经知道你心中的字母是……。 我感觉:应该是H11、琼斯教授的奖章 琼斯教授在W学院开设 "思维学"课程,在每次课程结束时,他总要把一枚奖章奖给最优秀的学生。然而,有一年,珍妮、凯瑟 琳、汤姆三个学生并列地成为最优秀的学生。 琼斯教授打算用一次测验打破这个均势。 有一天,琼斯教授请这三个学生到自己的家里,对他们说:"我准备在你们每个人头上戴一顶红帽子或蓝帽子。在我叫你们把眼晴睁开以前,都不许把眼睛睁开来。" 琼斯教授在他们的头上各戴了一顶红帽子。琼斯说:"现在请你们把眼睛都睁开来,假如看到有人戴的是红帽子就举手,谁第一个推断出自己所戴帽子的颜色,就给谁奖章。" 三个人睁开眼睛后都举了手。一分钟后,珍妮喊道:"琼斯教授,我知道我戴的帽子是红色的。" 珍妮是怎样推论的? 我的想法:跟最后那个村子的人一样的推理方法,以前听过是打疯狗的故事,其实这些都是一样的,掌握了同一个方法就都懂了。 逻辑推理考试准备方法(长…)但绝对精品 一、两大推理方向   一道逻辑题的题干分为论据与结论,这两部分构成论证结构。所有逻辑题无非分为以下三类:一类是选项与论证结构发生各种关系,如假设、支持、削弱、解释关系等;一类是选项对题干论证结构做出评价,如指出论证结构的缺陷,指出论证的形式的相似性等;还有一类是:结论与选项是重合的,这类题主要指的是归纳题。   因此,MBA逻辑考题从解题思路上可分为"自下而上"与"自上而下"两个推理方向,下面分别举例说明:   (一)自下而上   假设、支持、反对、评价这四类题型的解题思路是"自下而上",即这四类题型都是让我们从下面寻求一个选项放到上面段落中对段落推理起到评价作用,因此段落是由一个"有待评价的推理"组成的,又加上段落推理的成立还有赖于一些隐含假设,因此根据不同的问题目的而对隐含假设起到一定的作用是解题的关键。逻辑题的推理过程,是指前提到结论的思维变迁。下图表达了这四类题的主要解题侧重点:   假设、支持、反对、评价这四类题所面临的是一个"待评价的推理",也就是从前提到结论的这个推理是否能成立我们是不知道的,需要对它进行评价。根据不同的问题目的,确定不同的解题方向:问支持,就找一个选项说它可能对,即将答案放在论据(前提)和结论之间,对段落推理或者结论有支持作用就可以,所以既可非充分又可非必要;问削弱,就找一个选项说它不一定对,即将答案放在论据(前提)和结论之间,对段落推理或者结论有驳斥作用就可以,所以也是既可非充分又可非必要;问假设,则考虑使结论成立需要什么,哪一个是使其成立的一个必要条件;问评价,就是要去寻找一个能对段落推理起到正反两方面作用的选项。下面就这四类题型分别举一例:   -瑞兰国的人口调查数据表明未婚的三十岁以下的瑞兰的男性超过那个年龄组的未婚女性,比例大约十比一。这些男性中,大多数确实希望结婚。然而,明显地,除非他们中有许多人娶非瑞兰女性,大部分人将保持未婚。   上面的论述基于下列哪一个假设?   A.从瑞兰移民出境,在女性中比男性更普遍。   B.与瑞兰男性相同年龄的高比例的三十几岁的瑞兰女性宁愿保持未婚。   C.瑞兰未婚男人中的许多人不可能娶比他们年龄大的女性。   D.瑞兰有高比例的离婚率。   E.大多数未婚的瑞兰男性不愿意娶非瑞兰的女性。   [解题分析]正确答案:C   本题是假设题。选项C是必须假设的,否则,许多瑞兰未婚男人都愿娶比他们年龄大的女性,就不会必然得出"30岁以下的未婚男性除非其中的许多人娶非瑞兰女性,大部分人将保持未婚"的结论。   -今年滨海市开展了一次前所未有的化妆品广告大战。但调查表明,只有25%的滨海市居民实际使用化妆品。这说明化妆品公司的广告投入有很大的盲目性。   以下哪项陈述最有力地加强了上述结论?   A.化妆品公司做的广告一般都比较多。   B.化妆品公司做广告是因为产品供过于求。   C.去年实际使用化妆品的滨海市居民有30%。   D.大多数不使用化妆品的居民不关心其广告宣传。   E.正是因为有25%的居民使用化妆品,才要针对他们做广告。   [解题分析]正确答案C。   本题是支持题。选项C最能加强,去年有30%用化妆品,今年只有25%的滨海市居民实际使用化妆品,说明广告无作用。   选项D力度不够,大多数不使用化妆品的居民不关心广告宣传,那说明还有一些不使用化妆品的居民关心广告,说明广告还是必要的。   -某企业在林浩、张亮两位候选人中民主选举正厂长。在选举的前十天进行的民意测验显示,受调查者中36%打算选林浩,42%打算选张亮。而在最后的正式选举中,林浩的得票率是52%,他的对手的得票率仅46%。这说明,选举前的民意测验在操作上出现了失误。   以下哪项,如果是真的,最能减弱上述论证的结论?   A.选举前20天进行的民意测验显示,林浩的得票率是32%,张亮的得票率是40%。   B.在进行民意测验的时候,许多选举者还没拿定主意选谁。   C.在选举的前七天,林浩为厂里要回了30万元借款,张亮为厂里争得40万元贷款。   D.民意测验同时涉及选民对选举组织者的意见。   E.林浩在竞选中的演说能力要比张亮强。   [解题分析]正确答案:A   本题是削弱题。由答案A,选举前20天进行的民意测验显示,林浩的得票率是32%,张亮的得票率是40%。而题干又告诉我们,在选举前十天民意测验,林浩的得票率是36%,张亮的得票率是42%。这说明越接近正式选举,"打算选林浩的选民的增长速度"超过"打算选张亮的选民的增长速度"。因此,这样就能说明最后林浩的得票率大于张亮的得票率并不奇怪。所以,选项A能减弱"选举前的民意测验在操作上出现了失误"这个结论。   选项B"在进行民意测验的时候,许多选举者还没拿定主意选谁"是个事实,并不能有力地削弱结论。   -随着人们年龄的增长,他们每天需要的卡路里下降,而他们每天需要的维生素B6增加。明显地,除非老年人摄入维生素B6补充物或者吃比他们年轻时多的含维生素B6的饮食,他们只有很小的可能性获得需要量的维生素B6。   下列哪一个对评价上述的论述最有用?   A.每天需要的卡路里的相对减少是否比每天需要增加的维生素B6的量要大。   B.饮食补充品中的维生素B6的形式是否比饮食中维生素B6的形式更容易被身体吸收和利用。   C.未获得足够的每天需要量的维生素B6的后果对于老年人比青年人更严重。   D.大多数人的饮食,当他们是年轻人时,是否包括远超过他们每天需要的维生素B6的量。 E.老年人的饮食是否比年轻人更可能在一天不止一顿的饮食中吃缺乏维生素B6的食品。   [解题分析]正确答案:D   本题是评价题。选项D可以起评价作用,即如果"大多数人在年轻时的饮食,包括了远超过他们每天需要的维生素B6的量",那么,"老年人即使不摄入更多的含维生素B6的饮食,也可能够需要";反之,如果"大多数人在年轻时的饮食,没有包括远超过他们每天需要的维生素B6的量",那么,"老年人只有摄入更多的含维生素B6的饮食,才可能够需要"。   (二)自上而下   归纳与假设、支持、反对、评价题型的解题思路正好相反,其解题思路是"自上而下",即假定我们所面临的段落的推理成立,让我们从段落推理中推出某些结果。   -近年来,购买新汽车而不是二手车的购买者的比例已经下降了,一些消费者把这个变化归因于新汽车价格的增加。作为价格增加的证据,他们引用数字表明,即使经过通货膨胀调整后,新汽车购买者所付的价格平均远高于几年前的价格,然而这个证据是没有说服力的,因为   下列哪一个最合逻辑地完成上面论述?   A.购买的新车的价值下降得比买二手车的价值快得多。   B.在一个人买了一辆车后,许多年后那个人才买另一辆车。   C.购买新车的购车者比例的下降必然意味着买二手车的比例已经增加了。   D.二手车销售的相对增加可能由所有汽车购买者中的仅仅小比例的购车者的决定来解释。   E.购买新车平均价格的变化可能仅仅是由于更多的人不喜欢不昂贵的新车而倾向于二手车。   [解题分析]正确答案:E   本题是归纳题。题干告诉这样的事实:购买新车的比例下降,但并不是由于新车价格增加,虽然新车购买者所付出的平均价格比以前高了。选项E正好能说明这种情况,即更多不喜欢便宜新车的人转而购买了二手车,这样就导致了目前新车购买者所付出的平均价格比以前高了。选项D不符合题干的意思,是不是小比例,题干中并没明确说明。   -统计数据正确地揭示:整个20世纪,全球范围内火山爆发的次数逐年缓慢上升,只有在两次世界大战期间,火山爆发的次数明显下降。科学家同样正确地揭示:整个20世纪,全球火山的活动性处于一个几乎不变的水平上,这和19世纪的情况形成了鲜明的对比。   如果上述断定是真的,则以下哪项也一定是真的?   Ⅰ.如果本世纪不发生两次世界大战,全球范围内火山爆发的次数将无例外地呈逐年缓慢上升的趋势。   Ⅱ.火山自身的活动性,并不是造成火山爆发的唯一原因。   Ⅲ.19世纪全球火山爆发比20世纪要频繁。   A.只有Ⅰ。   B.只有Ⅱ。   C.只有Ⅲ。   D.只有Ⅰ和Ⅱ。   E.Ⅰ、Ⅱ和Ⅲ。   [解题分析]正确答案:B。   从题干看出,战争会影响火山爆发,可见火山自身的活动性,并不是造成火山爆发的唯一原因,因此Ⅱ一定为真。   两次世界大战对自然空间的影响,例如,武器能量的释放对地壳运动的影响,就是这种火山活动性以外影响火山爆发的因素,这使得在两次世界大战期间,火山爆发的次数明显下降。但显然不能由此说,如果本世纪不发生两次世界大战,火山自身的活动性,一定会成为造成火山爆发的唯一原因,因为,像两次世界大战这样的对自然空间的影响,完全可能由别的形式产生,例如,巨大规模并且次数频繁的核试验等等。因此,Ⅰ不一定是真的。   由题干,火山自身的活动性只是造成火山爆发原因之一,并非唯一原因,因此,19世纪全球火山的活动性变动比较大并不等于19世纪全球火山爆发比20世纪要频繁。所以,Ⅲ不一定是真的。   -在MBA的《财务管理》课期终考试后,班长想从老师那里打听成绩。班长说:"老师,这次考试不太难,我估计我们班同学们的成绩都在70分以上吧。老师说:"你的前半句话不错,后半句话不对。"   根据老师的意思,下列哪项必为事实。   A.多数同学的成绩在70分以上,有少数同学的成绩在60分以下。   B.有些同学的成绩在70分以上,有些同学的成绩在70分以下。   C.研究生的课程70分才算及格,肯定有的同学成绩不及格。   D.这次考试太难,多数同学的考试成绩不理想。   E.这次考试太容易,全班同学的考试成绩都在80分以上。   [解题分析]正确答案:C。   班长的"后半句话不对",意思是"有些同学的成绩在70分以下"。加上C中的界定"研究生的课程70分才算及格",可得到"肯定有的同学成绩不及格"的结论。选A不对,因为可能没有成绩在60分以下的同学;选B不妥,因为从题干应该从语义上得出"多数同学(而不仅仅是有些同学)的成绩在70分以上"的结论。选D不对,因为题干并未说明"多数"还是"少数"同学成绩不理想。若选E,明显是错的。 第三章 逻辑备考基本方法(二)   二、三大命题原则   (一)"一视同仁"原则   逻辑命题的一个首要原则是是对考生一视同仁,也就是真正体现公平性,试题作为对知识水平的测试,应尽可能避免许多客观因素的影响,任何人接受测试都应享有充分的被公 平对待的权利,并且保证无专业背景之间的差异。即MBA逻辑考试对任何一个考生都是公平的。具体地说,虽然逻辑考题都是基于一个段落,所涉及的内容像阅读理解的文章一样,内容包罗万象,但读懂文章、回答问题,无需任何学科、专业的特定知识,你回答问题所需的一切信息均已包容在段落之中,考生只需着重从逻辑推理的角度来思维。尽管有些考题涉及逻辑学中的专用术语,但并不需要正式逻辑学的知识来解答。下面举一例:   -去年全国通货膨胀率为17%,而今年到目前为止平均为11%,由此我们可以得出这样的结论:全国通货膨胀率正呈下降趋势,明年的通货膨胀率将会更低。   以下哪项如果为真,将严重削弱上述结论?   A.去年通货膨胀率大幅度上升的主要原因是因为全国遭受了历史罕见的严重自然灾害。   B.在发展中国家,通货膨胀率比较高是正常现象。   C.消费者对于高通货膨胀率越来越适应了。   D.政府开始把抑制通货膨胀看成是宏观控制的主要目标之一。   E.由于抑制通货膨胀,现在失业人数和居民平均收入都有所下降。   [解题分析]正确答案:A   本题的内容涉及通货膨胀,但作为考生并不需要有经济学的知识来解题,也就是说,是否学过经济学的考生在解本题时都是公平的,因为做逻辑题只需要用逻辑推理能力既可。   本题要削弱的结论是"明年的通胀率会更低"。A项最能削弱,因为A说明了去年通胀率高是特殊情况,正常情况可能比今年要低,这说明今年的通胀率反而可能有上涨的趋势,说明明年通胀率就有可能比今年还要高。选项E是无关项,无法削弱。   (二)"假设正确"原则   "假设正确"原则包含两层意思:   一是对假设、支持、削弱和评价这四类推理思路自下而上的考题,在问题部分典型的问法是"下面哪个,如果正确(可行),……"。这使得选项在即使违反常识或专业知识的情况下,我们也不能质疑其正确性。应牢记的是:在五个选项内容本身无可质疑的情况下,通过考察推理能否成立,有无缺陷,去实现问题目的。段落或选项中所表述的信息、观点或事实是否正确并不是我们所关注的问题。也就是说,段落中所呈现的观点或事实是否正确并不重要,重要的是体会推理方法是否成立,有何缺陷,以及如何通过问题的目的去评价它。   二是对归纳类推理思路自上而下的考题,我们要认为题干段落所描述的内容都是正确的,即题干内容在即使违反常识或专业知识的情况下,我们也不能质疑其正确性。我们所要做的就是从题干出发,进一步往下推,能推出什么结论。   -为了挽救濒临灭绝的大熊猫,一种有效的方法是把它们都捕获到动物园进行人工饲养和繁殖。   以下哪项如果为真,最能对上述结论提出质疑?   A.在北京动物园出生的小熊猫京京,在出生24小时后,意外地被它的母亲咬断颈动脉而不幸夭折。   B.近五年在全世界各动物园中出生的熊猫总数是9只,而在野生自然环境环境中出生的熊猫的数字,不可能准确地获得。   C.只有在熊猫生活的自然环境中,才有它们足够吃的嫩竹,而嫩竹几乎是熊猫的唯一食物。   D.动物学家警告,对野生动物的人工饲养将会改变它们的某些遗传特性。   E.提出上述观点的是一个动物园主,他的动议带有明显的商业动机。   [解题分析]正确答案:C。   如果C项的断定为真,则动物园不可能为所有的大熊猫提供足够的嫩竹,因此,如果把大熊猫都捕获到动物园进行人工饲养和繁殖,它们唯一的食物来源就会发生问题。这就对题干的结论提出了严重的质疑。D和E项也能对题干构成质疑,但力度显然不如C项。A和B项不能构成质疑。   至于选项C本身这个说法是否符合实际情况并不是我们所关注的问题,只要它对题干推理起的作用符合问题的要求,就是正确答案。   -所有的猕猴桃都是香蕉,猴子是猕猴桃,所以,猴子是香蕉。   如果题干为真,下面哪个选项一定是真的?   A.香蕉长在白云上,猴子吃香蕉,所以,猴子跟着白云飘。   B.猴子是猕猴桃,猴子是香蕉,所以,所有的猕猴桃都是香蕉。   C.香蕉是猴子,猴子跑在白云上,所以,香蕉追着白云跑。   D.老虎追猴子,猴子追香蕉,所以,老虎追猴子和香蕉。   E.猴子不是香蕉,但猴子确实是猕猴桃,所以,"所有的猕猴桃都是香蕉,并且天上白云飘"这句话是假的。   [解题分析]正确答案:E。   本题的题干和选项的内容都及其荒谬,但并不妨碍我们做正确的逻辑推理,我们不需质疑题干的内容正确与否,而是要假设题干是正确的,从中进一步能推出什么结果。   题干的推理是"A1且A2→B",其等价命题(逆否命题)是"非B→非A1或非A2"。因此由"猴子不是香蕉,但猴子确实是猕猴桃"可推出"所有的猕猴桃都是香蕉"是假的,从而可知"所有的猕猴桃都是香蕉,并且天上白云飘"这句话是假的。   (三)"五中选二"原则   逻辑考题编写的一个基本原则是五个选项中的一个选项必须比其他的四个好。然而,正确选项的设计不能比其他四个选项明显地好以致问题的难度变得太小。从某种意义上讲,命题者构造高质量的迷惑性选项比构造正确选项的难度要大得多,对命题者来说,编制逻辑考题的最大痛苦与挑战不在于如何编制出一个正确答案,而在于如何编制出一个非常像正确答案但又绝对不是正确答案的选项。因此,通常五个选项中只有两个选项值得考虑(两者中仅仅有一个是对的)。另外三个选项,尽管有时与推理主题有点关联,但它们都忽略了论点的推理结构。   这个原则在"最能支持型"和"最能削弱型"考题中体现最充分,比如在"最能削弱型"考题中,至少有两个选项能起到削弱作用,这时需要衡量其削弱的力度才能确定正确答案。   -最近一次战争里在重战区中执行任务的医疗人员,即使是那些身体未受伤害的,现在比在该战争不太激烈的战斗中执行任务的医疗人员收入低而离婚率高,在衡量整体幸福程度的心理状况测验中得分也较低。这一证据表明即使是那些激烈的战争环境下没有受到身体创伤的人,也会受到负面影响。   下面哪个,如果正确,最强有力地支持了以上得出的结论?   (A)重战区的医疗人员和其他战区的医疗人员相比,服役前所接受的学校教育明显比较少。   (B)重战区的医疗人员比其他战区的医疗人员刚入伍时年轻。   (C)重战区医疗人员的父母和其他战区医疗人员的父母,在收入、离婚率和整体幸福程度方面没有什么显著差别。   (D)那些在重战区服务的医疗人员和建筑工人在收入、离婚率和整体幸福程度等方面非常相似。 (E)早期战争中的重战区服务的医疗人员在收入、离婚率和整体幸福程度等方面,和其他在该战争中服役的医疗人员没有表现出太大差别。   [解题分析]正确答案:C。   本题的推理是由一个事实而得出一个解释性的结论,其隐含的假设是除了激烈的战争环境之外没有别的因素影响推论。选项C指出这两类人的父母没有显著差异,实际上指出没有遗传因素影响结论"即使是那些激烈的战争环境下没有受到身体创伤的人,也会受到负面影响"   E项也有支持作用,易误选,因为其讲的是"早期战争",不是题干论据中讲的"最近一次战争",因此,其支持力度不如C项。   -许多消费者在超级市场挑选食品时,往往喜欢挑选那些用透明材料包装的食品,其理由是透明包装可以直接看到包装内的食品,这样心里有一种安全感。   以下哪项如果为真,最能对上述心理感觉构成质疑?   A.光线对食品营养所造成的破坏,引起了科学家和营养专家的高度重视。   B.食品的包装与食品内部的卫生程度并没有直接的关系。   C.美国宾州州立大学的研究结果表明:牛奶暴露于光线之下,无论是何种光线,都会引起风味上的变化。   D.有些透明材料包装的食品,有时候让人看了会倒胃口,特别是不新鲜的蔬菜和水果。   E.世界上许多国家在食品包装上大量采用阻光包装。   [解题分析]正确答案:A。   如果A项为真,说明透明包装食品的营养容易受到破坏,这就对题干中顾客的心理感觉构成了质疑。   C项也能构成质疑,但它涉及的只是牛奶这一种食品,其质疑力度显然不如A项。B项断定食品的包装与食品内部的卫生程度并没能真接的关系,但完全可能有间接关系,因此不构成质疑。其余项显然不能构成质疑。   -我国共有5万多公里的铁路,承担着53%的客运量和70%的货运量。铁路运力紧张的矛盾十分突出。改造既有铁路线路,提高列车的运行速度,就成了现实的选择。   如果下列哪项为真,则上述的论证就要大大削弱?   A.国家已经计划并且正逐步兴建大量的新铁路。   B.我国铁路线路及车辆的维修和更新刻不容缓。   C.随着经济的发展,铁路货运量还将增加。   D.随着航空事业和高速公路的发展,铁路客运量会下降。   E.正在试行时速达140-160公里的快速列车,比一般列车快50%。   [解题分析]正确答案:A。   本题的逻辑主线是"铁路运力紧张→必须改造提速",要削弱论证就是要说明存在别的因素影响推论。A中讲的是正在逐步"兴建大量的新铁路",强烈暗示铁路运力紧张的矛盾会逐步缓解,因此,题干中关于"改造既有铁路"和"提高列车的运行速度"就不显得那么必要,即削弱了题干的论证。   D项也有一定的削弱作用,但其力度不如A项,因为D只说了铁路客运量会下降,没说到货运量的问题,因此,铁路运力可能依然紧张。   -江口市急救中心向市政府申请购置一辆新的救护车,以进一步增强该中心的急救能力。市政府否决了这项申请,理由是:急救中心所需的救护车数量,必须和中心的规模和综合能力相配套。根据该急救中心现有的医护人员和医疗设施的规模和综合能力,现有的救护车足够了。   以下哪项是市政府关于此项决定的论证所必须假设的?   A.江口市的急救对象的数量不会有大的增长。   B.市政府的财政面临困难无力购置新的救护车。   C.急救中心现有的救护车中,至少有一辆近期内不会退役。   D.江口市的其它大中医院有足够的能力配合急救中心抢救全市的危重病人。   E.市政府至少在五年内不会拨款以扩大急救中心的规模和综合能力。   [解题分析]正确答案:C。   C项是必须假设的。否则,如果急救中心现有的救护车近期内全部退役,那么,市政府否决上述申请的理由就不能成立。   E项讲的是五年内,不是讲的是近期的事,因此不如C好。   第三章 逻辑备考基本方法(三) http://www.sina.com.cn 2003/10/15 13:30 新浪教育   三、三大解题原则   (一)"答案不需充分性"原则   这个原则是针对以支持和削弱型考题为主要代表的解题思路为"自下而上"的题目而言的。   自然界和社会中的各个现象都是与其他现象互相联系、互相制约的。如果某一个现象的存在必然引起另一个现象的发生,那么这两个现象之间就具有因果联系。其中,引起某一现象产生的现象叫做原因,被另一现象引起的现象叫做结果。因果现象是相对的,一个现象对于某现象来说是结果,而对于另一现象来说可能又是原因。例如,爆炸既是火药达到一定温度的结果,又是造成人员伤亡的原因。因果联系的一个重要特点是在时间上具有先后相继的顺序,即对一对因果事件来说,总是原因在前,结果在后,概莫例外。因果联系的另一个重要特点是确定的原因总是产生确定的结果,而确定的结果总是由确定的原因产生的,两者的关系是确定的。既无无因之果,也无无果之因。但任何确定事物的发生都依赖于许多条件或许多原因,现实生活中,我们很难找到某一确定发生的所有条件或原因,因此当问题目的让我们寻求一个支持或反对段落推理的答案时,我们的目标是去寻求一个(仅仅是许多条件或许多原因中的一个)使结论成立的可能性增大或减少的答案,而绝非一定要去寻求一个使结论必然成立或必然错误的答案。切记:逻辑考题是要我们寻找一个与段落推理有关并对段落推理起到某种作用的选项,而并不是一个真理。   最典型的一道例题是曾有一道国外考题:题干是"某人正好看到天空中飞过两只渡鸦,因此,他看到的下一只飞鸟也是渡鸦",问题是"下列哪一个选项能支持上面论述",正确答案是"渡鸦成群飞"。有的考生认为这个答案不正确,因为他看到的两只渡鸦可能是成群飞的渡鸦中最后的两只。这种思维错误主要在对答案并不需要充分性没有清楚的认识。就这道题而言,渡鸦成群飞这个条件确定使得上面论述的结论"他看到的下一只鸟也是渡鸦"的可能性增大,因此它是正确答案。但即使这个条件找到了,我们 也并不保证结论就一定正确,我们对于支持题型,仅仅是找一个使结论成立可能性增大的选项而已。   请认真体会下面这个例子:   -某州州长任命了一名黄种人担任州旅游局长,许多白种人和黑种人指责这一任命是一种显示各族平等的政治姿态;后来州长又任命了一名黑人担任州警事总监,许多白种人和黄种人对这一任命又作出了同样的指责。确实,州长作出上述任命的时候很大程度上是出于政治上的考虑,但这又有什么错呢?况且,上述任命完全在州宪章赋予州长的权力范围之内。   以下哪项,如果为真,最能加强上述论证?   A.各族平等是业已受到宪法和公众确认的普遍原则。   B.被任命的旅游局长和警事总监完全能够胜任他们的职位。   C.本州州长政绩显赫,声誉颇佳,过去很少受到他人的指责。   D.在作出了上述任命之后,州长紧接着又任命了一名白种人担任财政总监。   E.评价一项任命的根据不仅是看这一任命是否符合法律程序,而且更要看其是否有利于公众的需要。   [解题分析]正确答案:B   选项B并不是题干段落推理成立的充分条件(即有B项存在,题干结论就一定能成立),但B项确实能加强题干论证,因此就是正确答案。   实质上,本题B项是推理成立必要条件,如果B项不成立,即被任命的旅游局长和警事总监不能够胜任他们的职位,那么可以说明州长的任命完全是一种显示各族平等的政治姿态;题干的意思是虽然在同等符合条件的候选人中从政治上有所考虑并没有错,但如果仅从政治上考虑而不顾候选人根本不能胜任,那就有错了。   (二)"收敛思维"原则   这是与"假设正确"命题原则相关的解题原则。由于逻辑题只是选项本身字面所体现的意思,并且在段落中只能就给出的信息进行推理,因此千万不能发散思维、联想。许多考生总是对考题进行递进推理,看到某一选项就得出一个并不必然的推理,总喜欢对选项进行诸如"如果,那么"的递进,但是MBA逻辑题的思维推理大多只在一个层面上进行,并且只能在段落所给出的信息的基础上进行推理。逻辑考题的正确答案必须是其本身含义加上段落之中的信息来实现问题目的,决不能对选项的意思进行递进推理。换而言之,思维只能在段落之内,不能有段落之外信息的进一步推导。   -"万物生长靠太阳",这是多少年来人们从实际生活中总结出来的一个公认的事实,然而,近年来科学家研究发现:月球对地球的影响远远大于太阳;孕育地球生命的力量,来自月球而非太阳。   以下哪项不能作为上述论断的证据?   A.在日照下,植物生长快且长得好,日照特别是对几厘米高、发芽不久的植物如向日葵、玉米等最有利。   B.当花枝因损伤出现严重伤口时,日光能清除伤口中那些不能再生长的纤维组织,加快新陈代谢,使伤口愈合。   C.植物只有靠了太阳光才能进行光合作用,动物也只有在阳光下才能茁壮成长。   D.月球在地球形成之初,影响地球产生了一个巨大磁场,屏蔽来自太空的宇宙射线对地球的侵袭。   E.科学家在太平洋加拉帕期群岛附近的深海海底,发现并采集了红色的蠕虫、张着壳的蛤、白色的蟹等,这可能与日照有关。   [解题分析]正确答案:C   不能带有任何背景知识来推理,题目中说什么就是什么,这就是"收敛思维"原则。   首先要注意到本题问的是"以下哪项不能作为上述论断的证据"。题干的论断是"月球对地球的影响远远大于太阳",不能作为其论据的选项就是要说明"太阳对地球的影响具有决定意义(即太阳对地球的影响大于月球)",而选项C正是表达了这个意思。   -如果二氧化碳大量地产生,它将聚集在大气中引起气候的温室效应,包括树在内的植物的腐烂,会产生出二氧化碳。不过,在森林中,由于活植物会吸入二氧化碳,释放氧气,从而会抵消掉这些二氧化碳。工业生产用的来自于植物的燃料在使用中也会产生大量二氧化碳,这些燃料包括木材、煤和石油等。   如果上述情况属实,则从中可以推导出以下哪项结论?   A.工业生产中产生的所有二氧化碳都直接或间接来源于植物。   B.由于植物在腐烂过程中要放出二氧化碳,温室效应不可避免。   C.如果不用某种途径吸收工业生产中因使用来自植物的燃料而排放到大气中的二氧化碳,则二氧化碳的净含量会增加。   D.森林向大气释放的二氧化碳量,与工业生产中使用的来自于植物的燃料所释放的二氧化碳量相当。   E.不论哪种工业用燃料都必会引起大气中二氧化碳总量的增加和温室效应。   [解题分析]正确答案:C   题干中所说"植物的腐烂产生的二氧化碳由活植物会吸入而抵消掉",因此可以推出"如果不用某种途径吸收工业生产中因使用来自植物的燃料而排放到大气中的二氧化碳,则二氧化碳的净含量会增加。"A不能推出,因为题干中只是说"工业生产用的来自于植物的燃料在使用中也会产生大量二氧化碳",并没有说明"工业中产生的所有二氧化碳都来源于植物"   (三)"相对最好"原则   这是与"五中选二"命题原则相关的解题原则。逻辑考题要求你根据段落里所给信息及逻辑推理,从下面五个选项中选出一个选项,从而实现问题目的。很多情况下,这个选项并不是专业知识背景下或常识中的最佳选项,但只要是五个选项中最能实现问题目的的选项就可以了,也就是要让考生从五个选项中挑选一个相对最好的选项。   请认真体会以下例子:   -一些恐龙的头盖骨和骨盆骨与所有现代鸟类的头盖骨和骨盆骨有许多相同特征。虽然不是所有的恐龙都有这些特征,但一些科学家声称,所有具有这些特征的动物都是恐龙。   如果上面的陈述和科学家的声明都是正确的,下列哪一项也一定正确?   (A)鸟类与恐龙的相似之处要多于鸟类与其它动物的相似之处。   (B)一些古代恐龙与现代鸟类是没有区别的。   (C)所有动物,如果它们的头盖骨和现代鸟类的头盖骨具有相同特征,那么它们的骨盆骨也一定和现代鸟类的骨盆骨具有相同特征。   (D)现代鸟类是恐龙。   (E)所有的恐龙都是鸟类。   [解题分析]正确答案:D   由题干知,一是,所有现代鸟类的头盖骨和骨盆骨与一些恐龙的头盖骨和骨盆骨有许多相同特征;二是,科学家声称,所有具有这些特征的动物都是恐龙。因此,可以推出现代鸟类是恐龙,虽然这不符合常识,但这不是我们做逻辑推理时所关心的。 第三章 逻辑备考基本方法(四) 四、强化阅读训练法   逻辑考试可以说是阅读理解和逻辑推理的杂交品,而且阅读理解是逻辑推理的基础,只有读明白了才能进行有效的推理。阅读是一切逻辑题成功解题的基础,也是考试中决定心理状态的关键因素。可以说逻辑考试是否成功至少一半以上的因素要归于阅读,提高阅读水平是解答逻辑题的首要条件,因此,从平时起加强对阅读能力的培养显得尤为重要。   (一)“快速阅读”训练。   MBA逻辑考试不同于一般逻辑考试的特点,首先是整个试卷的阅读量很大,2003年联考估计字数至少在6000~8000字以上,要求在45分钟内完成。由于25道考题的每道题均有一定长度,分配给每道题的时间平均大约也仅是1.8分钟,很多考生如果不经过训练,读完读明白一道题也得化1分多钟,因此,真正用来思考并作出选择的时间就很少了。可以说逻辑很大一部分是在考察考生快速阅读文字材料,准确把握文中观点和推理结构的能力。因此,考生一定要从平时开始有意识地训练自己快速阅读的能力。   快速阅读追求的是“全景视觉”,一次注视的是一个字群或者一个意群,甚至整段文字,速读法不仅是对书本的表面浏览,而且是一种积极、活跃、创造性的理解和记忆过程,它是一种真正的阅读艺术。其核心是在快速浏览的同时大脑积极思维,迅速提炼,从而抓取关键信息。在训练中,特别要注意在仅读一遍段落的情况下,迅速提炼出究竟哪句话为结论,然后看出和结论相关的直接原因是什么,也即是迅速找出段落的逻辑主线。-吴大成教授:各国的国情和传统不同,但是对于谋杀和其他严重刑事犯罪实施死刑,至少是大多数人可以接受的。公开宣判和执行死刑可以有效地阻止恶性刑事案件的发生,它所带来的正面影响比可能存在的负面影响肯定要大得多,这是社会自我保护的一种必要机制。   史密斯教授:我不能接受您的见解。因为在我看来,对于十恶不赦的罪犯来说,终身监禁是比死刑更严厉的惩罚,而一般的民众往往以为只有死刑才是最严厉的。   以下哪项是对上述对话的最恰当评价?   A.两个对各国的国情和传统有不同的理解。   B.两人对什么是最严厉的刑事惩罚有不同的理解。   C.两人对执行死刑的目的有不同的理解。   D.两人对产生恶性刑事案件的原因有不同的理解。   E.两人对是否大多数人都接受死刑有不同的理解。   [解题分析]正确答案:C。   这类二人对话的题目,一般是阅读量较大,因此,解题的关键在于要在快速阅读两位教授对话的过程中,迅速把握其说话的主旨。   由题干,不难得出结论:吴大成教授认为执行死刑的目的是有效地阻止恶性刑事案件的发生,而史密斯认为执行死刑的目的是给十恶不赦的罪犯以最严厉的惩罚。两人对执行死刑的目的有不同的理解。因此,C项的评价最为恰当。   (二)“抗烦阅读”训练。   天文、地理、生物、金融、消费者行为、计算机。150~300字一个段落。阅读自己平常较烦的内容的文章,体会阅读速度与效益的结合。   -鸡油菌这种野生蘑菇生长在宿主树下,如在道氏杉树的底部生长。道氏杉树为它提供生长所需的糖分。鸡油菌在地下用来汲取糖分的纤维部分为它的宿主提供养料和水。由于它们之间这种互利关系,过量采摘道氏杉树根部的鸡油菌会对道氏杉树的生长不利。   以下哪项如果为真,将对题干的论述构成质疑?   A.在最近的几年中,野生蘑菇的产量有所上升。   B.鸡油菌不只在道氏杉树底部生长,也在其他树木的底部生长。   C.很多在森林中生长的野生蘑菇在其他地方无法生长。   D.对某些野生蘑菇的采摘会促进其他有利于道氏杉树的蘑菇的生长。   E.如果没有鸡油菌的滋养,道氏杉树的种子不能成活。   [解题分析]正确答案:D。   在本题的阅读中,“鸡油菌”和“道氏杉树”等名词都是考生所不熟悉的,因此要从平时就要培养自己的抗烦阅读能力。   阅读题干,首先要概括成自己的一句话,即“菌有利于树,因此,过量采摘菌对树不利”。如果D项的断定为真,则说明虽然过量采摘鸡油菌会直接割断和道氏杉树的互利关系,但有可能促进其他有利于道氏杉树的蘑菇的生长,而最终仍间接地对道氏杉树有利。这就构成了对题干的质疑。其余各项均不能构成质疑。   (三)“空中摄影式”阅读训练   逻辑读题的最好方式是要用边阅读边快速地提取题干的推理脉络,简洁把握什么是前提,什么是结论。反之,读题的误区是欣赏与想象,切忌沉浸在题目之中,精力分散在各句话语之间而不能迅速判断其相互关系。   考生必须根据命题人员所设定的问题目的去读段落,按照不同的问题目的锁定不同的读题重点,且思维应在题目之内。你的眼睛就如摄影机的镜头摄入设定的风景,然后整个思维的过程犹如洗相片的过程,尽可能拷贝原来的风景且不至于失真。   -在目前财政拮据的情况下,在本市增加警力的动议不可取。在计算增加警力所需的经费开支时,光考虑到支付新增警员的工资是不够的,同时还要考虑到支付法庭和监狱新雇员的工资,由于警力的增加带来的逮捕、宣判和监管任务的增加,势必需要相关机构同时增员。   以下哪项如果为真,将最有力地削弱上述论证?   A.增加警力所需的费用,将由中央和地方财政共同负担。   B.目前的财政状况,决不至于拮据到连维护社会治安的费用都难以支付的地步。   C.湖州市与本市毗邻,去年警力增加19%,逮捕个案增加40%,判决个案增加13%。   D.并非所有侦察都导致逮捕,并非所有逮捕都导致宣判,并非所有宣判都导致监禁。   E.当警力增加到与市民的数量达到一个恰当的比例时,将会减少犯罪。   四、强化阅读训练法   [解题分析]正确答案:E。   读题干时,视线要快速掠取段落的主干脉络,本题的主干脉络就是“警力增加会导致相关机构增员”,要削弱论证就是要说明“警力增加”与“相关机构增员”并没有必然的联系,这两着之间是有差异的。   如果E项的断定为真,则说明警力的增加不一定会造成逮捕、宣判和监管任务的增加,相反,有可能因为犯罪的减少而降低这方面的开支。这就有力地削弱了题干的论证。其余各项均不能削弱题干的论证。其中,A和B项能削弱题干的结论,但不能削弱题干的论证;C项实际上加强了题干的论证。总之,通过有效的阅读训练,做逻辑题时,要有一种“会当凌绝顶,一览众山小”的宏观把握能力,这是成为一名逻辑高手的必要条件。阅读训练关键是快速抽取逻辑结构性的能力,具体来说,就是要迅速抓取一道逻辑题的核心,即它得出结论的方式或结论本身,抓住了这个核心也就掌握了解题的要领。 特别说明:由于各方面情况的不断调整与变化,新浪网所提供的所有考试信息仅供参考,敬请考生以权威部门公布的正式信息为准。 第三章 逻辑备考基本方法(五) http://www.sina.com.cn 2003/10/22 14:38 新浪教育   五、系统综合训练法   在阅读训练的基础上,建议考生有计划、有针对性地做些其他多方面的训练,将有助于提高自己的实战能力。具体训练方法如下:   (一)“设身处地”训练。   出5~10道不同类型的题目给自己做,考什么?怎么问?陷阱在何处?怎样构造迷惑性选项?   在平时的训练中,比如对一些考过的题,考生也可以换种问法来出题,以训练自己的思路,下面举一例:   -由于近期的干旱和高温,导致海湾盐度增加,引起了许多鱼的死亡。虾虽然可以适应高盐度,但盐度高也给养虾场带来了不幸。   以下哪个选项如果为真,能够提供解释以上现象的原因?   A.一些鱼会到低盐度的海域去,来逃脱死亡的厄运。   B.持续的干旱会使海湾的水位下降,这已经引起了有关机构的注意。   C.幼虾吃的有机物在盐度高的环境下几乎难以存活。   D.水温升高会使虾更快速地繁殖。   E.鱼多的海湾往往虾也多,虾少的海湾鱼也不多。   [解题分析]正确答案:C。   如果C项为真,说明虽然虾能适应高盐度,但是由于幼虾吃的有机物在高盐度下难以存活,因此,海湾盐度增高,同样威胁到虾的生存特别是繁衍,给养虾场带来不幸。其余各项均不能解释题干。   本题也可以改造成削弱题型:“虽然近期的干旱和高温导致了海湾盐度增加,引起了许多鱼的死亡,但虾可以适应高盐度,因此,盐度高不会给养虾场带来不幸。以下哪个选项如果为真,最能削弱上述结论?”在选项不变的情况下,答案不变。   (二)“题河战术”训练。   只有通过平时的强化训练,才能真正迅速、有效地培养考试现场的感觉,从而在正式应试时迅速找到解题方向。   复习迎考只是一个挖掘潜能、规范操作的过程,它绝不是去系统地学逻辑学,而是搞“题河战术”(“题海战术”是指超大量做题,“题河战术”是指做适量的题),以题论题绝对是短时间内提高逻辑考试成绩的最好办法。而在题目训练的过程中能否举一反三、触类旁通是逻辑复习效果的关键。   看本书的例题,最好也先做一遍,再看它的分析,平均一分半钟做一道。做题时心态要平和,不要钻牛角尖。感觉一下一分半钟的长度,实际还是很长的。个别题稍难,可能化的时间要长些,也不要着急Q盗返墓瘫旧砭褪俏舜锏绞炷苌桑饶闳险嫜卸镣瓯臼椋馓獾乃俣纫沧匀换崽嵘侠础<且淞Χ云绞钡难盗菲鸷艽笞饔茫嗨频奶饽孔龉槐橹螅Φ绷粝滤悸返暮奂#健白鲆坏朗且坏馈薄?/p>   (三)“解剖实验”训练。   平时训练时,对自己做错的题或虽然做对但费时较多的题,在回头重做时,一定要写下这道题的分析过程,进行“解剖”实验,规范自己的思路及操作。   下面举一例:   -如果李生喜欢表演,则他报考戏剧学院;如果他不喜欢表演,则他可以成为戏剧理论家;如果他不报考戏剧学院,则不能成为戏剧理论家。   由此可推出李生将:   A.不喜欢表演。   B.成为戏剧理论家。   C.不报考戏剧学院。   D.报考戏剧学院。   E.不成为戏剧理论家。   [解题分析]正确答案:D。   本题是一道复合命题推理的题型,其解题方法是边读题边抽象出推理关系,并记在草稿纸上,通过递推,即可找到答案。由本题题干,可得出以下推理关系:   喜欢表演→报考戏剧学院(1)   不喜欢表演→能成为理论家(2)   不报考戏剧学院→不能成为理论家(3)   因此,(3)等价于它的逆否命题:能成为理论家→报考戏剧学院(4)由(2)和(4)得出,不喜欢表演→报考戏剧学院(5)所以,由(1)和(5),不管李生喜不喜欢表演,都将报考戏剧学院   (四)“模拟实战”训练   平常在做练习时一定要集中限时训练,否则成效较小。由于MBA逻辑题并不难,如果没有时间限制一般都能做出来,不这样集中训练就起不到应有的效果。有的考生习惯于平时慢慢做,做一题对一下答案,这非常不利于MBA逻辑的备考,所以,一定要45分钟(最好是30分钟)内做25题,做完后对答案,再对做错的题进行分析,并充分理解。同时应注意:不要进行疲劳战术,一次正式训练就只做25题;要专心,不要隔一会儿做一道,有手无心;有时暂时的错误率比较高,也不要太担心。每次练习都要记时计分。具体做法是:(1)总用时限制在45分钟,最好在30分钟之内做完;(2)题目做过一遍之后,记一下时间;(3)用5分钟检查一遍;(4)然后写下自己对“错了几个”的估计;(5)对照答案。   考生一定要坚信:虽然逻辑考试以前没碰到过,但是报考MBA的考生均受过高等教育,应该都比较聪明,底子好,只是需要热一热身,进入一下实战环境,把速度和成功率这两项指标调整到最佳状态,就一定能在逻辑考试中取得好成绩。   (五)“ 规范思路”训练   这主要是针对临考前的训练,即临考前两三周不要再做太多的模拟题,重点是看以前做过的题,特别是对两类题:一是曾经做错的题,二是虽然做对但自己理解得并不确切。在看的过程中只看正确答案,并仔细体会其正确性,这样可以规范自己的思路。 第三章 逻辑备考基本方法(六) http://www.sina.com.cn 2003/10/23 09:19 新浪教育   六、高效快速解题法   虽然MBA逻辑考试并不是考专门的逻辑知识,题目所直接用到的逻辑知识其实很有限,也很简单;但是也应该看到,MBA逻辑考题本身并不简单。其主要原因来源于两个方面:一是,MBA逻辑考试涉及的面很广,试题内容几乎包括自然科学、人文社会科学、思维科学和一般性常识在内的一切领域。二是,题目的量很大,要求考生能够快速阅读,并善于撇开不相 干信息,提取和把握关键信息,从而迅速找出答案。   可以说,只要不受时间限制,未学过任何逻辑学的多数考生初次做MBA逻辑题也都可以答对大部分考题,但这是远远不够的,由于考试时间有限,要取得逻辑高分,就一定既要答得对还要答得快。而要做到这点,一个基本方法就是:边读题边用箭头、符号、图表来简化推理关系,明确逻辑主线,从而迅速找到解题突破口。下面提供七种逻辑解题方法,供考生参考。   (一)划线法   考生要善于抓住关键性信息。由于逻辑考题的阅读量较大,时间有限,所以不可能慢慢地阅读,而要在快速阅读中抓住关键信息,那么,什么样的信息是关键性信息?一句话,与题干中问题相关的信息就是关键性的,否则就是冗余的起干扰作用的信息。加快阅读的考试技巧是阅读时圈阅题眼。所谓“题眼”,就是关键的字词、语句。边阅读边给题眼做明显的标记,将大大提高做题的速度和准确性。具体做发法是,边读题边在题目上划出重点或将待处理的元素、条件都在读题过程中记在草稿纸上。   -可供收养的孩子数量比起想要收养的孩子数量要少得多。目前有200万对夫妇在排队等待收养孩子,但在1982年(我们最后所能查到的统计资料)只有大约5万个孩子被收养。   以下哪项陈述如果为真,则最能支持上述推论?   A.想要收养孩子的夫妇的数目有相当程度的增长。   B.寻求收养孩子的人们往往要通过中介机构进行长期的访问和调查。   C.今年被领养孩子的数量比以往任何一年都要多。   D.在一年中被收养孩子的数量基本上与同期可供被收养的孩子数量相等。   E.那些寻求收养孩子的人一般来讲能够成为好父母。   [解题分析]正确答案:D   本题的关键信息是涉及三个数量:想收养的孩子的数量、可供收养的孩子的数量、被收养的孩子的数量。在边读题的过程中边在题上划出来,以利于推理。   所谓支持,就是把待选的选项放入题干中,使结论成立的可能性增大的选项就是正确选项。题干的结论是可供收养的孩子的数量远小于想收养的家庭的数量,论据是被收养的孩子的数量=5万,想收养的孩子的数量=200万。由D项,被收养的孩子的数量=可供收养的孩子的数量,这样加上题干论据,就可以使“可供收养的孩子数量比起想要收养的孩子数量要少得多”这个推论成立。   (二)排除法   通常,逻辑题的五个选项中只有一个难以排除,其他三个选项可能根本与问题目的无关,所以排除法是提高解题速度的关键。其实,逻辑出题人在编写选项时也往往非常困难,既要保证一个选项正确,又要使其他选项具有迷惑性,所以许多逻辑题的四个错误选项的出法往往是荒谬的,考生必须从大量训练中体会正确选项的模式,找出错误选项的特征。   在实际解题时,在排除法不复杂的情况下,尽可能使用排除法(排除法是让人感到最放心的方法),首先上来就先划掉绝对不可能的选项。   -关于确定商务谈判代表的人选,甲、乙、丙三位公司老总的意见分别是:   甲:如果不选派李经理,那么不选派王经理。   乙:如果不选派王经理,那么选派李经理。   丙:要么选派李经理,要么选派王经理。   以下诸项中,同时满足甲、乙、丙三人意见的方案是?   A.选李经理,不选王经理。   B.选王经理,不选李经理。   C.两人都选派。   D.两人都不选派。   E.不存在这样的方案。   [解题分析]正确答案:A。   用排除法解。选项A,没问题,都满足。选项B,与甲矛盾。选项C,与丙矛盾。选项D,与乙、丙都矛盾。   -某届“百花奖“评选结束了。甲电影制片厂拍摄的《黄河颂》获得最佳故事片奖,乙电影制片厂拍摄的《孙悟空》获得最佳美术奖,丙电影制片厂拍摄的《白娘子》获得最佳戏曲片奖。   授奖大会以后,甲厂的导演说:“真是有趣得很,我们三个人的姓分别是三部片名的第一个字,再说,我们每个人的姓同自己所拍片子片名的第一个字又不一样。“这时候,另一个姓孙的导演笑起来说:“真是这样的!”   基于以上题干,可推出这三部片子的导演各姓什么?   A甲厂导演姓孙,乙厂导演姓白,丙厂导演姓黄。   B甲厂导演姓白,乙厂导演姓黄,丙厂导演姓孙。   C甲厂导演姓孙,乙厂导演姓黄,丙厂导演姓白。   D甲厂导演姓白,乙厂导演姓孙,丙厂导演姓黄。   E甲厂导演姓黄,乙厂导演姓白,丙厂导演姓孙。   [解题分析]正确答案:B。 用排除法解。因为甲厂的导演说完后另一个姓孙的导演又说,说明甲厂导演不姓孙,排除A;丙厂摄的是《白娘子》,因此丙厂导演不姓白,排除C;同样可排除D、E;最后剩下B即为正确答案。 -上河市的报纸销售量多于天河市。因此,上河市的居民比天河市的居民更多地知道世界上发生的大事。   下列选项中除了哪个选项都能削弱上述论断:   A.上河市的居民比天河市多。   B.天河市的绝大多数居民在上河市工作并在那里买报纸。   C.上河市居民的人均看报时间比天河市居民的人均看报时间少。   D.一种上河市报纸报道的内容局限于上河市内的新闻。   E.上河市报亭的平均报纸售价低于天河市的平均报纸售价。   [解题分析]正确答案:E   本题问的是“除了”,因此,可用排除法排除掉能够削弱的选项。   A能削弱,因此不是正确答案,理由如下:上河市报纸销量虽多,但由于人口也多,可能人均报纸拥有量比天河市低,这样,上河市的居民反而不如天河市的居民更多地知道世界大事。同样,B、C、D项也都能削弱题干论断。因此,A、B、C、D项可排除掉。   选项E所说“上河市报亭的平均报纸售价低于天河市的平均报纸售价”能说明“上河市的报纸销售量多于天河市”,但不能削弱“上河市的居民比天河市的居民更多地知道世界上发生的大事”这个论断。   (三)列表法   对一些综合型和匹配型题,边读题,边把已知条件列在表上,并在表上进一步进行推理,可以较快地找到正确答案,有利于解题。   -李浩、王鸣和张翔是同班同学,住在同一宿舍。其中,一个是湖南人,一是个重庆人,一个是辽宁人。李浩和重庆人不同岁,张翔的年龄比辽宁人小,重庆人比王鸣年龄大。   根据题干所述,可以推出以下哪项结论?   A.李浩是湖南人,王鸣是重庆人,张翔是辽宁人。   B.李浩是重庆人,王鸣是湖南人,张翔是辽宁人。   C.李浩是重庆人,王鸣是辽宁人,张翔是湖南人。   D.李浩是辽宁人,王鸣是湖南人,张翔是重庆人。   E.李浩是辽宁人,王鸣是重庆人,张翔是湖南人。   [解题分析]正确答案:D。   这类题目只要细心不忙乱,应当是轻易拿分的。根据题干可推出:   李浩不是重庆人,王鸣不是重庆人。因此,张翔是重庆人。   再进一步根据题干列出下表,即得出答案   (李浩) >张翔> (王鸣)   辽宁重庆湖南   -根据上题题干所述,以下哪项是关于他们三人的年龄次序(由大到小)的正确表述?   A.李浩、王鸣、张翔。   B.李浩、张翔、王鸣。   C.王鸣、李浩、张翔。   D.张翔、李浩、王鸣。   E.张翔、王鸣、李浩。   [解题分析]正确答案:B。   由上题的列表分析已得出正确答案。   -曙光机械厂、华业机械厂、祥瑞机械厂都在新宁市辖区。它们既是同一工业局下属的兄弟厂,在市场上也是竞争对手。在市场需求的五种机械产品中,曙光机械厂擅长生产产品1、产品2和产品4,华业机械厂擅长生产产品2、产品3和产品5,祥瑞机械厂擅长生产产品3和产品5。如果两个厂生产同样的产品,一方面是规模不经济,另一方面是会产生恶性内部竞争。如果一个厂生产三种产品,在人力和设备上也有问题。为了发挥好地区经济合作的优势,工业局召集三个厂的领导对各自的生产产品作了协调,作出了满意的决策。   以下哪项最可能是这几个厂的产品选择方案?   A.曙光机械厂生产产品1和产品5,华业机械厂只生产产品2。   B.曙光机械厂生产产品1和产品2,华业机械厂生产产品3和产品5。   C.华业机械厂生产产品2和产品3,祥瑞机械厂只生产产品4。   D.华业机械厂生产产品2和产品5,祥瑞机械厂生产产品3和产品4。   E.祥瑞机械厂生产产品3和产品5,华业机械厂只生产产品2。   [解题分析]正确答案:E。   曙光擅长产品1、2、4;华业擅长2、3、5;祥瑞擅长3、5。由于1、4只有曙光擅长,故应安排曙光生产;又因为一个厂生产三种产品有问题,故产品2曙光不应生产,则只能有华业生产。这样用排除法可排除选项A、B、C、D,只有E符合。列表如下: -某医学院学生小赵、小钱、小孙和小李在附属医院实习的第一天,分别给四位病人作出如下诊断:   病人甲:小赵诊断为疟疾,小钱诊断为流感。   病人乙:小钱诊断为胃炎,小孙诊断为胃溃疡。   病人丙:小孙诊断为痢疾,小李诊断为肠炎。   病人丁:小李诊断为肺结核,小赵诊断为支气管炎。   他们诊断之后,主治医师作了复诊,说:“每位病人都有一种诊断是正确的。四位实习生中,有一位诊断全对,一位诊断全错,小孙不是全对的。”   这时候化验结果出来了,病人甲血液中发现了疟原虫,病人丙的大便中发现痢疾杆菌。   请问诊断全对的实习生是:   A、小赵B、小钱C、小孙D、小李   E、无法判断   [解题分析]正确答案:A。   列个表,题目即可迎刃而解。解题步骤如下:   ①首先有题干知,病人甲血液中发现了疟原虫,病人丙的大便中发现痢疾杆菌,   ②因为小孙诊断不全对,则乙没患胃溃疡   ③因为每位病人都有一种诊断是正确的,   既然甲患了疟疾,则甲没患流感;   既然乙没患胃溃疡,则乙患了胃炎;   既然丙患了痢疾,则丙没患肠炎;   ④因为四位实习生中,有一位诊断全对,一位诊断全错;   而小钱、小孙的诊断都不全对也不全错,小赵已有一种诊断正确,小李已有一种诊断错误;因此,只能是小赵诊断全对,小李诊断全错。   (四)画图法   对一些集合型题,有时需要划个示意图,有助于完成推理。   -所有爱斯基摩土著人都是穿黑衣服的;所有的北婆罗洲土著人都是穿白衣服的;没有穿白衣服又穿黑衣服的人;H是穿白衣服的。基于以上事实,下列哪个判断必为真?   A.H是北婆罗洲土著人。   B.H不是爱斯基摩土著人。   C.H不是北婆罗洲土著人。   D.H是爱斯基摩土著人。   E.H既不是爱斯基摩土著人,也不是北婆罗洲土著人。   [解题分析]正确答案:B。   因为“穿黑衣服”是“爱斯基摩土著人”的必要条件,H(穿白衣服)不具备此必要条件,所以H不是爱斯基摩土著人。选项A不妥,因为除了题干所讲的两种人以外,还可能有其他的人,H可能是其他种族的人中的穿白衣服的。C、D明显不符合要求。选E也不对,因为H穿白衣服,可能就是北婆罗洲土著人(虽然不能肯定“是”,但也不能断言“不是”)。   具体划一个下图比较形象直观,有利于解题。   -以下是某市体委对该市业余体育运动爱好者一项调查中的若干结论:所有的桥牌爱好者都爱好围棋;有围棋爱好者爱好武术;所有的武术爱好者都不爱好健身操;有桥牌爱好者同时爱好健身操。如果上述结论都是真实的,则以下哪项不可能为真?   A.所有的围棋爱好者也都爱好桥牌。   B.有的桥牌爱好者爱好武术。   C.健身操爱好者都爱好围棋。   D.有桥牌爱好者不爱好健身操。   E.围棋爱好者都爱好健身操。   [解题分析]正确答案:E。   由条件,有围棋爱好者爱好武术,又所有的武术爱好者都不爱好健身操,因此,有围棋爱好者不爱好健身操。所以,E项的断定不可能为真。其余各项都可能真。   -如果在上题题干中再增加一个结论:每个围棋爱好者爱好武术或者健身操,则以下哪个人的业余体育爱好的题干断定的条件矛盾?   A.一个桥牌爱好者,既不爱好武术,也不爱好健身操。   B.一个健身操爱好者,既不爱好围棋,也不爱好桥牌。   C.一个武术爱好者,爱好围棋,但不爱好桥牌。   D.一个武术爱好者,既不爱好围棋,也不爱好桥牌。   E.一个围棋爱好者,爱好武术,但不爱好桥牌。   [解题分析]正 确答案:A。   由条件,所有的桥牌爱好者都爱好围棋,又每个围棋爱好者爱好武术或者健身操,所以每个桥牌爱好者爱好武术或者健身操,即不存在桥牌爱好者,既不爱好武术,也不爱好健身操。因此,A项和题干断定的条件矛盾。   健身操武术   (五)假设反证法(归谬法)   当正确选项难于确定,错误选项又难以排除时,就应该运用假设反证法,特别是对一些综合推断型、真话假话型考题,用假设反证法非常有效。具体的做法是,先假设某一备选项成立,然后代入题干,如果出现矛盾,说明该选项不成立,要排除掉;如果只剩一项不能导致矛盾,该选项就是正确答案。   -三位股评专家正在对三家上市公司明天的股价走势进行预测。甲说:“公司一的股价会有一些上升,但不能期望过高。”乙说:“公司二的股价可能下跌,除非公司一的股价上升超过5%。”丙说:“如果公司二的股价上升,公司三的股价也会上升。”三位股评专家果然厉害,一天后的事实表明他们的预言都对,而且公司三的股价跌了。以下哪项叙述最可能是那一天股价变动的情况?   A.公司一股价上升了9%,公司二股价上升了4%。   B.公司一股价上升了7%,公司二股价下跌了3%。   C.公司一股价上升了4%,公司二股价持平。   D.公司一股价上升了5%,公司二股价上升了2%。   E.公司一股价上升了2%,公司二股价有所上升。   [解题分析]正确答案:C。   若C中叙述为真,甲的预言正确,因为公司一股价上升了4%,不算太高(与题干中说的5%比较)。乙说:“公司二的股价可能下跌,除非公司一的股价上升超过5%,其意思是“若公司一的股价上升超过5%,公司二的股价不会下跌”。现在公司一只上升了4%,所以公司二的股价有下跌的可能。但是,“可能”的意思不是“肯定”、“必然”,其结果“公司二股价持平”是与乙的预计不矛盾的。选A、D、E都不对。可以用反证法。若A、D或E对,则公司二股价上升了;考虑到丙说的话也对,可以得出公司三的股价也应该上升。这与题干中的假设矛盾。选B不对,因为它与同丙说的矛盾。   -甲(男)、乙(男)、丙(女)、丁(女)、戊(女)五个人有亲戚关系,其中凡有一个以上兄弟姐妹并且有一个以上儿女的人总说真话;凡只有一个以上兄弟姐妹或只有一个以上儿女的人,所说的话真假交替;凡没有兄弟姐妹也没有儿女的人总说假话。他们各说了以下的话:   甲:丙是我的妻子,乙是我的儿子,戊是我的姑姑。   乙:丁是我的姐妹,戊是我的母亲,戊是甲的姐妹。   丙:我没有兄弟姐妹,甲是我的儿子,甲有一个儿子。   丁:我没有儿女,丙是我的姐妹,甲是我的兄弟。   戊:甲是我的侄子,丁是我的侄女,丙是我的女儿。根据题干给定的条件,能够推出下面哪一个选项是真的?   A.甲说的话都是真话,丙是他的妻子。   B.乙说的话真假交替,他的母亲是戊。   C.丙说的话真假交替,她是甲的母亲。   D.丁说的话都是假话,她是甲的姐妹。   E.戊说的话都是真话,丙是她的姐妹。   [解题分析]正确答案A   此题用假设反证法,我们注意到每人都说了三句话,而且只有丙和丁说的第一句话含否定的,因此,尝试从丙和丁说的第一句话入手。   对任何一句话来说只有两种情况,即要么真,要么假。第一种情况:假设丙说的第一句话“我没有兄弟姐妹”为真,即丙确实没有兄弟姐妹,她不可能都说真话(因为总说真话的人必须有一个以上兄弟姐妹并且有一个以上儿女),因此,丙说的三句话只能是真假交替,既第二句话必为假,第三句话必为真。第二种情况:假设丙说的第一句话为假,则说明她有兄弟姐妹,既然丙说的话里面有了一句假话并且她有兄弟姐妹,因此她说的话只能是真假交替,则第二句话为真,也就是说她有儿子,这样,丙就有一个以上兄弟姐妹并且有一个以上儿女的人,因此她说的话应该都为真,因此与“假设丙说的第一句话为假”矛盾,所以第二种情况不可能。从中得出,丙说的话只能是第一种情况,即“真假真”,有丙说的第三句话为真,可知甲必有一个儿子。   用同样的方法,可知丁说的三句话和丙一样,也只能是“真假真”,由此得出“甲是丁的兄弟”,也就是说甲有丁这个姐妹,加上甲有一个儿子。所以,甲肯定是总说真话。因此得出A为正确答案。   -某市的红光大厦工程建设任务进行招标。有四个建筑公司投标。为简便起见,称它们为公司甲、乙、丙、丁。在标底公布以前,各公司经理分别做出猜测。甲公司经理说:“我们公司最有可能中标,其他公司不可能。”乙公司经理说:“中标的公司一定出自乙和丙两个公司之中。”丙公司经理说:“中标的若不是甲公司就是我们公司。”丁公司经理说:“如果四个公司中必有一个中标,那就非我们莫属了!”   当标底公布后发现,四人中只有一个人的预测成真了。   以下哪项判断最可能为真?   A.甲公司经理猜对了,甲公司中标了。   B.乙公司经理猜对了,丙公司中标了。   C.甲公司和乙公司的经理都说错了。   D.乙公司和丁公司的经理都说错了。   E.甲公司和丁公司的经理都说错了。   [解题分析]正确答案:C。   本题在至少考了两次。这道题可综合运用排除、列表、假设反证等方法。注意题干问的是“以下哪项判断最可能为真”,最可能为真并不等于一定为真,可以用排除法做,只要推出其他选项与题干有矛盾,就要排除掉,剩下一项没矛盾的选项就是正确答案。   先读题干,我们可以列个表,对题干中几个公司的预言进行如下归纳:   对于选项C,由于甲公司和乙公司都说错了,可以得出丁中标了。由此可知,丙错了,只有丁说对了。因此,选项C的判断与题干叙述完全符合。或者说,由选项C推不出与题干矛盾的结论。   若选项A为真,甲说对了,而且甲中标了;由此可以推出丙也说对了,与题设矛盾。 若B为真,则乙和丙都猜对了,也不符合题干假设。   若D为真,则可以推出甲中标了,从而甲、丙都猜对了,与题干不符。   若选项E为真,则甲、丁为假,可以推出乙或丙公司中标了。那么就有两种情况:一种情况是,若乙公司中标,则乙真丙假,只有一人预测成真,无矛盾;另一种情况是,若丙中标,则乙、丙均真,有矛盾。因此,若假设选项E为真,可能推出与题干假设矛盾的结论。   (六)抽象思维法   由于MBA逻辑一般并不考核各个领域的具体知识,因此,在解答考题时,并不要求考生具备各个学科的背景知识,而是要求考生在理解题目内容的基础上,结合内容去考虑其中命题之间的逻辑关系,有时则要求考生具有撇开命题内容抽取形式结构的能力。   抽象思维是做逻辑的基本方法,逻辑推理是排斥形象思维的,如果在做题时想象段落所描述的形象,而不能迅速抽象出题干的逻辑推理关系,这是做逻辑之大忌。应该说快速阅读能力和抽象思维能力是逻辑考试成功的两大关键能力。   对问题的抽象指的是把文字叙述转化为形式化的表达,也就是用一些符号、字母来表达事物间的联系,具体地说,逻辑试题内容涉及的自然或社会科学的专门知识并不影响解题,可以对题目的专有名词或有关命题用A或B等符号抽象出来即可,而不用去探究其内容的正确性,只要推理合乎逻辑就对。加快思考的考试技巧是用适当的符号或关键词去表示相应的命题、推理或论证,由于符号和关键词非常简短,可以大大节省时间,加快解题速度。   请认真体会下面这个例子:   -以下是一个西方经济学家陈述的观点:一个国家如果能有效率地运作经济,就一定能创造财富而变得富有;而这样的一个国家想保持政治稳定,它所创造的财富必须得到公正的分配;而财富的公正分配将结束经济风险;但是,风险的存在正是经济有效率运作的不可或缺的先决条件。从这个经济学家的上述观点,可以得出以下哪项结论?   A.一个国家政治上的稳定和经济上的富有不可能并存。   B.一个国家政治上的稳定和经济上的有效率运作不可能并存。   C.一个富有国家的经济运作一定是有效率的。   D.在一个经济运作无效率的国家中,财富一定得到了公正的分配。   E.一个政治上稳定的国家,一定同时充满了经济风险。   [解题分析]正确答案:B。   在做逻辑题时,关键是要用抽象思维,切忌用形象思维,比如对本题,如果你边读题边想象国家的效率、富有、稳定、公正、风险等形象,要再从中找出正确答案那是极其费劲的,正确的做法是边读题,边在草稿纸上抽象出推理关系,然后对这些推理关系进行分析即可迅速找到答案。   经过对题干的缩写,段落的观点可整理为:   (1)有效率→富有   (2)稳定→公正   (3)公正→无风险   (4)无风险→无效率   由(2)、(3)和(4)可推出:如果“稳定”,则“无效率”。这正是B项所断定的。   (七)求同与求异法   绝大部分逻辑试题都可从本质上看成是对题干段落逻辑主线的支持或削弱作用。这里提供的求同与求异法很有利于从待选选项中寻找支持与削弱的选项。   求同就是支持,即与原文的一致的选项就是支持项;求异就是削弱,即与原文的不一致的选项就是削弱项。求同、求异主要是辨认在答案满足和原文有关时是保持和原文一致还是不一致,这一点往往对于难题来讲,特别还仅仅剩下两个选项需要辨别的时候,非常有用。   例如:“因为这里有树,所以猴子有尾巴。”对这一论证的支持或驳斥不应考虑为什么树和猴子有关,而只考虑在答案中说有树或说猴子尾巴的情况下,找其相同或不同就可以了,如“相邻处没有树,猴子也有尾巴”,在满足说猴子有尾巴时说的是没有树,和上面论述不一致,因此可削弱;同理说“在邻近处没有树,猴子也就无尾巴”就有支持作用,你所需做的仅仅是寻找相同或不同点而已。   -美国黑人患高血压的比美国白人高两倍。把西方化的非洲黑人和非洲白人相比,情况也是如此。研究者假设,西方化的黑人之所以会患高血压,是两个原因相互作用的结果,一个原因是西方食品含盐量高,另一个原因是黑人遗传基因中对于缺盐环境的适应机制。   以下哪项如果是真的,最能支持研究者的假设?   A.当代西方化非洲黑人塞内加尔人和冈比亚人后裔的血压通常不高,塞内加尔和冈比亚历史上一直不缺盐。   B.非洲某些地区的不同寻常的高盐摄入是危害居民健康的严重问题。   C.考虑到保健,大多数非洲白人也注意控制盐的摄入量。   D.西非约鲁巴人的血压通常不高,约鲁巴人有史以来一直居住在远离海盐的内陆,并远离非洲撒哈拉盐矿。   E.缺盐和不缺盐对于人的新陈代谢过程没发现有什么实质的不同影响。   [解题分析]正确答案:A。   因为从题干知,研究者的假设是:西方化黑人高血压的原因,一是内因,基因中对于缺盐的适应机制,二是外因,西方食品含盐高。   这个假设等价于其逆否命题,即:如果西方化黑人血压不高,则其基因中不存在对于缺盐的适应机制或西方食品含盐不高。   本题要求寻找一个支持这个假设的选项,也就是要寻找一个与其逆否命题相一致的选项。   A项所述两个事实:一是,塞、冈两国历史上不缺盐,说明塞、冈人基因中不存在缺盐的适应机制;二是,西方化的塞、冈人的后裔也血压不高。这两个事实与上述逆否命题相一致,因此,作为一个论据,A项符合研究者的假设,所以A项起到了支持作用。   因为所讲的加上而D项不妥,因为D项所述两个事实:一是,约鲁巴人历史上缺盐,说明约鲁巴人存在缺盐的适应机制;二是,约鲁巴人血压不高。这两个事实与上述逆否命题不一致,因此,D项对研究者的假设起到了削弱作用。 第三章 逻辑备考基本方法(七) http://www.sina.com.cn 2003/10/23 16:32 新浪教育   七、标准化答题策略   MBA逻辑试题都分为:题干、问题和选项三部分,解答逻辑试题首先要审清题干的内容和意义,然后注意问题提出的角度和方式,在此基础上,通过逻辑推理,对选项进行选择。 答一道题的标准化流程 考场如战场,在分秒必争的情况下,良好的答题策略,将有助于快速准确的解题。   ·答题三步曲:读取→抽象→推理   即第一步是快速读取题干与问题;第二步是抽象出题干的逻辑主线;第三步是根据问题要求,结合题干逻辑主线与备选选项推理出答案。   ·标准化流程   (1)用不握笔的那只手将整个题目框住(或视线落点只在本题之内)。   (2)耐心阅读题干(如果必要,标出重点词语),弄清因和果(前提和结论),并在脑子里简化为一句话。这是解题的最关键的一步,即在阅读中迅速提炼出题干的“逻辑主线”,也就是明确题目的推理过程(由什么推出了什么)。   (3)再看问题,确定解题推理方向。(一定要看仔细,可不要看错或漏看)   (4)快速浏览一下五个选项。   (5)划掉绝对不可能项。   (6)最后找答案,边看选项,边看题干,边反复掂量,两相比较,找出最优。(是否需要列个箭头推出关系?能否画个图表?用假设反证法?陷阱存在吗?)   (7)划上对勾。   ·例外情形   (1)这题我见过、做过   ──快速阅读题干、问题及选项,选出最优答案。   (2)题干读了一遍没懂   ──呼一口气,再读,边读边划   (3)有两个程度相当的选项   ──其它选项的排除有否差错?这两个选项的不同之处?   (4)没有可选的选项   ──立刻重读问题,是否理解反了问题?选项中是否有一个自己理解反了?   (二)最佳解题状态——“快、准、狠”   最佳的解题状态就是要简洁明快,具体要做到“快、准、狠”:   “快”是指快速从题干中读取有用的内容;   “准”是指准确找到题干中对于回答问题所至关重要的词句及关键概念;   “狠”是指明白了问题目的后,根据关键概念,定位具有这一特点的选项,阅读后如果达到问题的目的则迅速选择,不需顾及其他选项,也就是狠心放弃阅读其他选项。   -是否公开学生的学习成绩,已成为明讯管理学院的一个热点话题。很多学生认为学习成绩是个人隐私,需要得到保护,呼吁学院不要再公开发布学生的学习成绩。学院的管理部门经过慎重的考虑,决定今后所有的学习成绩统一通过电子函件的方式发送,每个学生将只能收到自己的学习成绩。   以下各项为得知学院的这个决定后大家的一些反馈意见,其中哪项最能让学院的管理部门重新思考或修正他们的决定?   A.学习成绩在奖学金的评定、研究生录取、毕业分配等方面是重要的指标,公开发布学生的学习成绩,能够让学生都来参与和监督这方面的工作。   B.通过电子函件发送学生的学习成绩,会增加管理部门的工作量,恐怕工作人员还需要一段时间的适应。   C.部分学生尚不熟悉电子函件的收发,如果弄丢了自己的学习成绩,会给工作带来不必要的麻烦。   D.公开发布学生的学习成绩,虽然能起到一定的激励作用,但也会损伤一部分同学的自尊心。   E.电子函件的保密性并不绝对可靠,如果发生泄密,个人隐私的保护也同样会出现问题。   [解题分析]正确答案:A。   首先快速读取题干,准确找到题干的推理脉络,即“由于成绩是隐私,因此学院决定不公开”。   再看问题,“重新思考或修正”决定,标识出这是一个削弱型题目,削弱学院管理部门的政策——以电子函件发送所有的学习成绩,每个学生只能收到自己的学习成绩。即,要寻找这个政策的负面效果。   最后找答案,即要找出一个能表达“公开成绩还是有益处的”选项,即A项,该项就能有力地削弱学院的决定,如果你题做多了,就知道这是一种“存在别的因素影响推论”的削弱方式(在本书精讲篇将详细讲述),就能感觉这就是正确答案,因此可以不再看其他选项,即狠心放弃阅读其他选项。   (三)临场考试应注意的细节   新大纲与原大纲对比,逻辑部分的考试性质、考试要求、考试内容未作任何变化,实际上自1997年全国MBA联考以来,这三方面的要求一直就固定下来。这次2003年大纲的新调整只是在考试形式及试卷结构方面的要求有所变化,即虽然逻辑由50道题减少到25题,但每题有1分增加到2分,50分分值未变,在联考总分400分(另外两门管理、英语各为100分)中的比例也未变。   从大纲变化看出,考生在2003年联考中能否取得成功,关键在于综合科目能否得高分,因为同样是180分种时间,综合科目就占了200分。而逻辑作为综合科目的重要组成部分,其重要性不言而喻。   2003年MBA联考的综合科目应该是考试最紧张的一门,考试中是先做数学、语文还是先做逻辑?我们认为但对大部分考生来说,应该是先做数学,再做逻辑为好,最后做作文。   关于应试策略还可以更仔细一点考虑:从阅读和逻辑推理能力的综合水平由高到低,我们可以把考生分成逻辑高手、逻辑份子和逻辑患者这三类。经过一定时间的训练,绝大部分考生都是前二类。逻辑高手应当是在30分钟之内连做带检查将逻辑部分彻底完成;逻辑份子可以用45分钟稳稳当当地将逻辑部分做完一遍,策略上的考虑应当在全面衡量数学、写作与逻辑三者整体感觉的基础上做出;对于阅读速度较慢、逻辑推理能力较弱的逻辑患者,必须很好地衡量数学、写作与逻辑三者得分上的边际收益,必要时也不妨牺牲另一部分的时间来争取总体上的更多得分。这一点,希望大家在上考场之前就先有个比较明确的想法,当时再根据实际的卷子难度分布加以调整。   考前做好三件事:一是查漏补缺,个别还没掌握的知识点和容易出错的地方再看一看;二是规范思路,临考前几天关键已不是再做新题,而是把以前做过的题再看一遍,而且只看正确答案,不再看自己曾选错的错误答案;三是,保持良好的心态,调整到考试时达到最佳状态。   要在逻辑考试中取得高分,有三个因素起作用:首先是天生的逻辑能力,这是基础;其次是有效的复习方法,这是个关键;最后就是良好的应试技巧,这才能使所有的能力得到最好的体现。下面,结合临场考试,提供一些有效的细节,以引起考生的注意。   (1)逻辑考试的最佳状态是平和并保持适度的紧张。即:一方面不要太紧张,切忌手忙脚乱;另一方面要保持适度的紧张,要随时写、划,一刻也不闲着。 (2)逻辑考试成功关键有二:敏锐和周到,缺一不可。 (3)有人认为考试时应先看问题再读题干,但笔者认为,还是按顺序先读题干为好,因为关键是看清题干推理过程最重要,把握了题干的逻辑主线也就心中有数了,题目怎么发问都不要紧。那种追求不读完题就做题的投机取巧的做法是不可取的。   (4)临场解题时,头脑就象有张白纸,没有个人主观的预设,完全只根据题意推导,每看一题就象在一张白纸上考虑这个问题,做完后这题马上消失,立即进入下题。   (5)解题时可以把选项和题干结合起来看比较好,因为,从题干中往往可以推出很多正确的结论,所以一定要注意选项中哪些有哪些没有。   (6)要争取在该题的逻辑线上思考问题,不要偏离题目的逻辑线去钻牛角尖。“换句话说”、“其实就是说”,这种换算是解题的关键所在,也是节省时间主要的地方。决不能把简单的东西复杂化,而要把复杂的东西简单化。   (7)由于考试时间比较紧张,一般不要寄希望于有回头复查的机会。所以做逻辑时千万要看清题意,力求一遍做对,遇到个别考虑不太清楚的题,也要凭感觉作出选择,不过做完后对不把握的题要做个标记,如果有剩余时间,再回来复查。这要在平时训练时就注意,养成只检查一遍的习惯,不要过多地反复推敲,不要把一个题目的选择答案改来改去。   (8)在考试中万一碰了个钉子,遇到了点麻烦,怎么办?建议对这道题先放过去,但用三角号作个标记!(空下来没做的一定要做标记),等做完后面的再回头收拾。   (9)万一我们在考场发现个别比较模糊或是牵强的题怎么办?也要认真对待,不能带有任何别的情绪。(应该相信,考题肯定是经过命题者的仔细推敲的,绝大部分考题是没有任何问题的,当然也不排除个别具有歧义或有争议的题,但这至多只是个别情况,万一遇到了就根据第一印象选一个答案。)下面举两道MBA联考的真题:   -李昌和王平的期终考试课程共五门。他俩的成绩除了历史课相同外,其它的都不同。他俩的各门考试都及格了,即每门成绩都在60分与100分之间。   以下哪项关于上述考试的断定如果为真,使你能结合依据题干的信息,确定李昌五门课程的平均成绩高于王平。   Ⅰ李昌的最低分高于王平的最高分。   Ⅱ至少有四门课程,李昌的平均分高于王平的平均分。   Ⅲ至少有一门课程,李昌的成绩分别高于王平各门课程的成绩。   A.只有Ⅰ。   B.只有Ⅱ。   C.只有Ⅲ。   D.只有Ⅰ和Ⅲ。   E.Ⅰ、Ⅱ和Ⅲ。   [解题分析]正确答案:A。   此题出的不严谨。既然李昌和王平的历史课成绩相同,那么李昌的最低分就不可能高于王平的最高分。因此选项Ⅰ应改为“在其他四门功课中,李昌的最低分高于王平的最高分”,这样就能成立。   选项Ⅱ不成立。例如,假设有四门课,李昌的平均分是63,王平的平均分是60,而第5门课李昌的成绩是60,王平的成绩是100。这一假设符合题干的条件,并且至少有四门课程,李昌的平均分高于五平的平均分,但王平五门课程的平均成绩高于李昌。   选项Ⅲ不成立。例如,假设王平五门课的成绩都是70,而李昌的成绩分别是71、70、60、60、60。这一假设符合题干的条件,并且至少有一门课程,李昌的成绩分别高于王平各门果程的成绩,但王平五门课程的平均成绩高于李昌。   -如果上题题干的断定都真,并且事实上李昌五门课程的平均成绩高于王平,则以下哪项关于上述考试的断定一定为真?   Ⅰ李昌的最低分高于王平的最高分。   Ⅱ至少有四门课程,李昌的平均分高于王平的平均分。   Ⅲ至少有一门课程,李昌的成绩分别高于王平各门课程的成绩。   A.只有Ⅰ。   B.只有Ⅱ。   C.只有Ⅲ。   D.只有Ⅰ和Ⅲ。   E.Ⅰ、Ⅱ和Ⅲ。   [解题分析]正确答案:B。   复选项Ⅰ显然不一定是真的。   复选项Ⅱ显然是真的。因为李昌五门课程的平均成绩高于王平的平均成绩,自然至少有四门课程,李昌的平均分高于王平的平均分。   复选项Ⅲ不一定是真的。假如,假设李昌的各门成绩都是70,王平的成绩分别是71、70、60、60、60,这一假设符合题干的条件,并且李昌五门课程的平均成绩高于王平,但是不存在一门课程,李昌的成绩分别高于王平各门课程的成绩。   -一位研究人员希望了解他所在社区的人们喜欢的口味是可口可乐还是百事可乐。他找了些喜欢可口可乐的人,要他们在一杯可口可乐和一杯百事可乐中,通过品尝指出喜好。杯子上不贴标签,以免商标引发明显的偏见,只是将可口可乐的杯子标志为“M”,将百事可乐的杯子标志为“Q”。结果显示,超过一半的人更喜欢百事可乐,而非可口可乐。   以下哪项如果为真最可能削弱上述论证的结论?   A.参加者受到了一定的暗示,觉得自已的回答会被认真对待。   B.参加实验者中很多人从来都没有同时喝过这两种可乐,甚至其中的30%的参加实验者只喝过其中一种可乐。   C.多数参加者对于可口可乐和百事可乐的市场占有情况是了解的,并且经过研究证明,他们普遍有一种同情弱者的心态。   D.在对参加实验的人所进行的另外一个对照实验中,发现了一个有趣的结果:这些实验者中的大部分更喜欢英文字母Q,而不大喜观M。   E.在参加实验前的一个星期中,百事可乐的形象代表正在举行大规模的演唱会,演唱会的场地中有百事可乐的大幅宣传画,并且在电视转播中反复出现。   [解题分析]正确答案:D。   此题在2000年联考时真题有错,当时卷子上题干中写的是“……只是将可口可乐的杯子标志为“Q”,将百事可乐的杯子标志为M”……”   本题涉及到科学研究必须考虑到的一个问题,就是在对比实验中,一定要排除其他因素的干扰,才能得出可靠的结论。 如果D项为真,则事实上存在这样一种可能:许多品尝者表示更喜欢标有“Q”的杯子中的饮料,是因为更喜欢英文字母Q,而不是因为更喜欢杯中的饮料。这就削弱了题干的结论。 其余各项均不能削弱题干的结论。其中C项和E项看来似乎能削弱题干,但事实上不能,因为品尝者并不知道自已喝的实际上是何种饮料。况且,从C项,得不出百事可乐就是弱方。   (10)时间不够用了怎么办?分别对待。不到差一分钟的时候,不随便乱猜乱划。差5分钟,还有5道题,挑短的题、自己熟悉的背景知识的题做,争取拿一分是一分。 第四章 逻辑解题套路精析(一) http://www.sina.com.cn 2003/10/24 13:33 新浪教育   逻辑备考的原则是“化繁为简,思维至上,以不变应万变”。为此,本章的套路精析概括了所有逻辑考题的解题思路。不管今后的考题怎么千变万化,万变不离其宗,其题型特点和解题思路都逃不脱本章所归类剖析的内容。我们确信,这些解题套路将是逻辑考试高分突破的真正秘诀,如果考生能熟练掌握,在遇到同类问题时,一定有助于尽快理清思路,找到正确答案。   特别要指出的,本章“解题套路精析”每一类题型及其各种解题思路都是分解动作,目的是为了训练大家的解题感觉,如果感觉已形成并已熟练掌握了,那么在正式解题时就应一气呵成,而不用拘泥于具体是哪种思路了。其实逻辑题的推理过程最重要,要从繁复的叙述中看清事物间的推理关系,推理过程清楚了,什么题型都好说,很多题型是相通的。   一、假设   逻辑考题由段落、问题目的以及五个选项组成。一般而言,段落陈述论点,论点一般由论据(或前提)和结论组成。论点的结构与解答逻辑考题关系密切。在整个逻辑考题中,假设、支持、反对、评价多是围绕论点与论据设置问题。因此,在解答逻辑题时,应带有目的去读段落,这目的就是论据(或前提)和结论。而两者比较,结论比论据(或前提)更重要。   假设、支持、反对、评价这四种题型在整个逻辑推理题中占了相当大的比重,而支持、反对与评价这三种题型的答案方向多是针对段落推理的隐含假设,再加上归纳题型的推理题有时就是隐含假设,所以假设在逻辑推理中占有基础性的地位和作用。   假设题型主要表现形式有:   (1)加上一个条件就变成了一个有效的三段论推理,比如题目中问到“上面的逻辑前提是哪个?”“再加上什么条件能够得出结论”;   (2)题干给出前提和结论,然后提问你假设是什么?或者要你提出正面的事实或有利于假设的说明,则加强论点,否则削弱论点。比如问到“上文的说法基于以下那一个假设?”“上述结论中隐含着下列哪项假设?”“上述议论中假设了下列哪项前提?”   (3)以题干为结论,要求在选项中确定能推出题干的前提。或者,需要补充什么样的前提,才能使题干中的推理成为逻辑上有效的推理?   由于这种题型是题干推理中的前提不足够充分以推出结论,要求在选项中确定合适的前提,去补充原前提或论据,从而能合乎逻辑地推出结论或有利于提高推理的证据支持度和结论的可靠性。因此,做这类题的基本思路是紧扣结论,简化推理过程,从因果关系上考虑,从前提到结论,中间一定有适当的假设,寻找断路或是因为“显然”而省略掉的论述,也就是要“搭桥”,很多时候凭语感或常识就可以找到所要问的隐含的前提。   下面给出假设的具体含义:   1、假设是使推理成立的一个必要条件。   2、若一个推理在没有某一条件时,这个推理就必然不成立,那么这个条件就是段落推理的一个假设。若A是B的一个必要条件,那么非A→非B。   由于假设仅仅是推理成立的一个必要条件,所以我们找到了推理的一个假设,并不能够肯定段落推理必然成立。我们只有找到了推理成立的所有必要条件,才能够得出一个确定性的结论,推理才能够成立。不过,在考试时,我们只需要找到一个使推理成立的必要条件,尽管不能保证推理一定正确,但由于答案不需要充分性,所以就做对了“假设”的题目。请认真体会下面这个例子:   -王教授说:“总的说来,工商管理学院的大学生的家庭困难情况比起以前有了大幅度的改观。这种情况十分明显,因为我的学生现在课余要求学校安排勤工俭学的人越来越少了”。   上面的结论是由下列哪个假设得出的?   A.现在大学生父母亲的收入随着改革开放的深入发展而增加,使得大学生不再需要勤工俭学来自己养活自己了。   B.尽管家境有了改善,也应当参加勤工俭学来锻炼自己的全面能力。   C.课余要求学校安排勤工俭学是学生家庭是否困难的一个重要标志。   D.大学生把更多的时间用在学业上,勤工俭学的人就少起来了。   E.学校安排的勤工俭学报酬相对越来越低,不能满足学生的要求。   [解题分析]正确答案:C。   本题的逻辑主线是“越来越少的王教授的学生要求勤工俭学”,就推出“工商管理学院的大学生的家庭困难情况比起以前有了大幅度的改观”。我们发现,本题推理的前提对象与结论讨论的对象属性并不完全一样,前提与结论之间有差异,即前提与结论之间有明显的跳跃:做勤工俭学的少就必然家庭困难有了改观吗?因此段落推理成立需要依赖选项(C)这个假设“课余要求学校安排勤工俭学是学生家庭是否困难的一个重要标志”,即选项(C)把前提与结论之间的一个跳跃连了起来,所以(C)正确。   其实,本题推理的成立还依赖两个假设,即①王教授教的学生是工商管理学院的学生;②王教授教的学生是整个工商管理学院学生的代表。在这三个假设的共同作用下,本题推理必然成立。但由于“答案不需要充分性”,我们只需要找出(C),就是找到了一个假设,所以(C)就是正确答案,但正是由于我们仅仅只是找到了推理成立的一个假设,所以我们说假设题型(包括支持、反对、评价)的段落推理是“有待评价的推理”。   那么,如何来排除不是假设的选项呢?我们可以加入否定来判断。根据假设的定义,若某条件是推理成立所做的假设,那么没有这个条件,推理必不成立。如果某备选项加入否定以后,对段落推理不起作任何作用,推理仍然可以成立,那么该备选项必然不是一个假设。请看下面这个例题:   -据最近的统计,在需要同等学历的十个不同职业中,教师的平均工资五年前排列第九,而目前上升到第六;另外,目前教师的平均工资是其它上述职业的平均工资的86%,而五年前只是55%。因此,教师工资相对偏低的状况有了较大的改善,教师的相对生活水平有了很大的提高。   上述论证基于以下哪项假设?   I.近五年来的通货膨胀率基本保持稳定。   II.和其他职业一样,教师中的最高工资和最低工资的差别是很悬殊的。   III.学历是确定工资标准的主要依据。   IV.工资是实际收入的主要部分。   A.仅I、III。   B.仅II、IV。   C.仅III。 D.仅IV。 E.仅III、IV。   [解题分析]正确答案:D   什么是假设?就是把待选的选项取否定后放到段落中去,能使段落推理不成立的选项就是正确答案。   I.并不是需要假设的,即如果近五年来的通货膨胀率不稳定,也不影响题干段落推理的成立。   II.并不是需要假设的,即如果教师中的最高工资和最低工资的差别不大,也不影响题干段落推理的成立。   Ⅲ并不是需要假设的,即如果学历并不是确定工资标准的主要依据,不影响题干段落推理的成立。(题干只是对同等学历的十个职业的收入作比较而已,并不隐含学历是确定工资标准的主要依据,也许职称职位的作用更大)   IV是必须假设的,否则,工资如果不是实际收入的主要部分,则即使教师工资相对偏低的状况有了较大的改善,教师的相对生活水平也不一定有很大的提高。   (一)前提与结论之间有本质联系   若题干的前提与结论之间有明显的跳跃,那么,这个段落推理成立所隐含的一个假设是前提的讨论对象与结论的讨论对象是有本质联系的,这就是所谓的“搭桥”。请体会如下推理:   “科学是真理,因此,科学是不怕批评的。”   我们发现前提的探讨对象是“真理”,而结论的探讨对象是“不怕批评的”,它们之间有差异,因此这个推理要成立,必须要求前提与结论之间是有本质联系,即需“真理是不怕批评的”这一假设。反之,若没有这个假设,则段落推理必然不成立。   因此,当假设思路为“前提与结论之间有本质联系”时,主要是寻找段落推理的核心关键词,且用此核心关键词来定位选项。   -某年,国内某电视台在综合报道了当年的诺贝尔各项奖金的获得者的消息后,做了以下评论:今年又有一位华裔科学家获得了诺贝尔物理学奖,这是中国人的骄傲。但是到目前为止,还没有中国人获得诺贝尔经济学奖和诺贝尔文学奖,看来中国在人文社会科学方面的研究与世界先进水平相比还有比较大的差距。   以上评论中所得出的结论最可能把以下哪项断定作为隐含的前提?   A.中国在物理学等理科研究方面与世界先进水平的差距在逐步缩小。   B.中国的人文科学有先进的理论基础和雄厚的历史基础,目前和世界先进水平的差距是不正常的。   C.诺贝尔奖是衡量一个国家某个学科发展水平的重要标志。   D.诺贝尔奖的评比在原则上对各人种是公平的,但实际上很难做到。   E.包括经济学在内的人文社会科学研究与各国的文化传统有非常密切的联系。   [解题分析]正确答案:C。   题干的逻辑主线是“没有中国人获得诺贝尔经济学奖和诺贝尔文学奖→中国在人文社会科学方面的研究与世界先进水平相比还有比较大的差距”,其推理成立的假设就是要说明这两者之间有本质联系。选项C就类似一个三段式推论的前提,如果没有隐含C,那么,就无法得出题干结论。其他的选项均不是题干推论所必须假设的,其中选项D还对题干推论有质疑。   -人类学家发现早在旧石器时代,人类就有了死后复生的信念。在发掘出的那个时代的古墓中,死者的身边有衣服、饰物和武器等陪葬物,这是最早的关于人类具有死后复生信念的证据。   以下哪项,是上述议论所假定的?   A.死者身边的陪葬物是死者生前所使用过的。   B.死后复生是大多数宗教信仰的核心信念。   C.宗教信仰是大多数古代文明社会的特征。   D.放置陪葬物是后人表示对死者的怀念与崇敬。   E.陪葬物是为了死者在复生后使用而准备的。   [解题分析]正确答案:E。   题干的逻辑主线是“旧石器时代就有了陪葬物→旧石器时代人类就有了死后复生的信念”,其推理成立的假设就是要说明这两者之间有本质联系。E项是题干必须假设的。否则,如果陪葬物的安放是出于其他目的,那么就不能把死者身边的这些陪葬物作为人类具有死后复生信念的证据。其余各项均不是必须假设的。   -前年引进美国大片《廊桥遗梦》,仅仅在滨洲市放映了一周时间,各影剧院的总票房收入就达到八百万元。这一次滨洲市又引进了《泰坦尼克号》,准备连续放映10天,一千万元的票房收入应该能够突破。   根据上文包括的信息,分析以上推断最可能隐含了以下哪项假设?   A.滨洲市很多人因为映期时间短都没有看上《廊桥遗梦》,这一次可以得到补偿。   B.这一次各影剧院普遍更新了设备,音响效果比以前有很大改善。   C.这两部片子都是艺术精品,预计每天的上座率、票价等非常类似。   D.连续放映10天是以往比较少见的映期安排,可以吸引更多的观众。   E.灾难片加上爱情片,《泰坦尼克号》的影响力和票房号召力是巨大的。   [解题分析]正确答案:C。   题干的逻辑主线是“《廊桥遗梦》票房收入好→《泰坦尼克号》票房收入好”,其推理成立的假设就是要说明这两者之间有本质联系。C项是题干的推断最可能假设的。否则,如果两部影片在上座率、票价方面有明显差异,题干对《泰坦尼克号》的预计显然缺乏说服力。其余选项作为答案均不恰当。   要在两样东西或是两种方法间类推,很重要的一点就是这两者之间具有可类比类推性。这也是一个常考的考点,对于本题来说,映期上可以类比,剩下的就是票价和上座率了,这些指标的综合就是一部影片的票房。   (二)推论可行或有意义   若能使一个推论可行或有意义,那么这样的假定就是段落推理成立的必要条件。因为若推论根本就不可行或没有实际意义,那么段落推理必然不成立,所以这个假定是假设。   请体会这个例子:某企业为减人增效,决定把最无价值的员工精简掉。   本题推理为“为达到一个目的而提出一个方法”,这个推理若想成立,那么依赖的假设是“该企业能够辨别员工是否有价值”,即“推论可行”。若这个学校不能辨别员工是否有价值,那么就不可能达到“减人增效”的目的。所以“推论可行或有意义”是推理成立的一个很好的假设。   同时,我们知道,若A是B的一个必要条件,可表示为B→A,那么其逆否命题为“非A→非B”且与原命题等价。换句话说,由于假设是推理成立的必要条件,若我们能得出“非A→非B”,那么我们就可以得出 A是推理成立的必要条件,即“非A→非B”是段落推理成立所依赖的一个假设。 -心脏的搏动引起血液循环。对同一个人,心率越快,单位时间进入循环的血液量越多。血液中的红血球运输氧气。一般地说,一个人单位时间通过血液循环获得的氧气越多,他的体能及其发挥就越佳。因此,为了提高运动员在体育比赛中的竞技水平,应该加强他们在高海拔地区的训练,因为在高海拔地区,人体内每单位体积血液中含有的红血球数量,要高于在低海拔地区。   以下哪项是题干的论证必须假设的?   A.海拔的高低对运动员的心率不发生影响。   B.不同运动员的心率基本相同。   C.运动员的心率比普通人慢。   D.在高海拔地区训练能使运动员的心率加快。   E.运动员在高海拔地区的心率不低于在低海拔地区。   [解题分析]正确答案:E。   E项是题干的论证必须假设的,否则,如果事实上运动员在高海拔地区的心率低于在低海拔地区,那么即使在高海拔地区,人体内每单位体积血液中含有的红血球数量,要高于在低海拔地区,但由于心率较慢,单位时间进入循环的轿液量较少,因而单位时间里血液中运输氧气的白血球并不见得就多,因而通过血液循环获得的氧气并不见得就多,因而在高海拔地区训练的运动员的体能及其发挥并不能较佳。   A项的断定过强,不是题干的论证必须假设的。假如,如果事实上海拔越高,运动员的心率越快,即A项的断定不成立,但题干的论证并不因此不成立。   D项能加强题干的论证,但同样不是题干的论证必须假设的。   其余各项均不是题干的论证必须假设的。   -有的地质学家认为,如果地球的未勘探地区中单位面积的平均石油储藏量能和已勘探地区一样的话,那么,目前关于地下未开采的能源含量的正确估计因此要乘上一万倍,由此可得出结论,全球的石油需求,至少可以在未来五个世纪中得到满足,即便此种需求每年呈加速上升的趋势。为使上述论证成立,以下哪项是必须假设的?   A.地球上未勘探地区的总面积是已勘探地区的一万倍。   B.地球上未勘探地区中储藏的石油可以被勘测和开采出来。   C.新技术将使未来对石油的勘探和开采比现在更为可行。   D.在未来至少五个世纪中,石油仍然是全球主要的能源。   E.在未来至少五个世纪中,世界人口的增长率不会超过对石油需求的增长率。   [解题分析]正确答案:B。   题干的结论是:全球的石油需求,至少可以在未来五个世纪中得到满足,即使此种需求每年呈加速上升的趋势。其根据是:根据地质学家的观点,目前包括未勘探地区在内的地下未开采的能源含量比原来估计的要多一万倍。   要使这一论证成立,有一个条件必须满足,即地球上未勘探地区中储藏的石油事实上可以被勘测和开采出来。B项正是断定了这一点。因此,B项是题干的论证必须假设的。   -尽管计算机可以帮助人们进行沟通,计算机游戏却妨碍了青少年沟通能力的发展。他们把课余时间都花费在玩游戏上,而不是与人交流上。所以说,把课余时间花费在玩游戏上的青少年比其他孩子有较少的沟通能力。   以下哪项是上述议论最可能假设的?   A.一些被动的活动,如看电视和听音乐,并不会阻碍孩子们的交流能力的发展。   B.大多数孩子在玩电子游戏之外还有其他事情可做。   C.在课余时间不玩电子游戏的孩子至少有一些时候是在与人交流。   D.传统的教育体制对增强孩子们与人交流的能力没有帮助。   E.由玩电子游戏带来的思维能力的增强对孩子们的智力开发并没有实质性的益处。   [解题分析]正确答案:C。   题干的逻辑主线是“课余时间都在玩游戏→沟通能力差”。C项是题干的议论必须假设的,否则,如果事实上在课余时间不玩电子游戏的孩子在任何时候都不与人交流,那么,就不能根据青少年在课余时间玩游戏而不是与人交流,就得出结论,把课余时间花费在玩游戏上的青少年比其他孩子缺少沟通能力。   B项也是题干的议论应当假设的。但由于B项断定的是大多数孩子,而不是所有的孩子,因此,题干对于B项的依赖程度弱于C项。其余各项均不是需要假设的。   -新一年的电影节的影片评比,准备打破过去的只有一部最佳影片的限制,而按照历史片、爱情片等几种专门的类型分别评选最佳影片,这样可以使电影工作者的工作能够得到更为公平的对待,也可以使观众和电影爱好者对电影的优劣有更多的发言权。   以下哪项假设最可能是上述评比制度改革隐含的前提?   A.划分影片类型,对于规范影片拍摄有重要的引导作用。   B.每一部影片都可以按照这几种专门的类型来进行分类。   C.观众和电影爱好者在进行电影评论时喜欢进行类型的划分。   D.按照类型来进行影片的划分,不会使有些冷门题材的影片被忽视。   E.过去因为只有一部最佳影片,影响了电影工作者参加电影节评比的积极性。   [解题分析]正确答案:B。   选项B显然是上述评比制度改革所必须的,否则无法分类,则制度改革无从谈起。至于选项A、C、D、E,都正面支持了这种制度改革,但是否一定是先决条件呢,理由似乎都不充分。   -以前有几项研究表明,食用巧克力会增加食用者患心脏病的可能性。而一项最新的、更为可靠的研究得出的结论为:食用巧克力与心脏病发病率无关。估计这项研究成果公布之后,巧克力的消费量将会大大增加。上述推论基于以下哪项假设?   A.大量食用巧克力的人中,并不是有很高的比例患心脏病。   B.尽管有些人知道食用巧克力会增加患心脏病的可能性,却照样大吃特吃。   C.人们从来也不相信进食巧克力会更容易患心脏病的说法。   D.现在许多人吃巧克力是因为他们没听过巧克力会导致心脏病的说法。   E.现在许多人不吃巧克力完全是因为他们相信巧克力会诱发心脏病。   [ 解题分析]正确答案:E。 题干的逻辑主线是“食用巧克力不会导致患心脏病→巧克力的消费量将会大大增加”。E项是题干的推论必须假设的。否则,如果许多人不吃巧克力不是因为他们相信巧克力会诱发心脏病,那么,即使他们知道了食用巧克力与心脏病无关,也不会因此转而购买巧克力。这就使题干的推理无法成立。其余各项不是必须假设的。   (三)没有别的因素影响推论   当段落推理是要达到的一个目的而提出一个方法或建议,或由一个调查、数据、研究或实验等而得出解释性结论时,推理成立所做的隐含假设多为“没有别的因素影响推论”。请体会这个例子:   “虽然美国和加拿大内科医生占人口总数的比例大致相同,但美国人均拥有的外科医生比加拿大多33%。显然,正是由于这个原因,在美国平均每人做的手术比加拿大人多40%”上面解释依据的假设是什么呢?   由于题干的推理是“美国人均医生多,因此,人均手术也多”,其假设是“没有别的因素影响推论”,即“美国患者对手术的需要不如加拿大的患者多”是必须假设的,否则,如果“美国患者对手术的需要确实比加拿大的患者多”,那么,就不能说明“由于美国人均医生多才造成人均手术多的”。   可见,当假设思路为“没有别的因素影响推论”时,那么假设答案中大多有否定概念的出现(相当于没有某物的变化因素)。   -政府应该不允许烟草公司在其营业收入中扣除广告费用。这样的话,烟草公司将会缴纳更多的税金。它们只好提高自己的产品价格,而产品价格的提高正好可以起到减少烟草购买的作用。   以下哪个选项是上述论点的前提?   A.烟草公司不可能降低其他方面的成本来抵消多缴的税金。   B.如果它们需要付高额的税金,烟草公司将不再继续做广告。   C.如果烟草公司不做广告,香烟的销售量将受到很大影响。   D.政府从烟草公司的应税收入增加所得的收入将用于宣传吸烟的害处。   E.烟草公司由此所增加的税金应该等于价格上涨所增加的盈利。   [解题分析]正确答案:A。   题干的逻辑主线是“缴纳更多的税金→只好提高自己的产品价格”,要使推理成立,必须保证没有别的因素影响这个推论。A项是题干的论证所必须假设的。否则,如果事实上烟草公司可以通过降低其他方面的成本,来抵消因为不扣除广告费用而多缴的税金,那么,烟草公司就不会因此被迫提高价格而减少烟草的销售。这就动摇了题干结论的说服力。E项不是题干必须假设的。题干确实必须假设,因扣除广告费用所增加的税金,会对烟草公司产生压力而迫使它提高产品价格;但是不必过强地假设,烟草公司由此所增加的税金应该等于价格上涨所增加的盈利。   -在当前的音像市场上,正版的激光唱盘和影视盘销售不佳,而盗版的激光唱盘和影视盘却屡禁不绝,销售异常火爆。有的分析人员认为这主要是因为在价格上盗版盘更有优势,所以在市场上更有活力。   以下哪项是这位分析人员在分析中隐含的假定?   A.正版的激光唱盘和影视盘往往内容呆板,不适应市场的需要。   B.与价格的差别相比,正版与盗版盘在质量方面的差别不大。   C.盗版的激光唱盘和影视盘比正版的盘进货渠道畅通。   D.正版的激光唱盘和影视盘不如盗版的盘销售网络完善。   E.加强对知识产权的保护和对盗版行为的打击使得盗版盘的价格上涨。   [解题分析]正确答案:B。   题干的逻辑主线是“盗版盘比正版盘便宜→盗版盘比正版盘销售好”,要使推理成立,必须保证没有别的因素影响这个推论。盗版盘价格上有优势,所以就畅销,这中间暗含着一个假设,就是盗版盘的质量差别不大,至少比它那个价要值得多。其实还有的一个假设就是不会毁机器。这在选项中没有,要有也该选。   (四)综合假设   在解答假设题时,首先应能寻找出段落推理的论点与已存在的前提,并利用段落中的核心关键词来定位选项;其次用上面的三种思维方向去判断该选项是否为正确答案;最后,当只剩下两个选项而不知谁为假设时,则通过对选项加入否定的方法来判断推理是否成立。若加入否定,段落推理必不成立,则其必为假设;若加入否定,段落推理仍可成立,则绝对不是假设。   注意:假设一定是支持,但支持不一定是假设。因此,命题者加大假设题难度无一例外地是在加大阅读的前提下设计出一个支持选项,而这时的易混淆支持选项必然不是段落推理成立的必要条件,所以可用加入否定的方法去掉这个易误选的支持答案。   由于在考试时,不可能也没必要迅速判断假设究竟属于哪一类,所以在平时练习假设题型时,要训练迅速读出段落推理的论点与已存在的前提的能力,然后根据其关系做出判断。本部分“综合假设”指的是要使段落推理成立,需要多个假设,是前面三个假设方向的综合运用。   -清朝雍正年间,市面流通的铸币,其金属构成是铜六铅四,即六成为铜,四成为铅。不少商人出以利计,纷纷融币取铜,使得市面的铸币严生匮乏,不少地方出现以物易物。但朝廷征于市民的赋税,须以铸币缴纳,不得代以实物或银子。市民只得以银子向官吏购兑铸币用以纳税,不少官吏因此大发了一笔。这种情况,雍正之前的明清两朝历代从未出现过。从以上陈述,可推出以下哪项结论?   I.上述铸币中所含铜的价值要高于该铸币的面值。   II.上述用银子购兑铸币的交易中,不少并不按朝廷规定的比价成交。   III.雍正以前明清两朝历代,铸币的铜含量,均在六成以下。   A.只有I。   B.只有II。   C.只有III。   D.只有I和II。   E.I、II和III。   [解题分析]正确答案:D。   Ⅰ可以从题干的陈述中推出。因为如果事实上上述铸币中所含铜的价值不高于该铸币的面值,那么融币取铜就会无利可图,就不会出现题干中所说的商人纷纷融币取铜,从而造成市面铸币严重匮乏的现象。   Ⅱ可以从题干的陈述中推出。因为如果上述银子购兑铸币的交易,都能严格按朝廷规定的比价成交,就不会有官吏通过上述交易大发一笔,题干中陈述的盯关现象就不会出现。   Ⅲ不能从题干的陈述中推出。铸币铜含量在六成以上,有可能导致商人融币取铜,但不一定导致商人纷纷融币取铜,例如,如果有严明的王法。因此,不能由雍正以前明清两朝历代未见有题干陈述的现象,就得出其铸币铜含量均在六成以下的结论。   -最近五年来,共有五架W-160客机失事。面对W-160设计有误的指控,W-160的生产厂商明确加以否定,其理由是,每次W-160空难的调查都表明,失事的原因是飞行员的操作失误。   为使厂商的上述反驳成立,以下哪项是必须假设的?   Ⅰ如果飞行员不操作失误,W-160就不会失事。 Ⅱ飞行员的操作失误,和W-160任一部分的设计都没有关系。 Ⅲ每次对W-160空难的调查结论都可信。   A.只有Ⅰ。   B.只有Ⅱ。   C.只有Ⅲ。   D.只有Ⅱ和Ⅲ。   E.Ⅰ、Ⅱ和Ⅲ。   [解题分析]正确答案:D。   复选项Ⅰ不是必须假设的。厂商的反驳需要假设的是:如果飞行员不操作失误,W-160也会失事,但这种失事和W-160的设计无关。   复选项Ⅱ是必须假设的。否则,如果飞行员的操作失误,和W-160的设计有关,那么,就不能否定对W-160设计有误的指控。   复选项Ⅲ是必须假设的。否则,如果对W-160空难的调查结论有的不可信,那就题干中厂商的反驳的根据也就不可信。   -无论是工业用电还是民用电,现行的电费价格一直偏低。某区推出一项举措,对超出月额定数的用电量,无论是工业用电还是民用电,一律按上调高价收费。这一举措将对该区的节约用电产生重大的促进作用。上述举措要达到预期的目的,以下哪项必须是真的?   Ⅰ.有相当数量的浪费用电是因为电价格偏低而造成的。   II.有相当数量的用户是因为电价格偏低而浪费用电的。   III.超额用电价格的上调幅度一般地足以对浪费用电的用户产生经济压力。   A.Ⅰ、Ⅱ和Ⅲ。   B.仅Ⅰ和Ⅱ。   C.仅Ⅰ和Ⅲ。   D.仅Ⅱ和Ⅲ。   E.Ⅰ、Ⅱ和Ⅲ都不必须是真的。   [解题分析]正确答案:C。   Ⅰ是必须假设的,否则,如果相当数量的浪费用电不是因为电价格偏低而造成的,那么,提高价格也无助于节电。   容易把“有相当数量的用户是因为电价格偏低而浪费用电的”这个选项也选进来,其实不妥。比如尽管相当数量的用户因为电价低而浪费用电,但浪费的总量很有限,在浪费用电的总量中有相当数量的浪费用电是因为公费支出的(所以大家浪费不在乎)或是缺乏节电意识造成的。那么你提高电价就不一定达到目的,而不如采取别的措施(比如限制公费缴纳电费的总额或是加强宣传节电的力度等等)了。   -许多影视放映场所为了增加其票房收入,把一些并不包含有关限制内容的影视片也标以“少儿不宜”。他们这样做是因为确信以下哪项断定?   Ⅰ.成年观众在数量上要大大超过少儿观众。   Ⅱ.“少儿不宜”的影视对成年人无害。   Ⅲ.成年人普遍对标明“少儿不宜”的影视片感兴趣。   A.仅Ⅰ。   B.仅Ⅱ。   C.仅Ⅰ、Ⅲ。   D.仅Ⅱ、Ⅲ。   E.Ⅰ、Ⅱ、Ⅲ。   [解题分析]正确答案:C。   首先,要看到把本不属于需要限制的影片标为“少儿不宜”是一种欺骗行为,其目的是为了吸引观众,增加盈利。若注意到影片标上“少儿不宜”会减少从少儿观众那里得到的收入,影院一定是希望吸引更多的成人观众。因此,影视放映场所确信Ⅰ和Ⅲ的叙述是合乎情理的。至于Ⅱ,“少儿不宜”的影响片是否对成年人无害,不会是搞歪门邪道的影视放映场所的负责人所关心的。显而易见,其他选项不妥。   -某家私人公交公司通过增加班次、降低票价、开辟新线路等方式,吸引了顾客,增加了利润。为了使公司的利润指标再上一个台阶,该公司决定更换旧型汽车,换上新型大客车,包括双层客车。   该公司的上述计划假设了以下各项,除了   A.在该公司经营的区域内,客流量将有增加。   B.更换汽车的投入费用将在预期的利润中得到补偿。   C.新汽车在质量、效能等方面足以保证公司获得预期的利润。   D.驾驶新汽车将不比驾驶旧汽车更复杂、更困难。   E.新换的双层大客车在该公司经营的区域内将不会受到诸如高度、载重等方面的限制。   [解题分析]正确答案:D。 选项A、B、C和E都是公司实行新计划所必须假设,否则新计划就不会获得预期的收益。选项D讨论驾驶新旧汽车复杂性的不同,这与利润指标的实现没多大关系 第四章 逻辑解题套路精析(二) http://www.sina.com.cn 2003/10/24 13:54 新浪教育   二、支持   在MBA逻辑考试中,围绕前提和结论之间的支持或反驳关系,设计了多种形式的考题,主要有加强前提型和削弱结论型。   前提对结论的支持或反驳程度,有许多推理或论证尽管不满足保真性,即前提的真不 能确保结论的真,但前提却对结论提供一定程度的支持,或者前提对结论构成一定程度的反驳。一个推理的证据支持度越高,则在前提真实的条件下,推出的结论可靠性越大。在批判性思维以及MBA逻辑考试中,重点考察的就是思维的论证性,即对各种已有的推理或论证做批判性评价:对某个论点是否给出了理由?所给出的理由真实吗?与所要论证的论点相关吗?如果相关,对论点的支持度有多高?是必然性支持(若理由真,则论点或结论必真)还是或然性支持(若理由真,结论很可能真,也可能是假)?是强支持还是弱支持?给出什么样的理由能够更好地支持该结论?给出什么样的理由能够有力地驳倒该结论,或者至少是削弱它?   支持型考题的特点是在段落中给出一个推理或论证,但或由于前提的条件不够充分,不足以推出其结论;或者由于论证的论据不够全面,不足以得出其结论,因此需用某一选项去补充其前提或论据,使推理或论证成立的可能性增大。但由于“答案不需要充分性”的原则,所以只要某一选项放在段落推理的论据(前提)或结论之间,对段落推理成立或结论正确有支持作用,使段落推理成立、结论正确的可能性增大,那么这个选项就是支持的正确答案。所以支持的答案既可以是段落推理成立或结论正确的一个充分条件,也可以是一个必要条件(这时等同于假设,因为假设答案必将可以支持推理),可以是非充分条件,也可以是非必要条件。   (一)前提与结论之间有联系   如果支持题型的逻辑主线的前提和结论之间存在跳跃,要支持结论或论证,就必须“搭桥”,即找一个选项把前提与结论联系起来。   -壳牌石油公司连续三年在全球500家最大公司净利润总额排名中位列第一,其主要原因是该公司比其他公司有更多的国际业务。   下列哪项如果为真,则最能支持上述说法?   A.与壳牌公司规模相当但国际业务少的石油公司的利润都比壳牌石油公司低。   B.历史上全球500家大公司的净利润冠军都是石油公司。   C.近三年来全球最大的500家公司都在努力走向国际化。   D.近三年来石油和成品油的价格都很稳定。   E.壳牌石油公司是英国和荷兰两国所共同拥有的。   [解题分析]正确答案:A。   本题的逻辑主线是“国际业务多→利润高”,这两者之间存在跳跃,必须“搭桥”,即找一个选项把“国际业务”与“利润”这两者之间联系起来。   与同类的(规模相当)的石油公司比较,壳牌石油公司位列第一,原因可能有多种。但A中举出它们的国际业务量不同,可以作为题干中说的“主要原因是该公司比其他公司有更多的国际业务”的论据。   B、D与国际化无关;C谈的是“500家公司”都在“国际化”,而题干强调的是各公司的国际化程度的差异;E好像与国际业务有一定的关系,但国际业务的多少与公司是否由多国共管并无直接的关系。   -全国政协常委、著名社会学家、法律专家钟万春教授认为:我们应当制定全国性的政策,用立法的方式规定父母每日与未成年子女共处的时间下限。这样的法律能够减少子女平日的压力。因此,这样的法律也就能够使家庭幸福。   以下各项如果为真,哪项最能够加强上述的推论?   A.父母有责任抚养好自已的孩子,这是社会对每一个公民的起码要求。   B.大部分的孩子平常都能够与父母经常地在一起。   C.这项政策的目标是降低孩子们在平日生活中的压力。   D.未成年孩子较高的压力水平是成长过程以及长大后家庭幸福很大的障碍。   E.父母现在对孩子多一分关心,就会减少日后父母很多的操心。   [解题分析]正确答案:D。   本题的逻辑主线是“这样的法律能够使子女压力减少→这样的法律能够使家庭幸福”,这两者之间存在跳跃,必须“搭桥”,即找一个选项把“子女压力减少”与“家庭幸福”这两者之间联系起来。   D项如果为真,则说明能够减少未成年孩子压力的法律,有利于排除家庭幸福潜在的障碍,因此,这样的法律能够使家庭幸福。因此,D项有力地加强了题干的推论。E项对题干有所加强,但力度不如D项,其余项不能加强题干。   (二)推论可行或有意义   如果支持题型的某个备选选项,是题干推理成立的必要条件,也就是说该选项的存在使题干推论可行或有意义,那么该选项就是正确答案。   由于假设是段落推理的必要条件,找到了段落推理的一个假设,那么其推理成立的可能性就必然增大,这个假设对段落推理起到了支持作用,所以假设必然是支持,因此这类支持题型相当于寻找段落推理成立的一个假设。   -在美国,近年来在电视卫星的发射和操作中事故不断,这使得不少保险公司不得不面临巨额赔偿,这不可避免地导致了电视卫星的保险金的猛涨,使得发射和操作电视卫星的费用变得更为昂贵。为了应付昂贵的成本,必须进一步开发电视卫星更多的尖端功能来提高电视卫星的售价。以下哪项,如果为真,和题干的断定一起,最能支持这样一个结论,即电视卫星的成本将继续上涨?   A.承担电视卫星保险业风险的只有为数不多的几家大公司,这使得保险金必定很高。   B.美国电视卫星业面临的问题,在西方发达国家带有普遍性。   C.电视卫星目前具备的功能已能满足需要,用户并没有对此提出新的要求。   D.卫星的故障大都发生在进入轨道以后,对这类故障的分析及排除变得十分困难。   E.电视卫星具备的尖端功能越多,越容易出问题。   [解题分析]正确答案:E。   由题干,进一步开发电视卫星更多的尖端功能的初衷是提高电视卫星的售价,用以应付昂贵的成本。如果E项为真,则电视卫星具备的尖端功能越多,越容易出问题,因而又将导致保险金的新一轮上涨,使得电视卫星的成本继续上涨。 其余各项不足以说明电视卫星的成本将继续上涨。 -喜欢甜味的习性曾经对人类有益,因为它使人在健康食品和非健康食品之间选择前者。例如,成熟的水果是甜的,不成熟的水果则不甜,喜欢甜味的习性促使人类选择成熟的水果。但是,现在的食糖的经过精制的。因此,喜欢甜味不再是一种对人有益的心性,因为精制食糖不是健康食品。以下哪项如果为真,最能加强上述论证?   A.绝大多数人都喜欢甜味。   B.许多食物虽然生吃有害健康,但经过烹饪则可成为极有营养的健康食品。   C.有些喜欢甜味的人,在一道甜点心和一盘成熟的水果之间,更可能选择后者。   D.喜欢甜味的人,在含食糖的食品和有甜味的自然食品(例如成熟的水果)之间,更可能选择前者   E.史前人类只有依赖味觉才能区分健康食品。   [解题分析]正确答案:D。   题干的结论是喜欢甜味不再是一种对人有益的习性,其根据是有甜味的精制食糖不是健康食品。要使题干的论证成立,有一个条件必须满足,即喜欢甜味的人,在含食糖的食品和有甜味的自然食品(例如成熟的水果)之间,更可能选择前者。否则,只要有甜味的自然食品(例如成熟的水果)仍然是人可供选择并更喜欢的食品,就没有理由认为喜欢甜味这种习性不再对人有益。D项正是断定了这一点。因此,如果D项为真,能有力地加强题干的论证。   其余各项均不能加强题干,其中C项削弱了题干。   -经A省的防疫部门检测,在该省境内接受检疫的长尾猴中,有1%感染上了狂犬病。但是只有与人及其宠物有接触的长尾猴才接受检疫。防疫部门的专家因此推测,该省长尾猴中感染有狂犬病的比例,将大大小于1%。   以下哪项如果为真,将最有力地支持专家的推测?   A.在A省境内,与人及其宠物有接触的长尾猴,只占长尾猴总数的不到10%。   B.在A省,感染有狂犬病的宠物,约占宠物总数的0.1%。   C.在与A省毗邻的B省境内,至今没有关于长尾猴感染狂犬病的疫情报告。   D.与和人的接触相比,健康的长尾猴更愿意与人的宠物接触。   E.与健康的长尾猴相比,感染有狂犬病的长尾猴更愿意与人及其宠物接触。   [解题分析]正确答案:E。   题干的逻辑主线是“受检猴都与人及其庞物有接触,其患病率为1%→猴患病率大大小于1%”,E项的存在能使这个推理成立。如果E项的断定为真,又根据题干,只有与人及其宠物有接触的长尾猴才接受检疫,则说明在接受检疫的长尾猴中感染狂犬病的比例,要高于未接受检疫的长尾猴。这就有力地支持了专家的推测。其余各项均不能支持专家的推测。   -美国联邦所得税是累进税,收入越高,纳税率越高。美国有的州还在自己管辖的范围内,在绝大部分出售商品的价格上附加7%左右的销售税。如果销售税也被视为所得税的一种形式的话,那么,这种税收是违背累进原则的:收入越低,纳税率越高。   以下哪项,如果为真,最能加强题干的议论?   A.人们花在购物上的钱基本上是一样的。   B.近年来,美国的收入差别显著扩大。   C.低收入者有能力支付销售税,因为他们缴纳的联邦所得税相对较低。   D.销售税的实施,并没有减少商品的销售总量,但售出商品的比例有所变动。   E.美国的大多数州并没有征收销售税。   [解题分析]正确答案是A。   题干的议论要能成立,必须基于一个假设,即人们花在购物上的钱基本上是一样的。否则,如果收入越高,花在购物上的钱越多,则销售税并没有违背累进税的原则:收入越高,纳税率越高。因此,A项能有力加强题干。其余各项均不能加强题干。   (三)没有别的因素影响推论   如果支持题型的题干是有一个调查、研究、数据或实验等得出一个解释性的结论时,那么“没有别的因素影响推论”就是支持其结论或论证的一种有效方式。   -帕金森氏病是一种严重危害大脑的疾病。那些在体内不能生成细胞色素P405的人,和那些体内能生成这种细胞色素的人相比,在他们进入中老年后,患帕多森病的可能性要大三倍。因为细胞色素P405具有使大脑免受有毒化学物质侵害的功能,所以有害化学物质很可能是造成帕金森病的重要原因。   以下哪项如果为真,最能加强上述论证?   A.人类很快就能人工合成细胞色素P405,并把它用于治疗因体内不能生成这种细胞色素而导致的疾病。   B.那些体内无法生成细胞色素P405的人,往往同时无法生成其它多种人体有用物质。   C.细胞色素P405除了能使大脑免受有毒化学物质侵害之外,对大脑没有其它影响。   D.多巴胺是一种在大脑中自然生成的化学物质,当对帕金森氏病患者使用多巴胺进行治疗时,他们的症状大都明显缓解。   E.许多帕金森病患者具有在体内自然产生细胞色素P405的能力。   [解题分析]正确答案:C。   题干的结论是:有害化学物质很可能是造成帕金森病的重要原因。要使这个结论成立,有一个条件必须满足,这就是细胞色素P405除了能使大脑免受有毒化学物质侵害之外,对大脑没有其他影响。否则,如果细胞色素P405除了能使大脑免受有毒化学物质侵害之外,对大脑还有其他影响,那么,很可能正是此种影响,而不是抗有毒化学物质侵害的功能,与有效抵制帕金林氏病有关,这样,就缺乏理由认为有害化学物质是造成帕金森病的原因。C项断定的正是上述条件,因此,如果C项为真,最能加强题干的论证。   -"本公司自1980的以来生产的轿车,至今仍有一半在公路上奔驰:其它公司自1980年以来生产的轿车,目前至多有三分之一没有被淘汰。"该公司希望以此广告向消费者显示,该汽车公司生产的轿车的耐用性能极佳。下列哪项如果为真,能够最有效地支持上述广告的观点?   A.扣除通货膨胀的因素,该公司目前生产的新车的价格只比1980年生产的稍高一点。   B.自1980年以来,其他公司轿车的年产量有显著增长。   C.该公司轿车的车主,经常都把车保养得很好。   D.自1980年以来,该公司在生产轿车上的改进远远小于其它公司对轿车的改进。   E.自1980年以来,该公司每年生产的轿车数量没有显著增长。 [解题分析]正确答案:E。 E项断定,自1980年以来,该公司每年生产的轿车数量没有显著增长。这是上述广告有说服力的一个重要条件,事实上,如果该广告发布的时间是2000年,并且20年来该公司每年的车产量基本持平,那么,根据该公司自1980年以来生产的轿车仍有一半在奔驰,可计算出该公司的轿车的平均寿命是10年;另一方面,如果年产量有显著增长,那么年增量越大,被淘汰的车中近年产的车比例越大,因而说明车的平均寿命越短。因此,如果E项为真,能有效地支持上述广告的观点。   同理,B项能说明其他公司生产的轿车质量差,但不能直接说明该公司的质量好。因此,B项对题干广告的支持不如E项。   (四)直接支持推论   由于支持答案最终要对段落推理尤其是结论起作用,因此找到推理结论的核心关键词是最为重要的,特别是在问题目的中已清晰地告诉我们所要支持的内容的时候,我们可以用要支持的内容的核心关键词去定位选项。即解答支持题首先要抓住要支持的结论,然后用支持的内容的核心关键词去定位选项,进而进行判断。一些支持题型的正确答案就是直接支持结论的选项。   -玫瑰城需要100万美元来修理所有的道路。在一年内完成这样的修理之后,估计玫瑰城每年将因此避免支付大约300万美元的赔偿金,这笔赔偿金历年来一直作为给因道路长年失修而损坏的汽车的合理费。   以下哪项如果为真,最能支持题干的估计?   A.与玫瑰城邻近的其它城市,同样也要为它们年久失修的道路赔偿车辆修理费。   B.该地的道路修理好之后,在近几年内不会因道路原因对行驶车辆造成损坏。   C.为了修路,该地要征税。   D.恶劣天气以道路造成的损害在不同年份之间差别很大。   E.道路的损坏主要是由卡车造成的,但是其车主同样为劣质路面造成的车辆损坏进行索赔。   [解题分析]正确答案:B。   如果B项为真,显然能有力地支持题干的估计;完成道路修理之后,玫瑰城每年将因此避免赔付因道路失修损坏汽车的大笔赔偿金。   -提高教师应聘标准并不是引起目前中小学师资短缺的主要原因。引起中小学师资短缺的主要原因,是近年来中小学教学条件的改进缓慢,以及教师的工资的增长未能与其他行业同步。   以下哪项,如果为真,最能加强上述断定?   A.虽然还有别的原因,但收入低是许多教师离开教育岗位的理由。   B.许多教师把应聘标准的提高视为师资短缺的理由。   C.有些能胜任教师的人,把应聘标准的提高作为自已不愿执教的理由。   D.许多在岗但不胜任的教师,把低工资作为自已不努力进取的理由。   E.决策部门强调提高应聘标准是师资短缺的主要原因,以此作为不给教师加工资的理由。   [解题分析]正确答案是A。   题干中断定,中小学师资短缺的主要原因之一,是教师的工资增长滞后。A项加强了这一断定。其余各项均不能加强题干的断定。 第四章 逻辑解题套路精析(三) http://www.sina.com.cn 2003/10/27 14:23 新浪教育   三、削弱   削弱题型是MBA逻辑考试的一个重点,归结为此类题型的考题是数量是最大的。考试难度增加的主要的一个方面就是削弱题型增加,由于这类题型对考生来说有一定难度,看来这是今后的一个方向,要引起考生足够的重视。   削弱题型的解题思路与支持题型的解题思路大致一样,只不过是其答案对段落推理的作用刚好相反。只要将某选项放入前提与结论之间,使段落推理成立或结论正确的可能性降低,这个选项就是正确答案,所以削弱答案既可以是段落推理不成立的必要条件,也可以是充分条件,还可以是既非充分又非必要条件。   削弱的主要方式有:前提与结论没有联系、推论不可行或没有意义、直接削弱等   削弱题型的解题关键是首先应明确原文的推理关系,即什么是前提,什么是结论;在此基础上,寻找削弱的基本方向是针对前提、结论还是论证本身。下面对如何削弱,做一个详细分析。首先要明确,要使一个结论为真,必须满足两个条件:①前提真实,②推理或论证形式有效。于是要反驳或削弱某个结论,通常有这样几条途径:削弱论题(推理的结论)、削弱论据(推理的前提)和削弱论证方式(推理形式)。   如果是削弱结论,那么首先要搞清题干中的结论是什么;如果是反对什么观点,特别要注意的是问题问的是反对的是谁的观点,什么观点;如果对推理提出质疑,那么就要搞清题干的推理结构和前提条件是什么,一个有效的推理必须前提成立,推理形式正确,才能得出正确的结论。这类题目要求在选项中确定哪一项为真,能构成对题干中论证的一个反驳,从而也就削弱了该论证的结论。另外,在削弱结论型考题中,有时虽然要确立的选项不直接构成对论题、论据或推理形式的反驳,但作为前提加入到题干的原前提中去以后,会减低证据支持度,减低结论的可靠性,从而削弱题干的论证。   如果是削弱论证,那么一定要搞清其实是要削弱什么?这就要求我们要对题干部分的论证进行尽可能的简化,抓住中间最主要的推理关系。解题思路是寻找一种弱化的方式,使其既可以是肯定选项中与题干的结论不相容的选项,也可以从选项中找到一个使题干的论证不能成立的条件。要构成对题干中的推理的一个反驳,归谬法是一种有效方法,具体就是举出另一个推理,它有同样的形式并且有真实的前提,却得出了假的结论。   (一)前提与结论之间没有联系或有差异   如果前提与结论之间没有联系或有差异,那么,就从这个前提不能必然得出结论,也就很好地起到了削弱作用。   -过去,大多数航空公司都尽量减轻飞机的重量,从而达到节省燃油的目的。那时最安全的飞机座椅是非常重的,因此只安装很少的这类座椅。今年,最安全的座椅卖得最好。这非常明显地证明,现在的航空公司在安全和省油这两方面更倾向重视安全了。   以下哪项,如果为真,能够最有力地削弱上述结论?   A.去年销售量最大的飞机座椅并不是最安全的座椅。   B.所有航空公司总是宣称他们比其他公司更加重视安全。   C.与安全座椅销售不好的那些年比,今年的油价有所提高。   D.由于原材料成本提高,今年的座椅价格比以往都贵。   E.由于技术创新,今年最安全的座椅反而比一般的座椅重量轻。   [解题分析]正确答案:E。   题干的 逻辑主线是“航空公司购买了更多安全座椅→航空公司在安全和省油这两方面更倾向重视安全。”,要削弱这个推理,就是要说明这两者之间是没有联系的。 本题的答案是明显的。题干的论证必须基于一个假设,即今年出售的最安全的座椅,仍然如同过去的那样,由于比一般座椅较重而导致较多的耗油量。否则,就没有理由因为今年最安全的座椅卖得最好,而得出结论,现在的航空公司在安全和省油这两方面更倾向重视安全。E项断定这一假设不能成立,因此,有力地削弱了题干的结论。其余各项均没有削弱,并且事实上支持了题干的论证。   -在美国,实行死刑的州,其犯罪率要比不实行死刑的州低。因此,死刑能够减少犯罪。   以下哪项如果为真,最可能质疑上述推断?   A.犯罪的少年,较之守法的少年更多出自无父亲的家庭。因此,失去了父亲能够引发少年犯罪。   B.美国的法律规定了在犯罪地起诉并按其法律裁决,许多罪犯因此经常流窜犯罪。   C.在最近几年,美国民间呼吁废除死刑的力量在不断减弱,一些政治人物也已经不再像过去那样在竞选中承诺废除死刑了。   D.经过长期的跟踪研究发现,监禁在某种程度上成为酝酿进一步犯罪的温室。   E.调查结果表明:犯罪分子在犯罪时多数都曾经想过自已的行为可能会受到死刑或常年监禁的惩罚。   [解题分析]正确答案:B。   题干的逻辑主线是“实行死刑的州犯罪率低→死刑能够减少犯罪”,要削弱这个推理,就是要说明这两者之间是没有联系或有差异的。   如果B项真,则可以认为,许多罪犯,为了躲避死刑的风险,宁愿采取流窜作案的方式,选择不实行死刑的州作案。这样,虽然实行死刑的州犯罪率因此下降,但全美国的犯罪率并没有下降。所以不能由此得出死刑能够减少犯罪的结论。其余各项均不能质疑题干的推断。   -长天汽车制造公司的研究人员发现,轿车的减震系统越“硬”,驾驶人员越是在驾驶中感到刺激。因此,他们建议长天汽车制造公司把所有的新产品的减震系统都设计得更“硬”一些,以提高产品的销量。   下面哪一项如果为真,最能削弱该研究人员的建议?   A.长天公司原来生产的轿车的减震系统都比较“软”。   B.驾驶汽车的刺激性越大,车就容易开得越快,越容易出交通事故。   C.大多数人买车是为了便利和舒适,而“硬”的减震系统让人颠得实在难受。   D.目前“硬”减震系统逐步流行起来,尤其是在青年开车族中。   E.买车的人中有些年长者不是为了追求驾驶中的刺激。   [解题分析]正确答案:C。   题干的逻辑主线是“轿车越硬越刺激→越硬越好销。”,要削弱这个推理,就是要说明这两者之间是没有联系的,即大多数人买车并不喜欢“硬”。   具体说,题干中的建议是基于一个假设之上的,即多数轿车买主喜欢在驾驶中感到刺激。C项和E项都对这一假设构成质疑,但E项仅涉及年长者,力度不大,而C项断定大多数人买车是为了便利的舒适,而并不喜欢在颠簸中的刺激。这就使得题干中的建议难以成立。   B项断定的是驾驶汽车具有刺激性的危害。即使这种危害性确实存在,只要多数汽车买主喜欢刺激,题干的建议仍然成立,因为此项建议的直接目的是提高销量。因此,即使B项能对题干的建议有所削弱,其力度也不大。其余项不能削弱题干。   -高脂肪、高糖含量的食物有害人的健康。因此,既然越来越多的国家明令禁止未成年人吸烟和喝含酒精的饮料,那么,为什么不能用同样的方法对待那些有害健康的食品呢?   应该明令禁止18岁以下的人食用高脂肪、高糖食品。   以下哪一项如果为真,则最能削弱上述建议?   A.许多国家已经把未成年人的标准定为16岁以下。   B.烟、酒对人体的危害比高脂肪、高糖食物的危害要大。   C.并非所有的国家都禁止未成年人吸烟喝酒。   D.禁止有害健康食品的生产,要比禁止有害健康食品的食用更有效。   E.高脂肪、高糖食品主要危害中年人的健康。   [解题分析]正确答案:E。   题干的逻辑主线是“高脂肪高糖含量的食物有害人的健康→未成年人应禁用。”,要削弱这个推理,就是要说明这两者之间是没有联系的,即“高脂肪、高糖食品主要危害中年人的健康”,对未成年人的危害并不大,这就削弱了题干的建议。   (二)推论不可行或没有意义   削弱的另一种重要方式是指出推论不可行或没有意义,或直接反对原因,即直接说明原文推理的前提不正确,就达到推翻结论的目的。   -因偷盗、抢劫或流氓罪入狱的刑满释放人员的重新犯罪率,要远远高于因索贿受贿等职务犯罪入狱的刑满释放人员。这说明,在狱中对上述前一类罪犯教育改造的效果,远不如对后一类罪犯。   以下哪项如果为真,最能削弱上述论证?   A.与其他类型的罪犯相比,职务犯罪者往往有较高的文化水平。   B.对贪污、受贿的刑事打击,并没能有效地扼制腐败,有些地方的腐败反而愈演愈烈。   C.刑满释放人员很难再得到官职。   D.职务犯罪的罪犯在整个服刑犯中只占很小的比例。   E.统计显示,职务犯罪者很少有前科。   [解题分析]正确答案:C。   索贿受贿等职务犯罪的条件是具有一定的职务和权力。C项指出了这样一个事实,即刑满释放人员很难再得到官职,这说明职务犯罪的刑满释放人员,和因偷盗、抢劫或流氓罪入狱的型满释放人员相比,较难具备重新犯罪的条件,因此,不能根据偷盗、抢劫或流氓罪入狱的刑满释放人员的重新犯罪率,高于职务犯罪的刑满释放人员,而得出结论,在狱中对上述前一类罪犯教育改造的效果,远不如对后一类罪犯。这就有力地削弱了题干的论证。   其余各项均不能削弱题干。   -有些外科手术需要一种特殊类型的线带,使外科伤口缝合达到十天,这是外科伤口需要线带的最长时间。D型带是这种线带的一个新品种。D型带的销售人员声称D型带将会提高治疗功效,因为D型带的粘附时间是目前使用的线带的两倍长。 以下哪项如是成立,最能说明D型带销售人员所做声明中的漏洞: A.大多数外科伤口使命大约需要十天。   B.大多数外科线带是从医院而不是从药店得到的。   C.目前使用的线带的粘性足够使伤口缝合十天。   D.现在还不清楚究竟是D型带还是目前使用的线带更有利于皮肤的愈合。   E. D型带线带对已经预先涂上一层药物的皮肤的粘性只有目前使用的线带的一半好。   [解题分析]正确答案:C。   D型带的销售人员声称D型带将会提高治疗功效,其根据是D型带的粘附时间是目前使用的线带的两倍长。要使这一声称有说服力,必须说明目前使用的线带的粘性不足够长。C项断定:目前使用的线带的粘性足够使伤口缝合十天,而由题干,使外科伤口缝合达到十天,是外科伤口需要线带的最长时间。因此,如果C项为真,最能说明D型带销售人员所做声明中的存在漏洞。   -毫无疑问,未成年人吸烟应该加以禁止。但是,我们不能为了防止给未成年人吸烟以可乘之机,就明令禁止自动售烟机的使用。马路上不是到处有避孕套自动销售机吗?为什么不担心有人从中买了避孕套去嫖娼呢?以下哪项如果为真,最能削弱题干的论证?   A.嫖娼是触犯法律的,但未成年人吸烟并不触犯法律。   B.公众场合是否适合置放避孕套自动销售机,一直是一个有争议的问题。   C.人工售烟营业点明令禁止向未成年人售烟。   D.在司法部门的严厉打击下,卖淫嫖娼等社会丑恶现象逐年减少。   E.据统计,近年来未成年人吸烟者的比例有所上升。   [解题分析]正确答案:C。   如果C项为真,说明题干进行类比的两类现象中,存在一个实质性的区别,即自动售烟机是未成年人取得香烟的几乎惟一的渠道,而避孕套自动销售机对于嫖娼者来说,是可有可无的。这就有力地削弱了题干的论证。-北方航空公司实行对教师机票六五折优惠,这实际上是吸引乘客的一种经营策略,该航空公司并没有实际让利,因为当某天某航班的满员率超过90%时,就停售当天优惠价机票,而即使在高峰期,航班的满员率也很少超过90%的。有座位空着,何不以优惠价促销它呢。   以下哪项如果是真的,将最有力地削弱上述论证?   A.绝大多数教师乘客并不是因为票价优惠才选择北方航空公司的航班的。   B.该航空公司实行施优惠价的7月份的营业额比未实行优惠价的2月份增加了30%。   C.实施教师优惠票价是表示对教师职业的一种尊重,不应从功利角度对此进行评价。   D.该航空公司在实施教师优惠价的同时,实施季节性调价。   E.该航空公司各航班全年的平均满员率是50%。   [解题分析]正确答案:A。   因为“绝大多数教师乘客并不是因为票价优惠才选择北方航空公司的航班的”,那么北方航空公司实行对教师机票六五折优惠,实际上起不到吸引乘客的作用,反而降低了公司的收入,即此种做法不可行。选项C、D是无关的评论,选项B、E强化了题干的论证,不应选。   -长盛公司的管理者发现:和同行业其它企业相比,该公司产品的总成本远远高于其它企业,因而在市场上只能以偏高的价格出售,导致竞争力较弱。通过研究,公司决定降低工人工资,使之和同行业企业差不多。以下哪项,如果为真,将使公司的决定见效不大?   A.长盛公司的产品质量和其它公司的相比,相差无几。   B.长盛公司的销售费用比其它公司大。   C.长盛公司员工工资总额只占产品成本的一小部分。   D.长盛公司的设备比较落后。   E.长盛公司交货速度不是特别快。   [解题分析]正确答案:C。   你想采取的措施其实根本不可行,这是这类“削弱型”题目的解题思路。选项C讲“长盛公司员工工资总额只占产品成本的一小部分”,这就使得你想通过降低工人工资来大幅度减少公司的总成本的梦想告吹。其它几个选项都与该措施没有太大关系。   (三)存在别的因素影响推论   如果题干是以一个事实、研究、发现或一系列数据为前提推出一个解释上述事实或数据的结论,要削弱这个结论,就可以通过指出由其他可能来解释原文事实,即存在别的因素影响推论。   -被疟原虫寄生的红血球在人体内的存在时间不会超过120天。因为疟原虫不可能从一个它所寄生衰亡的红血球进入一个新生的红血球,因此,如果一个疟疾患者在进入了一个绝对不会再被疟蚊叮咬的地方120天后仍然周期性高烧不退,那么,这种高烧不会是由疟原虫引起的。   以下哪项,如果为真,最能削弱上述结论?   A.由疟原虫引起的高烧和由感冒病毒引起的高烧有时不容易区别。   B.携带疟原虫的疟蚊和普通的蚊子很难区别。   C.引起周期性高烧的疟原虫有时会进入人的脾脏细胞,这种细胞在人体内的存在时间要长于红血球。   D.除了周期性的高烧只有到疟疾治愈后才会消失外,疟疾的其它某些症状会随着药物治疗而缓解乃至消失,但在120天内仍会再次出现。   E.疟原虫只有在疟蚊体内和人的细胞内才能生存与繁殖。   [解题分析]正确答案:C。   如果C项为真,则能说明:如果一个疟疾患者在进入了一个绝对不会再被疟蚊叮咬的地方120天后仍然周期性高烧不退,那么,这种高烧仍然可能是由进入人的脾脏细胞的疟原虫引起的,这就有力地削弱了题干的结论。   其余各项均不能削弱题干。   -最近10年,地震、火山爆发和异常天气对人类造成的灾害比数十年前明显增多,这说明,地球正变得对人类愈来愈充满敌意和危险。这是人类在追求经济高速发展中因破坏生态环境而付出的代价。   以下哪项如果为真,最能削弱上述论证? 经济发展使人类有可能运用高科技手段来减轻自然灾害的危害。 B.经济发展并不必然导致全球生态环境的恶化。   C. W国和H国是两个毗邻的小国,W国经济发达,H国经济落后,地震、火山爆发和异常天气所造成的灾害,在H国显然比W国严重。   D.自然灾害对人类造成的危害,远低于战争、恐怖主义等人为灾害。   E.全球经济发展的不平衡所造成的人口膨胀和相对贫困,使得越来越多的人不得不居住在生态环境恶劣甚至危险的地区。   [解题分析]正确答案:E。   题干的结论是:地球正变得对人类愈来愈充满敌意和危险,这是人类在追求经济高速发展中因破坏生态环境而付出的代价;得出这一结论的根据是:最近10年,地震、火山爆发和异常天气对人类造成的灾害比数十年前明显增多。   如果E项为真,则有助于说明:最近10年,地震、火山爆发和异常天气对人类造成的灾害比数十年前明显增多的原因,不在于生态环境本身的恶化,而在于越来越多的人不得不居住在生态环境恶劣甚至危险的地区。根据E项的断定,这自然也是人类为追求经济高速发展而付出的代价,但却不是题干所断定的因破坏生态环境而付出的代价。这就有力地削弱了题干的论证。   其余各项均不能削弱题干的论证。其中,选项C所提及的W国和H国是两个毗邻的小国,而地震、火山爆发和异常天气所涉及的是大生态环境,因此,对二者的经济发展和受灾状况进行比较,对于揭示经济发展和生态环境的关系几乎没有意义。   -近10年来,移居清河界森林周边地区生活的居民越来越多。环保组织的调查统计表明,清河界森林中的百灵鸟的数量近十年呈明显下降的趋势。但是恐怕不能把这归咎于森林周边地区居民的增多,因为森林的面积并没有因为周边居民人口的增多而减少。   以下哪项如果为真,最能削弱题干的论证?   A.警方每年都接到报案,来自全国各地的不法分子无视禁令,深入清河界森林捕猎   B.清河界森林的面积虽然没减少,但主要由于几个大木材集团公司的滥砍滥伐,森林中树林的数量锐减。   C.清河界森林周边居民丢弃的生活垃圾吸引了越来越多的鸟鹃,这是一种专门觅食百灵鸟卵的鸟类。   D.清河界森林周边的居民大都经营农业,只有少数经营商业。   E.清河界森林中除百灵鸟的数量最近十年来呈明显下降趋势外,其余的野生动物生长态势良好。   [解题分析]正确答案:C。   本题的逻辑主线是“森林的面积并没有减少→百灵鸟的数量下降不能归咎于居民的增多”,要削弱论证就是要说明存在别的因素影响推论。   如果C项的断定为真,则说明清河界森林周边居民的增多,造成了丢弃的生活垃圾的增多;丢弃垃圾的增多,造成了森林中乌鹃的增多;森林中乌鹃的增多,造成了对百灵鸟繁衍的破坏,因而造成了清河界森林中百灵鸟数量的减少。因此,虽然森林的面积没有减少,但清河界周边居民的增多,确实是百灵鸟减少的一个原因。这就有力地削弱了题干的论证。其余各项均不能削弱题干的论证。   -自1940年以来,全世界的离婚率不断上升。因此,目前世界上的单亲儿童,即只与生身父母中的某一位一起生活的儿童,在整个儿童中所占的比例,一定高于1940年。   以下哪项关于世界范围内相关情况的断定,如果为真,最能对上述推断提出质疑?   A. 1940年以来,特别是70年代以来,相对和平的环境的医疗技术的发展,使中青年已婚男女的死亡率极大地降低。   B. 1980年以来,离婚男女中的再婚率逐年提高,但其中的复婚率却极低。   C.目前全世界儿童的总数,是1940年的两倍以上。   D. 1970年以来,初婚夫妇的平均年龄在逐年上升。   E.目前每对夫妇所生子女的平均数,要低于1940年。   [解题分析]正确答案:A。   本题的逻辑主线是“离婚率不断上升→单亲儿童的比例上升”,要削弱论证就是要说明存在别的因素影响推论。   如果A项的断定为真,则1940年的离婚率虽然低于目前,但中青年已婚男女的死亡率却大大高于目前,也就是说,在1940年,世界上与生身父母中离异的一位一起生活的单亲儿童的比例一定低于目前,但与生身父母中丧偶的一位一起生活的单亲儿童的比例一定高于目前。这就对题干的推断提出了严重的质疑。其余各项均不能构成质疑。   -K国的公司能够在V国销售半导体,并且,售价比V国公司的生产成本低。为了帮助V国的那些公司,V国的立法机构制定了一项计划,规定K国公司生产的半导体在V国的最低售价必须比V国公司的平均生产成本高百分之十。以下哪项如果为真,将最严重地影响该项计划的成功?   A.预计明年K国的通货膨胀率超过百分之十。   B.现在K国的半导体不仅仅销往V国。   C.一些销售半导体的V国公司宣布,它们打算降低半导体的售价。   D. K国政府也制定半导体在本国的最低售价。   E.越来越多的非K国的公司去V国销售半导体,并且售价比K国的产品低。   [解题分析]正确答案:E。   本题的逻辑主线是“V国规定了K国公司在本国销售半导的最低价格→V国保护了本国公司”,要削弱论证就是要说明存在别的因素影响推论。即如果非K国的公司在V国销售半导体的价格比K国的定价低,也可以低于V国公司的生产成本。因而,V国立法机构的规定不一定能达到帮助V国的半导体公司的目的。   选A不妥。影响外贸的主要是汇率的变化而不是某国的通货膨胀率,而且A中没有提到V国的通货膨胀率如何变化。B和C若为真,会对V国的半导体公司产品销售产生有利的影响。D与V国的半导体市场关系不大。   -一段时间以来,国产洗发液在国内市场的占有率逐渐减小。研究发现,国外公司的产品广告比国内的广告更吸引人。因此,国产洗发液生产商需要加大广告投入,以增加市场占有率。   以下哪项如果为真,将严重地弱化上述的论证?   A.一些国外洗发液的广告是由国内广告公司制作并由国内媒体传播的。   B.广告只能引起人们对某种商品的注意,质量才能使人们产生对商品的喜爱。   C.国产洗发液生产商的广告费现在只有国外厂商的一半。   D.尽管国外洗发液销售额增加,国产洗发液销售额同样在增加。 E.准备购买新的洗发液的人喜欢从广告中发现合意的品牌。 [解题分析]正确答案:B。   本题的逻辑主线是“国外公司的产品市场占有率大并且其广告比国内的广告更吸引人→国产产商加大广告投入就能增加市场占有率。”,要削弱论证就是要说明存在别的因素影响推论。如果“广告只能引起人们对某种商品的注意,质量才能使人们产生对商品的喜爱”,要增加市场占有率的办法应该把重点放在提高质量上。   选项A的意思是国内广告公司和国内传播媒体也可以做高质量的广告,但原因可能与广告费的高低有关,所以不会弱化题干的论证。C和E对题干有增强作用。D对题干有削弱作用,但没有对国外和国产洗发液的销售额的增加量进行比较,难以得出明确结论。   -越来越多的有说服力的统计数据表明,具有某种性格特征的人易患高血压,而另一种性格特征的人易患心脏病,如此等等。因此,随着对性格特征的进一步分类研究,通过主动修正行为和调整性格特征以达到防治疾病的可能性将大大提高。   以下哪项,最能反驳上述观点?   A.一个人可能会患有与各种不同性格特征均有关系的多种疾病。   B.某种性格与其相关的疾病可能由相同的生理因素导致。   C.某一种性格特征与某一种疾病的联系可能只是数据上的巧合,并不具有一般性意义。   D.人们往往是在病情已难以扭转的情况下,才愿意修正自己的行为,但已为时太晚。   E.用心理手段医治与性格特征相关的疾病的研究,导致心理疗法遭到淘汰。   [解题分析]正确答案:B。   题干中根据统计发现:甲现象(某性格特征)总伴随着乙现象(某疾病)出现,因此推断,甲是乙的原因。   由B项,甲和乙可能是丙(某种生理因素)的共同结果。这就有力地反驳了题干中甲和乙存在因果关系(可通过调整性格来治病)的观点   (四)直接削弱推论   首先明确题干的结论或观点是什么,如果我们找到一个选项与其刚好相违背即为正确答案,这种削弱方式就是“直接削弱推论”。   -美国的一个动物保护组织试图改变蝙蝠在人们心目中一直存在的恐怖形象。这个组织认为,蝙蝠之所以让人觉得可怕和遭到捕杀,仅仅是因为这些羞怯的动物在夜间表现出特别的活跃。   以下哪项如果为真,将对上述动物保护组织的观点构成最严重的质疑?   A.蝙蝠之所以能在夜间特别活跃,是由于它们具有在夜间感知各种射线和声波的特殊能力。   B.蝙蝠是夜间飞行昆虫的主要捕食者。在这样的夜间飞行昆虫中,有很多是危害人类健康的。   C.蝙蝠在中国及其它许多国家同样被认为是一种恐怖的飞禽。   D.美国人熟知的浣熊和中国人熟知的食蚊雀,都是些在夜间特别活跃的羞怯动物,但在众的印象中一般并没有恐怖的印象。   E.许多视觉艺术品,特别是动画片丑化了蝙蝠的形象。   [解题分析]正确答案:D。   题干中所陈述的动物保护组织的观点是:蝙蝠之所以让人觉得可怕并遭到捕杀,仅仅是因为这些羞怯的动物在夜间表现出特别的活跃。   D项如果为真,则对上述观点提出了一个有力的反例:浣熊和食蚊雀,都是在夜间特别活跃的羞怯动物,但在人们的印象中一般并没有恐怖的印象,因而是题干的观点的有力质疑。   -某些种类的海豚利用回声定位来发现猎物:它们发射出滴答的声音,然后接收水域中远处物体反射的回音。海洋生物学家推测这些滴答声可能有另一个作用:海豚用异常高频的滴答声使猎物的感官超负荷,从而击晕近距离的猎物。   以下哪项如果为真,最能对上述推测构成质疑?   A.海豚用回声定位不仅能发现远距离的猎物,而且能发现中距离的猎物。   B.作为一种发现猎物的讯号,海豚发出的滴答声,是它的猎物的感官所不能感知的,只有海豚能够感知从而定位。   C.海豚发出的高频记号即使能击晕它们的猎物,这种效果也是很短暂的。   D.蝙蝠发出的声波不仅能使它发现猎物,而且这种声波能对猎物形成特殊刺激,从而有助于蝙蝠捕获它的猎物。   E.海豚想捕获的猎物离自己越远,它发出的滴答声就越高。   [解题分析]正确答案:B。   如果B项的断定为真,则由于海豚发出的滴答声,不能使它的猎物感知,更谈不上使其感官超负荷从而被击晕,因此,海洋生物学家的推测显然不能成立。其余各项均不能构成质疑。   -澳大利亚是个地广人稀的国家,不仅劳动力价格昂贵,而且很难雇到工人,许多牧场主均为此发愁。有个叫德尔的牧场主采用了一种办法,他用电网把自己的牧场圈起来,既安全可靠,又不需要多少牧牛工人。但是反对者认为这样会造成大量的电力浪费,对牧场主来说增加了开支,对国家的资源也不够节约。   以下哪项,如果为真,能够削弱批评者对德尔的指责?   A.电网在通电10天后就不再耗电,牛群因为有了惩罚性的经验,不会再靠近和触碰电网。   B.节省人力资源对于国家来说也是一笔很大的财富。   C.使用电网对于牛群来说是暴力式的放牧,不符合保护动物的基本理念。   D.德尔的这种做法,既可以防止牛走失,也可以防范居心不良的人偷牛。   E.德尔的这种做法思路新颖,可以考虑用在别的领域以节省宝贵的人力资源。   [解题分析]正确答案:A。   如果A项的断定为真,则题干中反对者所指责的电力浪费,即使存在,也至多只会持续10天,这就有力地削弱了题干中对德尔的指责。B项对题干中的指责有所削弱,但未能说明节省的人力资源是否足以补偿电力浪费,因此,力度不大。其余各项均不能削弱题干中的指责。   -在目前财政拮据的情况下,在本市增加警力的动议不可取。在计算增加警力所需的经费开支时,光考虑到支付新增警员的工资是不够的,同时还要考虑到支付法庭和监狱新雇员的工资。由于警力的增加带来的逮捕、宣判和监管任务的增加,势必需要相关部门同时增员。以下哪项,如果为真,将最有力地削弱上述论证?   A.增加警力所需的费用,将由中央和地方财政共同负担。   B.目前的财政状况,决不至于拮据到连维护社会治安的费用都难以支付的地步。 C.湖州市与本市毗邻,去年警力增加19%,逮捕个案增加40%,判决个案增加13%。 D.并非所有侦察都导致逮捕,并非所有逮捕都导致宣判,并非所有宣判都导致监禁。   E.当警力增加到与市民的数量达到一个恰当的比例时,将减少犯罪。   [解题分析]正确答案:E。   E项断定,当警力增加到与市民的数量达到一个恰当的比例时,将减少犯罪,这就意味着,在这样的条件下,相应的逮捕、宣判和监管任务不但没有增加,反而减少,因此,并不需要相关部门同时增员。这就有力地削弱了题干的论证。其余各项对题干的论证都有所削弱,但力度都不如E项。   -在历史上,从来都是科学技术新发明的浪潮导致了新产业的诞生和兴旺,在此基础上逐步形成区域性直到世界性的经济繁荣,从汽车、飞机产业到化工、制药、电子等领域,情况都是如此。因此,目前产业界普遍增加在科学研究和开发上的投入,必将有力地促进经济繁荣。   以下哪项,如果为真,最能削弱上面的推论?   A.在目前的资金水平上,公司的研究开发部门申请专利的数量比起十年前来要少得多。   B.大部分产业的研究开发部门关心的只是对现有产品进行有利于经销的低成本改进,而不是开发有远大前途的高成本新技术。   C.历史上,只有一些新的主干行业是直接依赖公司研究开发部门获得技术突破的。   D.公司在科学研究和开发上的投入与公司每年新的发明专利的数量直接相关。   E.政府对科学研究和开发的投入将在未来五年中大大缩减。   [解题分析]正确答案:B。   选项B对推论的削弱在于,即使你普遍增加在科学研究和开发上的投入,这些投入也全投到有利于经销的低成本改进上,而不是开发有远大前途的高成本新技术,最终也无法达成新产业的诞生与兴旺的初衷。   -春江市师范大学的同学们普遍抱怨各个食堂的伙食太差。然而唯独一年前反映最差的风味食堂,这一次同学抱怨的人数比较少。学校后勤部门号召其它各个食堂向风味食堂学习,共同改善学校学生关心的伙食问题。下列哪项如果为真,则表明学校后勤部门的这个决定是错误的?   A.各个食堂的问题不同,不能一刀切,要因地制宜,采取不同的措施。   B.风味食堂的进步是与其它各个食堂的支持分不开的。   C.粮食价格一天天上涨,食堂再努力,也“难为无米之炊”啊。   D.因为差,风味食堂就餐的人数比起其它食堂要少得多。   E.风味食堂的花样多,但是价格高,困难同学可吃不起。   [解题分析]正确答案:D。   抱怨的人数比较少,而不是抱怨的比例比较低,这是题目的关键。为什么抱怨的人数比较少呢?不是因为伙食特别好,而是因为太差了,同学们都不去吃了,当然抱怨的绝对人数就少了。   (五)以偏盖全   以偏盖全是不完全归纳推理时容易出现的逻辑错误,如果题干的推理出现了这种逻辑错误,削弱的方式就是拿出理由,说明用这种不完全归纳得出的结论是错误的。其实这类题型的削弱方式是“前提与结论没有联系或有差异”的一种特例。   -为了估计当前人们对管理基本知识掌握的水平,《管理者》杂志为读者开展了一次管理知识有奖答卷活动。答卷评分后发现,60%的参加者对于管理基本知识掌握的水平很高,30%左右的参加者也表现出了一定的水平。《管理者》杂志因此得出结论,目前社会群众对于管理基本知识的掌握还是不错的。   以下哪项,如果为真,则最削弱以上结论?   A.管理基本知识的范围很广,仅凭一次答卷得出结论未免过于草率。   B.管理基本知识的掌握与管理水平的真正提高还有相当的差距。   C.并非所有的《管理者》的读者都参加了此次答卷活动。   D.从定价、发行渠道等方面看,《管理者》的读者主要集中在高等学历知识阶层。   E.可能有几位杂志社的工作人员的亲戚也参加了此次答卷,并获了奖。   [解题分析]正确答案:D。   题干的逻辑主线是“《管理者》杂志的读者管理知识水平高→社会群众对于管理知识的掌握得好。”,要削弱这个推理,就是要说明这两者之间是有差异的,即《管理者》杂志的读者不是社会群众的代表,题干推理犯了以偏概全的错误。   如果D项为真,则由于事实上《管理者》的读者主要是高学历者和实际的经营管理者,因此就不能因为他们在问答中表现出较高的管理知识水平,就得出目前社会群众对于管理基本知识的掌握不错的结论。   选项B与题干结论无关,选项A、C、E对题干结论构成轻度质疑,C、E在质疑抽样数据的可靠性与可信性,但比较而言,D项的质疑最根本。   -今年上半年,即从1月到6月间,全国大约有300万台录像机售出。这个数字仅是去年全部录像机销售量的35%。由此可知,今年的录像机销售量一定会比去年少。   以下哪项如果为真,最能削弱以上的结论?   A.去年的录像机销售量比前年要少。   B.大多数对录像机感兴趣的家庭都已至少备有一台。   C.录像机的销售价格今年比去年便宜。   D.去年销售的录像机中有6成左右是在1月售出的。   E.一般说来,录像机的全年销售量70%以上是在年末两个月中完成的。   [解题分析]正确答案:E。   题干的逻辑主线是“今年上半年只占去年全年销售量的35%→今年的销量一定会比去年少”,要削弱这个推理,就是要说明这两者之间是有差异的,即上半年并不能说明全年,题干推理犯了以偏概全的错误。   E项断定,一般说来,录像机的全年销售量70%以上是在年末两个月中完成的。如果这一断定为真,则虽然今年上半年的销量仅是去年全年的35%,但全年的产量却极可能超过去年。这就严重地削弱了题干的结论。   C项对题干的结论有所削弱,但力度显然不如E项。其余各项都不能削弱题干。 -我国多数软件开发工作者的“版权意识”十分淡漠,不懂得通过版权来保护自己的合法权益。最近对500多位软件开发者的调查表明,在制定开发计划时也同时制定了版权申请计划的仅占20%。 以下哪项如果为真,最能削弱上述结论?   A.制定了版权申请计划并不代表有很强的“版权意识”,是否有“版权意识”要看实践。   B.有许多软件开发者事先没有制定版权申请计划,但在软件完成后申请了版权。   C.有些软件开发者不知道应该到什么地方去申请版权。有些版权受理机构服务态度也不怎么样。   D.版权意识的培养需要有一个好的法制环境。人们既要保护自己的版权,也要尊重他人的版权。   E.在被调查的500名软件开发者以外还有上万名计算机软件开发者,他们的“版权意识”如何,有待进一步调查。   [解题分析]正确答案:B。   许多软件开发者在软件完成后申请了版权,仍然是有版权意识。削弱了题干关于“我国多数软件开发工作者的版权意识十分淡漠”的结论。   选项A对“制定了版权申请计划”的软件开发者也提出了质疑,怀疑他们是否真有“版权意识”,会增强题干的结论。C对题干论点有削弱的意思,但强度远不如B,请注意“有许多”和“有些”的区别。选项D讲的是理想环境,而题干讲的是现实状况。E讲需要再调查,但对题干的论点没有明显的削弱。因为,现在的调查已经是个大样本。另外,进一步的调查,也可能得出进一步加强题干论点的结论。   (六)因果倒置   如果某两类因素A和B紧密相关,题干就指出A是造成B的原因,那么要削弱它,就可以说明B才是造成A的原因,这就是所谓的“因果倒置”。   -一项调查统计显示,肥胖者参加体育锻炼的月平均量,只占正常体重者的不到一半,而肥胖者的食物摄入的月平均量,基本和正常体重者持平。专家由此得出结论,导致肥胖的主要原因是缺乏锻炼,而不是摄入过多的热量。   以下哪项如果为真,将严重削弱上述论证?   A.肥胖者的食物摄入平均量总体上和正常体重者基本持平,但肥胖者中有人是在节食。   B.肥胖者由于体重的负担,比正常体重者较为不乐意参加体育锻炼。   C.某些肥胖者体育锻炼的平均量,要大于正常体重者。   D.体育锻炼通常会刺激食欲,从而增加食物摄入量。   E.通过节食减肥有损健康。   [解题分析]正确答案:B。   如果B项为真,则有助于说明,对于肥胖者来说,是体重的负担导致较少锻炼,而不是相反。这就有力地削弱了题干的论证。   -最近举行的一项调查表明,师大附中的学生对滚轴溜冰的着迷程度远远超过其他任何游戏,同时调查发现经常玩滚轴溜冰的学生的平均学习成绩相对其他学生更好一些。看来,玩滚轴溜冰可以提高学生的学习成绩。以下哪项如果为真,最能削弱上面的推论?   A.师大附中与学生定长订了协议,如果孩子的学习成绩的名次没有排在前二十名,双方共同禁止学生玩滚轴溜冰。   B.玩滚轴溜冰能够锻炼身体,保证学习效率的提高。   C.玩滚轴溜冰的同学受到了学校有效的指导,其中一部分同学才不至于因此荒废学业。   D.玩滚轴溜冰有助于智力开发,从而提高学习成绩。   E.玩滚轴溜冰很难,能够锻炼学生克服困难做好一件事情的毅力,这对学习是有帮助的。   [解题分析]正确答案:A。   典型的因果倒置型题目。选项A揭示了一个额外信息,说明经常玩滚轴溜冰的学生是被筛选过的,是因为成绩好才能玩,而不是因为玩才成绩好。选项B、D、E都是支持题干推论的,排除;选项C虽然有一定削弱作用,但是程度太弱。   -新民住宅小区扩建后,新搬入的住户纷纷向房产承销公司投诉附近机场噪声太大令人难以忍受。然而,老住户们并没有声援说他们同样感到噪声巨大。尽管房产承销公司宣称不会置住户的健康于不顾,但还是决定对投诉不准备采取措施。他们认为机场的噪声并不大,因为老住户并没有投诉。   下列哪项如果为真,则最能表明房产承销公司对投诉不采取措施的做法是错误的?   A.房产承销商们的住宅并不在该小区,所以不能体会噪声的巨大危害。   B.有些老住户自己配备了耳塞来解决这个问题,他们觉得挺有效果的。   C.老住户觉得自己并没有与房产承销商有什么联系,也没有太大的矛盾。   D.老住户认为噪声并不巨大而没有声援投诉,是因为他们的听觉长期受噪音影响已经迟钝失灵。   E.房产承销公司从来没有隐瞒过小区位于飞机场旁边这一事实。   [解题分析]正确答案:D。   此题为因果倒置的题型。不是因为“机场的噪声并不大”,所以“老住户”才“没有投诉”,而恰恰是因为机场的噪音影响老住户的听觉导致迟钝失灵,所以已经感觉不到噪音了。   (七)综合支持与削弱   支持和削弱是密切相关的,两者的解题思路基本一致,只不过是其答案对段落推理的作用刚好相反。而且对某些题目,到底是属于支持还是削弱也很难区分,有的题目貌似支持实际上是削弱,有的题目貌似削弱实际上是支持。比如原文认为A不是导致B的原因,要对其进行削弱,就可以指出A是B的间接原因,即指出A通过导致C而间接的导致了B。实际上也就支持了A是B的原因。   因此,加上提问方式的不同和解题时运用排除法等,支持题型也可转化为削弱题型来考虑,削弱题型也可转化为支持题型来考虑。要明确的是,不论支持还是削弱,题干的选项都必须首先与题干相关,紧扣题干,与题干不相干、不一致的选项都不能加强题干,也不能削弱题干。因此,不管是哪一类的支持或削弱方式,支持或削弱都最终对推理或结论起作用,所以找到结论非常重要,并且选项应与结论有关。   支持题型的解题思路是,要注意寻找与题干一致的选项。而如果是最不能支持型,当然与题干相矛盾或不一致的选项就最不能支持了。同样,对于削弱题型也要分清最能削弱型还是最不能削弱型。如果是最不能削弱型,解题时可采取排除法,应先将能与题干唱反调的选项排除掉,最后剩下的选项不管是与题干不相干还是支持题干的都是最不能削弱的。如果是最能削弱型,则应首先将选项与题干一致的选项排除掉,同时寻找与题干相矛盾或不一致的选项,从中进一步比较削弱的程度。这里再说明几个注意点:   1、最能支持(或削弱)型考题,一般选项中至少有两个能支持(或削弱)的选项,因此,在确定答案时必须比较其支持(或削弱)的程度   2、前面讲的几种支持与削弱的方式只是给考生解题 时提供的一种思路,对某些考题可能用其中的几种思路都说得通,因此,考生不要拘泥于具体每一道逻辑题到底归于哪一类,特别是真正到考场,我们会发现没有时间判断考题属于哪一类,在考试中主要还是凭平时训练积累起来的感觉来迅速解题。 3、要注意提问方式,一定要审题清楚。近来,支持与削弱类考题增加难度的一个重要方面就是提问方式的灵活多变,问题多绕几个弯,比如:   (1)以下哪项如果为真,则最能支持(或削弱)以上反对者的结论?   (2)以下哪项如果为真,则最不能支持(或削弱)***的结论?   (3)支持型和削弱型的变种:除***之外,都支持(或削弱)***   (4)以下都是对题干结论的支持(或削弱),除了   所以,保持头脑的清醒、搞清不同提问方式的不同特点,抓住本质,阅读细心和注意审题是取得逻辑考试高分的一个重要条件。   -有人对某位法官在性别歧视类案件审理中的公正性提出了质疑。这一质疑不能成立,因为有记录表明,该法官审理的这类案件中南海60%的获胜方为女性,这说明该法官并未在性别歧视类案件的审理中有失公正。以下哪项如果为真,能对上述论证构成质疑?   I.在性别歧视案件中,女性原告如果没有确凿的理由和证据,一般不会起诉。   II.一个为人公正的法官在性别歧视案件的审理中保持公正也是件很困难的事情。   III.统计数据表明,如果不是因为遭到性别歧视,女性应该在60%以上的此类案件的诉讼中获胜。   A.只有I。   B.只有II。   C.只有III。   D.只有I和III。   E. I、II和III。   [解题分析]正确答案:D。   复选项Ⅱ不能对题干的论证构成质疑。因为,第一,它不能对题干的论据构成质疑;第二,它不能对题干中从论据到结论的推断构成质疑;第三,它同样不能对题干的结论构成质疑,因为一般地,某人做某件事有难度,不能对某人做成了这件事构成质疑,例如,登上珠穆朗玛峰很困难,这不能对中国人登上了珠穆朗玛峰构成质疑。复选项Ⅰ能构成质疑。复选项Ⅲ显然能构成质疑。   -某保险公司计划推出一项医疗保险,对象是60岁以上经体检无重大疾病的老年人。投保者在有生之年如果患心血管疾病或癌症,则其医疗费用的90%将由保险公司赔付。为了吸引投保者,保险金又不能定得太高。有人估计保险金将不是以支付赔付金,因而会是个赔本生意。尽管如此,保险公司的老总们仍决定推出该项保险。   以下各项断定如果为真,其中,哪项最不可能是老总们做出上述决策的依据?   A.题干中的估计只是一种悲观估计,事实上还存在着乐观的估计。   B.推出这种带有明显折赔风险的险种,有利于树立保险公司的道义形象和信誉,而这更有利于开拓更大的保险市场。   C.随着全民健身的普及,中老年人中癌症和心血管疾病的发病率呈逐年下降的趋势。   D.随着相关科研的深入和医疗技术的提高,对癌症和心血管疾病的检测和医治近年内将会出现突破性的进展。   E.推出上述险种,可以从国际老年人容福利基金组织申请资助。   [解题分析]正确答案:D。   D项所断定的对癌症和心血管疾病的检测和医治近年内将会出现突破性的进展,对上述保险计划既不会产生正面影响,也不会产生负面影响,因此,最不可能是题干中决策的依据。   其余各项都可能是上述决策的依据。   -据S市的卫生检疫部门统计,和去年相比,今年该市肠炎患者的数量有明显的下降。权威人士认为,这是由于该市的饮用水净化工程正式投入了使用。   以下哪项,最不能削弱上述权威人士的结论?   A.和天然饮用水相比,S市经过净化的饮用水中缺少了几种重要的微量元素。   B. S市的饮用水净化工程在五年前动工,于前年正式投入了使用。   C.去年S市对餐饮业特别是卫生条件较差的大排档进行了严格的卫生检查和整顿。   D.由于引进了新的诊断技术,许多以前被诊断为肠炎的病案,今年被确诊为肠溃疡。   E.全国范围的统计数字显示,我国肠炎患者的数量呈逐年明显下降的趋势。   [解题分析]正确答案:A。   题干中权威人士的结论是:S市今年肠炎患者的数量比去年明显下降的原因,是由于该市的饮用水净化工程正式投入了使用。   如果B项的断定为真,则由于S市的饮用水净化工程于前年就投入了使用,因此,这一工程的使用,显然不能成为S市今年肠炎患者的数量比去年明显下降的原因。这削弱了题干的结论。   如果C项的断定为真,则存在这种可能性,S市对餐饮业严格的卫生检查和整顿是在接近去年年底进行的。作为这种检查的结果,今年该市餐饮业的卫生状况比去年有明显改善;这又可能是今年肠炎患者的数量比去年明显下降的主要原因。这削弱了题干的结论。   如果D项的断定为真,则今年肠炎患者的数量比去年明显下降的主要原因,可能是在去年会被诊断为肠炎的病例,今年被确诊为肠溃疡。这削弱了题干的结论。   如果E项的断定为真,则说明可能是某种在全国范围内一般性的原因造成了S市肠炎患者的逐年减少。这削弱了题干的结论。   如果A项为真,则只有满足下述条件,题干的结论才可能被削弱:缺少所提及的微量元素会降低人对肠炎的抵抗力。没有根据认为此条件可满足。因此,A项不能削弱题干的结论。   -一位海关检查员认为,他在特殊工作经历中培养了一种特殊的技能,即能够准确地判定一个人是否在欺骗他。他的根据是,在海关通道执行公务时,短短的几句对话就能使他确定对方是否可疑;而在他认为可疑的人身上,无一例地都查出了违禁物品。   以下哪项如果为真,能削弱上述海关检查员的论证?   Ⅰ.在他认为不可疑而未经检查的入关人员中,有人无意地携带了违禁物品。   Ⅱ.在他认为不可疑而未经检查的入关人员中,有人有意地携带了违禁物品。   Ⅲ.在他认为可疑并查出违禁物品的入关人员中,有人是无意地携带的违禁物品。 只有Ⅰ。 B.只有Ⅱ。   C.只有Ⅲ。   D.只有Ⅱ和Ⅲ。   E.Ⅰ、Ⅱ和Ⅲ。   [解题分析]正确答案:D。   选项Ⅰ不能削弱海关检查员的论证。因为判定一个无意地携带了违禁物品的入关人员为不可疑,不能说明检查员受了欺骗,同样不能说明检查员在判定一个人是否在欺骗他时不够准确。   选项Ⅱ能削弱海关检查员的论证。因为判定一个有意地携带了违禁物品的人关入员为不可疑,说明检查员受了欺骗,因而能说明检查员在判定一个人是否在欺骗他时不够准确。   选项Ⅲ能削弱海关检查员的论证。因为判定有人无意地携带了违禁物品的入关人员为可疑,虽然不能说明检查员受了欺骗,但是能说明检查员在判断一个人是否在欺骗他时不够准确。   -一个已经公认的结论是,北美洲人的祖先来自亚洲。至于亚洲人是如何到达北美的呢,科学家们一直假设,亚洲人是跨越在14000年以前还连结着北美和亚洲,后来沉入海底的陆地进入北美的,在艰难的迁徙途中,他们靠捕猎沿途陆地上的动物为食。最近的新发现导致了一个新的假设,亚洲人是驾船沿着上述陆地的南部海岸,沿途以鱼和海洋生物为食而进入北美的。   以下哪项如果为真,最能使人有理由在两个假设中更相信后者?   A.当北美和亚洲还连在一起的时候,亚洲人主要以捕猎陆地上的动物为生。   B.上述连结北美和亚洲的陆地气候极为寒冷,植物品种和数量都极为稀少,无法维持动物的生存。   C.存在于8000年以前的亚洲和北美文化,显示出极大的类似性。   D.在欧洲,靠海洋生物为人的食物来源的海洋文化,最早发端于10000年以前。   E.在亚洲南部,靠海洋生物为人的食物来源的海洋文化,最早发端于14000年以前。   [解题分析]正确答案:B。   如果B项的断定为真,则题干中的第一个假设就难以成立。因为第一个假设断定,迁涉者是以沿途的动物为食,而据B项可知,这样的动物当时难以存在。A项是支持第一个假充的理由。E项能支持第二个假设,但力度不大。其余各项与问题无关。   -有着悠久历史的肯尼亚国家自然公园以野生动物在其中自由出没有著称。在这个公园中,已经有10多年没有出现灰狼了。最近,公园的董事会决定引进灰狼。董事会认为,灰狼不会对游客造成危害,因为灰狼的习性是避免与人接触的;灰狼也不会对公园中的其他野生动物造成危害,因为公园为灰狼准备了足够的家畜如山羊、兔子等作为食物。   以下各项如果为真,都能加强题干中董事会的论证,除了   A.作为灰狼食物的山羊兔子等,和野生动物一样在公园中自由出没,这增加了公园的自然气息和游客的乐趣。   B.灰狼在进入公园前将经过严格的检疫,事实证明,只有患有狂犬病的灰狼才会主动攻击人。   C.自然公园中,游客通常坐在汽车中游览,不会遭到野兽的直接攻击。   D.麋鹿是一种反应极其敏捷的野生动物。灰狼在公园中对麋鹿可能的捕食将减少其中的不良个体,从总体上有利于麋鹿的优化繁衍。   E.公园有完备的排险设施,能及时地监控并有效地排除人或野生动物遭遇的险情。   [解题分析]正确答案:A。   A项不能加强(事实上削弱了)题干中的论证,因为作为灰狼食物的山羊兔子等和野生动物一起出没,这使得灰狼在捕食食物时,可能会对公园中的其他野生动物造成危害,这就削弱了题干中的论证。其余各项都能加强题干的论证。   -孩子出生后的第一年在托儿所度过,会引发孩子的紧张不安。在我们的研究中,有464名12~13岁的儿童接受了特异情景测试法的测验,该项测验意在测试儿童1岁时的状况与对母亲的依附心理之间的关系。其结果:有41.5%曾在托儿所看护的儿童和25.7%曾在家看护的儿童被认为紧张不安,过于依附母亲。   以下哪项如果为真,最没有可能对上述研究的推断提出质疑?   A.研究中所测验的孩子并不是从托儿所看护和在家看护两种情况下随机选取的。因此,这两组样本儿童的家庭很可能有系统的差异存在。   B.这项研究的主持者被证实曾经在自已的幼儿时期受到过长时间来自托儿所阿姨的冷漠。   C.针对孩子的母亲另一部分研究发现:由于孩子在家里表现出过度的依附心理,父母因此希望将其送入托儿所予以矫正。   D.因为风俗的关系,在464名被测者中,在托儿所看护的大多数为女童,而在家看护的多数为男童。一般地说,女童比男童更易表现为紧张不安和依附母亲。   E.出生后第一年在家看护的孩子多数是由祖父母或外祖父母看护的,并形成浓厚的亲情。   [解题分析]正确答案:E。   题干的结论是:孩子出生后的第一年在托儿所度过,会引发孩子的紧张不安。其根据是:表现出紧张不安和依附母亲的被测度儿童,在1岁时曾由托儿所看护的儿童中所占的比例,要高于1岁时曾在家中看护的儿童。A项能构成质疑。因为如果两组进行比较的儿童本身可能存在系统性的差异,那么,他们是否较易紧张不安,完全可能由此种差异造成,而并非因为1岁时是否由托儿所看护。B项能构成质疑。因为由此可以怀疑,题干中研究者的推断是否带上了研究者本人的个人偏向和主观色彩。C项能构成质疑。因为由此可以得出结论:至少有一部分孩子,不是由于去了托儿所才有了依附心理,恰恰相反,而是表现出了过度的依附心理才被送进托儿所。D项能构成质疑。因为由此可以认为,表现出紧张不安和依附母亲的被测试儿童,在1岁时曾由托儿所看护的儿童中所占的比例较高,是因为该组中女童所占的比例较高,因此,不能认为是托儿所引发了孩子的紧张不安。与上述各项相比,E项显然最不可能构成质疑。   -一项全球范围的调查显示,近10年来:吸烟者的总数基本保持不变;每年只有10%的吸烟者改变自己的品牌,即放弃原有的品牌而改吸其他品牌:烟草制造商用在广告上的支出占其毛收入的10%。   在Z烟草公司的年终董事会上,董事A认为,上述统计表明,烟草业在广告上的收益正好等于其支出,因此,此类广告完全可以不做。董事B认为,由于上述10%的吸烟者所改吸的香烟品牌中几乎不包括本公司的品牌,因此,本公司的广告开支实际上是笔亏损性开支。   以下哪项,构成对董事A的结论的最有力质疑?   A.董事A的结论忽视了:对广告开支的有说服力的计算方法,应该计算其占整个开支的百分比,而不应该计算其占毛收入的百分比。   B.董事A的结论忽视了:近年来各种品牌的香烟的价格有了很大的变动。   C.董事A的结论基于一个错误的假设:每个吸烟者在某个时候只喜欢一种品牌。   D.董事A的结论基于一个错误的假设:每个烟草制造商只生产一种品牌。   E.董事A的结论忽视了:世界烟草业是一个由处于竞争状态的众多经济实体组成的。  [解题分析]正确答案:E。 题干中统计的烟草业的广告收益,是世界各烟草企业收益的合计;同样,这样的广告支出,也是世界各烟草企业支出的合计。因此,虽然从合计上看收支相当,但对单个的烟草企业来说,其在广告上的支出和收益可以表现出很大的差别。因此显然不能因为题干的数据显示烟草业在广告上的收益等于其支出,而得出此类广告完全可以不做的结论。董事A的观点正是忽视了这样一个事实,即世界烟草业是一个由处于竞争状态的众多经济实体组成的。E项指明了这一点,因而构成对董事A的结论的有力质疑。其余各项对董事A的质疑均不得要领。   -本题题干与上题相同。   以下哪项,如果为真,能构成对董事B的结论的质疑?   Ⅰ.如果没有Z公司的烟草广告,许多消费Z公司品牌的吸烟者将改吸其他品牌。   Ⅱ.上述改变品牌的10%的吸烟者所放弃的品牌中,几乎没有Z公司的品牌。   Ⅲ.烟草广告的效果之一,是吸引新吸烟者取代停止吸烟(死亡的吸烟者或戒烟者)而消费自己的品牌。   A.只有Ⅰ。   B.只有Ⅱ。   C.只有Ⅲ。   D.只有Ⅰ和Ⅱ。   E.Ⅰ、Ⅱ和Ⅲ。   [解题分析]正确答案:E。   烟草广告的效果,至少可以体现在三方面:第一,吸引消费其他品牌的吸烟者改吸自己的品牌;第二,说服消费自己品牌的吸烟者继续消费本品牌;第三,吸引新吸烟者消费自己的品牌。Ⅰ、Ⅱ和Ⅲ的断定如果为真,都能说明,不能因为Z公司的烟草广告在上述第一方面的效果不明显,就认为该公司的广告是笔亏损性开支。因此,Ⅰ、Ⅱ和Ⅲ都能构成对董事B的质疑。 第四章 逻辑解题套路精析(四) http://www.sina.com.cn 2003/10/27 14:27 新浪教育   四、评价   评价考题主要考查我们评价论点的能力,由于评价在很多情况下是针对段落推理成立的隐含假设起作用,所以读题时要注意体会段落推理的隐含假设,然后去寻找一个能对段落推理起到正反两方面作用的选项。   当选项为一般疑问句时,对这个问句有两方面的回答——“是”和“否”。若对这个问句回答“是”是对段落推理起到了支持作用,同时对这个问句回答“否”,就对段落推理起到了驳斥作用;若对这个问句回答“是”对段落推理起到了反对作用,同时对这个问句回答“否”,就对段落推理起到了支持作用。于是,这个问题就对段落推理有评价作用。要特别引起注意的是,正确的选项一定是对这个问句的“是”与“否”的回答都起作用,如果仅仅对一方面回答起作用,则不是评价。请看下面这个例子:   -哈丁争论说,人们使用起共同拥有的(即对任何使用者开放的)牧场比使用私人的牧场更不注意。每个放牧者都有过度使用公地的冲动,因为从中获得的利益将归于个人,而由于过度使用土地而引起的土地质量下降的成本由所有使用者分摊。但一项研究比较了2.17亿英亩的公用牧场和4.33亿英亩的私人牧场,表明公用牧场的条件更好。   与哈丁的宣称做比较,评价以上描述的这项研究的意义时,以下哪一个问题的答案将最有用?   (A)有没有一些放牧者,他们的土地属于被研究之列,既使用公用又使用私人土地?   (B)那些自己的土地属于被研究之列的放牧者是否倾向于更愿意使用公地而不使用私人土地来放牧?   (C)在用来放牧之前该研究中的私人土地是否与公地的质量相当?   (D)该研究中的公地使用者是否至少与私人土地的使用者一样有钱?   (E)是否有任何牧群的所有者只在公地不在私人土地上放牧?   [解题分析]正确答案:C。   哈丁的宣称是因为过度使用,公地比私人牧地条件恶化更快。研究表明,公地现在的状况更好,但如果在放牧前,私人土地与公地的质量不相当,公用牧地的条件就远远好于私人牧地,该研究中指出的现象就不会削弱哈丁的宣称。反之,如果在放牧之前私人土地与公地的质量确实相当,那么该研究中指出的现象就能削弱哈丁的宣称。因此,C能起到评价作用。A和E不合适,因为不管某些放牧者使用两种土地还是只使用公地,该研究都可削弱哈丁的宣称;同样,不管放牧者是否更喜欢使用公地,如B所说,该研究都可削弱哈丁的宣称;而D不合适是因为如果公地的使用者更加富有,或更不富有,都不会使研究的说服力减小。   当选项为特殊疑问句或陈述句时,如果对这些选项的回答的精确信息,可以使上面的推理成立或不成立,即对这个精确的信息,加入“否定”之后,对段落推理成立方向刚好与原来相反,那么这个选项就为评价。评价型考题虽然在联考试卷中涉及不多,但这确实是恢趾芎玫穆呒庑停Ω贸晌窈罂际缘闹髁魈庑椭唬鹂忌愎坏闹厥印?/p>   -随着年龄的增长,人体对卡路里的日需求量逐渐减少,而对维生素的需求却日趋势多。因此,为了摄取足够的维生素,老年人应当服用一些补充维生素的保健品,或者应当注意比年轻时食用更多的含有维生素的食物。为了对上述断定做出评价,回答以下哪个问题最关重要?   A.对老年人来说,人体对卡路里需求量的减少幅度,是否小于对维生素需求量的增加幅度?   B.保健品中的维生素,是否比日常食品中的维生素更易被人体吸收?   C.缺乏维生素所造成的后果,对老年人是否比对年轻人更严重?   D.一般地说,年轻人的日常食物中的维生素含量,是否较多地超过人体的实际需要?   E.保健品是否会产生危害健康的副作用?   [解题分析]正确答案:D。   题干的议论要成立,需要满足一个条件,即年轻人的日常食物中的维生素含量,并非较多地超过人体的实际需要,否则,如果年轻人的日常食物中的维生素含量,实际上较多地超过人体的实际需要,那么,老年人只要维持年轻时的日常食物就可以了,毋须服用一些补充维生素的保健品,或者比年轻时食用更多的含有维生素的食物,因为年轻时的日常食物中超过实际需要的维生素,正发可以用来补充老年人增长的需要,这样题干的议论就不能成立。D项的问题涉及的正是这个条件,因此对于评判题干至关重要。其余各项中,A和C项与评判题干无关,B和E项与评判题干有关,但其重要性不如D项。   -据一项统计显示,在婚后的13年中,妇女的体重平均增加了15公斤,男子的体重平均增加了12公斤。因此,结婚是人变得肥胖的重要原因。为了对上述论证作出评价,回答以下哪个问题最为重要?   A.为什么这项统计要要选择13年这个时间段作为依据?为什么不选择其它时间段,例如为什么不是12年或14年? B.在上述统计中,婚后体重减轻的人有没有?如果有的话,占多大的比例? C.在被统计对象中,男女各占多少比例?   D.这项统计的对象,是平均体重较重的北方人,还是平均体重较轻的南方人?如果二者都有的话,各占多少比例?   E.在上述13年中,处于相同年龄段的单身男女的体重增减状况是怎样的?   [解题分析]正确答案:E。   E项提出的总是问题对评价题干的论证最为重要。因为如果在上述13年中,处于相同年龄段的单身男女的体重增减状况和题干的统计结果类似,那么,题干的结论就不能成立。   -人们对于搭乘航班的恐惧其实是毫无道理的。所统计,光1995年,全世界死于地面交通事故的人数超出80万,而在自1990年至1999年的10年间,全世界平均每年死于空难的还不到500人,而在这10年间,我国平均每年罹于空难的还不到25人。   为了评价上述论证的正确性,回答以下哪个问题最为重要?   A.在上述10年间,我国平均每年有多少人死于地面交通事故?   B.在上述10年间,我国平均每年有多少人加入地面交通,有多少人加入航运?   C.在上述10年间,全世界平均每年有多少人加入地面交通,有多少人加入航运?   D.在上述10年间,1995年全世界死于地面交通事故的人数是否是最高的?   E.在上述10年间,哪一年死于空难的人数最多?人数是多少?   [解题分析]正确答案:C。   回答C提出的问题对评价题干的论证最为重要。因为在对航运和地面交通的安全性进行比较时,在事故罹难者的绝对数量之间进行比较是没有意义的,正确的方法应是在事故率和事故死亡率之间进行比较。为了进行这种比较,不光要知道统计年限内航运和地面交通事故罹难者的绝对数字,而且要知道有多少人加入地面交通,有多少人加入航运。选项C提出的正是这个问题。   选项B提出的是类似的问题,但它仅涉及我国,不符合题干。   -在北欧一个称为古堡的城镇的郊外,有一个不乏凶禽猛兽的天然猎场。每年秋季,吸引了来自世界各地富于冒险精神的狩猎者。一个秋季下来,古堡镇的居民发现,他们之中此期间在马路边散步时被汽车撞伤的人的数量,比在狩猎时受到野兽意外伤害的人数多出了两倍!因此,对于古堡镇的居民来说,在狩猎季节,呆在猎场中比马路边散步更安全。   为了要评价上述结论的可信程度,最可能提出以下哪个问题?   A.在这个秋季,古堡镇有多少数量的居民去猎场狩猎?   B.在这个秋季,古堡镇有多少比例的居民去猎场狩猎?   C.古堡镇的交通安全纪录在周边几个城镇中是否是最差的?   D.来自世界各地的狩猎者在这个季节中有多少比例的人在狩猎时意外受伤?   E.古堡镇的居民中有多少好猎手?   [解题分析]正确答案:B。   一般地说,秋季古堡镇的居民在马路边散步的人数比较是很高的,只有在知道了古堡镇居民中去猎场狩猎的人数比较,对这两个场合中的受到意外伤害的人数进行比较才有意义。B项提出的正是这个问题,它对评价题干的结论最为重要。   如果题干中给出了在两个场合下受到意外伤害的具体人数以及古堡镇的居民人数,那么回答A项提出的问题就可以准确地认真出两个场合下的事故率并进行比较,但是题干中并没有给出这样的具体数字,因此,A项提出的问题无助于对题干的结论进行评价。   -毫无疑问,未成年人吸烟应该加以禁止。但是,我们不能为了防止给未成年人吸烟以可乘之机,就明令禁止自动售烟机的使用。这种禁令就如同为了禁止无证驾车在道路上没立路障,这道路障自然禁止了无证驾车,但同时也阻挡了99%以上的有证驾驶者。   为了对上述论证作出评价,回答以下哪个问题最为重要?   A.未成年吸烟者在整个吸烟者中所占的比例是否超过1%?   B.禁止使用自动售烟机带给成年购烟者的不便究竟有多大?   C.无证驾车者在整个驾车者中所占的比例是否真的不超过1%?   D.从自动售烟机中是否能买到任何一种品牌的香烟?   E.未成年人吸烟的危害,是否真如公众认为的那样严重?   [解题分析]正确答案:B。   如果自动售烟机也是成年吸烟者主要的甚至是惟一的购烟渠道的话,那么,题干的论证就能成立;否则,题干的论证将受到严重的质疑。因此,为了对题干的论证作出评价,回答B项的问题最为重要。事实上,禁止使用自动售烟机带给成年购烟者的不便越大,则对题干论证的支持力度越大;否则,对题干的削弱力度越大。   -在经历了全球范围的股市暴跌的冲击以后,T国政府宣称,它所经历的这场股市暴跌的冲击,是由于最近国内一些企业过快的非国有化造成的。以下哪项,如果事实上是可操作的,最有利于评价T政府的上述宣称?   A.在宏观和微观两个层面,对T国一些企业最近的非国有化进程的正面影响和负面影响进行对比。   B.把T国受这场股市暴跌的冲击程度,和那些经济情况和T国类似,但最近没有实行企业非国有化的国家所受到的冲击程度进行对比。   C.把T国受这场股市暴跌的冲击程度,和那些经济情况和T国有很大差异,但最近同样实行了企业非国有化的国家所受到的冲击程度进行对比。   D.计算出在这场股市风波中T国的个体企业的平均亏损值。   E.运用经济计量方法预测T国的下一次股市风波的时间。   [解题分析]正确答案:B。   对某个事物的评价,首先要有个评价的基准,也就是可比较的标准。   按照B的设计操作,那些经济情况和T国类似,但最近没有实行企业非国有化的国家,如果同样受到类似于T国的股市暴跌的冲击,则T国政府的宣称将受到严重质疑;否则,T国政府的宣称将受到支持。显然,这一操作有利于评价T政府的宣称。其余各项,对评价T国政府的宣称没有意义,或意义不大。 第四章 逻辑解题套路精析(五) 五、归纳   归纳题的解题思路是“自上而下”,即假定我们所面临的段落的推理成立,让我们从段落推理中推出某些结>咛宓厮担槟捎爰偕琛⒅С帧⒎炊浴⑵兰厶庑偷淖畲蟛钜煸谟冢汗槟伤媪俚亩温渫评硎强隙ǔ闪⒌耐评恚虼斯槟墒谴由厦娑温渲斜厝荒艿玫绞裁矗颐遣荒芑骋啥温渫评淼暮侠硇裕欢偕琛⒅С帧⒎炊浴⑵兰劭继馑媪俚亩温涫怯写兰鄣?!--NEWSZW_HZH_BEGIN--> 推理,因此这四类考题是让我们从五个选项中选择一个选项放到段落中对段落推理起到一定作用。   如何理解归纳推理中的必要条件对正确理解归纳推理非常重要。在归纳题型中,我们所面临的段落推理是完全成立的推理,所以推理之中的隐含假设(而这个隐含假设是段落推理成立的必要条件,而且许多推断题的答案本质上就是隐含假设)必定成立。在归纳中,有时段落推理可能仅仅只列举了使推理成立的一些必要条件,但段落推理的成立可能依赖于许多条件,只有所有的必要条件都找到了,才可以构成充分条件推导出推理的结论。诚然,有原因才能有确定的结果,但只有找到了所有影响某一确定结果的原因,我们才能得出这个确定的结果。但如果我们知道了某一确定结果,必定可以推断它的一些原因(必要条件)存在。同样,有条件才能有确定的结论,但只有找到了所有影响某一确定结论的条件,我们才能得出这个确定的结论,但是如果我们知道了某一确定的结论,必定可以推出它的必要条件存在,而只由一个或几个必要条件一般不能推出它的结论必然成立。   由于归纳题型的段落推理是肯定成立的推理,因此,我们不能够对段落的内容是否正确、结论是否荒谬、推理是否合理做出评价,我们的目标应锁定在怎么样才能找到能从段落推理中得出的一个选项。很多归纳题的阅读难度要大于假设、支持、反对、评价这四类题,并且由于归纳题的段落中每一句话都可以作为出题方向,所以读题要求比较高,读题时需要注意从逻辑层次结构上去把握段落推理关系,要学会一边读题一边思考原文的层次是什么,并在快速读完一遍题后脑中要将题干内容的层次分清楚。而且由于归纳题的段落推理是必定成立的推理,而且其答案是从上面段落中推出来的,因此,选项应为段落所涉及的内容。   (一)直接推断   这类题型的具体形式是:以题干为前提,要求在选项中确定合乎逻辑的结论;或者,从题干出发,不可能推出什么样的结论。解决直接推断型考题,考生只需运用日常逻辑推理就可以找到答案,几乎没有什么技巧可言。   -大多数工人的专业知识和技能都会逐渐过时,而从掌握到过时所需的时间目前由于新的生产工艺(AMT)和出现而被缩短。考虑到AMT的更新速度,一般的工人从技能的掌握到过时的时间逐渐缩短为4年。   以下哪项如果可行,将使企业在上述技能的加速折旧中,能最充分地利用工人的技能?   A.公司把能力强的雇员在他们进入公司的6年之后送去培训。   B.公司每年都对其为期5年的AMT计划追加投资。   C.公司定期走访诫雇员,来确定AMT计划对他们的影响。   D.在AMT计划实行之前,公司将开设一个教育机构来向雇员说明AMT计划将对他们产生的影响。   E.公司为其雇员定期开办培训,使他们不断适应工作的需要。   [解题分析]正确答案:E。   A和B项的时间周期过长,跟不上AMT的更新速度。C和D项无法说明如何充分地利用工人的技能。相比之下,E项如果可行,将使企业在上述技能的加速折旧中,能充分地利用工人的技能。   -如果能有效地利用互联网,能快速方便地查询世界各地的信息,对科学研究、商业往来乃至寻医求药都带来很大的好处。然而,如果上网成瘾,也有许多弊端,还可能带来严重的危害。尤其是青少年,上网成瘾可能荒废学业、影响工作。为了解决这一问题,某个网点上登载了“互联网瘾”自我测试办法。   以下各项提问,除了哪项,都与“互联网瘾”的表现形式有关?   A.你是否有时上网到深夜并为联结某个网站时间过长而着急?   B.你是否曾一再试图限制、减少或停止上网而不果?   C.你试图减少或停止上网时,是否会感到烦躁、压抑或容易动怒?   D.你是否曾因上网而危及一段重要关系或一份工作机会?   E.你是否曾向家人、治疗师或其他人谎称你并未沉迷互联网?   [解题分析]正确答案是A。   A项的提问涉及的只是“有时”上网时的表现,与上网成瘾无关。其余各项均涉及上网成瘾或其对工作的影响。   -以下诸项结论都是东方理工学院学生处根据各个系收到的1997~1998学年度奖助学金 申请表 食品经营许可证新办申请表下载调动申请表下载出差申请表下载就业申请表下载数据下载申请表 综合得出的。在此项综合统计做出后,因为落实灾区政策,有的系又收到了一些学生补交上来的申请表。   以下哪项结论最不可能被补交奖助学金申请表的新事实所推翻?   A.汽车系仅有14名学生交申请表,总申请金额至少有5700元。   B.物理系最多有7名学生交申请表,总申请金额为2800元。   C.数学系共有8名学生交申请表,总申请金额等于3000元。   D.化学系至少有5名学生交申请表,总申请金额多于2000元。   E.生物系至少有7名学生交申请表,总申请金额不会多于汽车系。   [解题分析]正确答案:D。   有上限的断言最容易被补交申请表的情况所推翻。而有上限的断言是:选项B、E。其次是数量准确的断言容易被补交申请表的情况所推翻,即选项A、C。选项D完全是低限断言,完全不受补交申请表的影响。   -在黑、蓝、黄、白四种由深至浅排列的涂料中,一种涂料只能被它自身或者比它颜色更深的涂料所覆盖。   若上述断定为真,则以下哪一项确切地概括了能被蓝色覆盖的颜色?   Ⅰ.这种颜色不是蓝色。   Ⅱ.这种颜色不是黑色。   Ⅲ.这种颜色不如蓝色深。   A.只有Ⅰ。 B.只有Ⅱ。 C.只有Ⅲ。   D.只有Ⅰ和Ⅱ。   E.Ⅰ、Ⅱ和Ⅲ。   [解题分析]正确答案:B。   Ⅲ没有包含蓝色本身,Ⅰ本身是假命题,因为蓝色也是能被蓝色覆盖的颜色,而且黑色不能被蓝色覆盖。   (二)确定论点及继续推论   确定论点及继续推论型题型的具体表现形式是给出一段文字或对话,要求总结它们所表达的中心内容是什么、什么内容没在题干中表达或段落最强调的是什么。或给出一段论述,要求推出结论或段落的主要观点(确定论点型暨继续推论型的变种:我们不可能得出的结论是什么)。其解题基本思路是对语言的理解,解此类题型主要是要凭语感、常识和日常的逻辑推理能力去寻找隐含的结论或内在的含义。   -有一种通过寄生方式来繁衍后代的黄蜂,它能够在适合自己后代寄生的各种昆虫的大小不同的虫卵中,注入恰好数量的自己的卵。如果它在宿主的卵中注入的卵过多,它的幼虫就会在互相竞争中因为得不到足够的空间和营养而死亡;如果它在宿主的卵中注入的卵过少,宿主卵中的多余营养部分就会腐败,这又会导致它的幼虫的死亡。   如果上述断定是真的,则以下哪项有关断定也一定是真的?   I.上述黄蜂的寄生繁衍机制中,包括它准确区分宿主虫卵大小的能力。   II.在虫卵较大的昆虫聚集区出现的上述黄蜂比在虫卵较小的昆虫聚集区多。   III.黄蜂注入过多的虫卵比注入过少的虫卵更易引起寄生幼虫的死亡。   A.仅I。   B.仅II。   C.仅III。   D.仅I和II。   E. I、II和III。   [解题分析]正确答案:A。   复选项Ⅰ一定是真的。否则,如果上述黄蜂的寄生繁衍机制中,不包括它准确区分宿主虫卵大小的能力,那么,它就不能在适合自己后代寄生的各种昆虫的大小不同的虫卵中,注入恰好数量的自己的卵。   复选项Ⅱ不一定是真的,因为完全可能虫卵较大的昆虫数量比虫卵较小的昆虫少得多,这样,上述黄蜂就会相对集中在虫卵较小的昆虫聚集区。   复选项Ⅲ显然不一定是真的。   -左撇子的人比右撇子的人更容易患某些免疫失调症,例如过敏。然而,左撇子也有优于右撇子的地方,例如,左撇子更擅长于由右脑半球执行的工作。而人的数学推理的工作一般是由右脑半球执行的。   从上述断定能推出以下哪个结论?   I.患有过敏或其他免疫失调症的人中,左撇子比右撇子多。   II.在所有数学推理能力强的人当中左撇子的比例,高于所有推理能力弱的人中左撇子的比例。   III.在所有左撇子中,数学推理能力强的比例,高于数学推理能力弱的比例。   A.仅I。   B.仅II。   C.仅III。   D.仅I和III。   E. I、II和III。   [解题分析]正确答案:C。   选项Ⅰ不能作为结论从题干中推出,因为从“左撇子的人比右撇子的人更容易患某些免疫失调症”,推不出“患免疫失调症的人中,左撇子比右撇子多”。例如,从“苗族姑娘比汉族姑娘更擅长跳舞”,推不出“擅长跳舞的人中,苗族姑娘比汉族姑娘多”。   选项Ⅱ也不能作为结论从题干中推出。   选项Ⅲ可以作为结论从题干中推出。否则,如果在所有左撇子中,数学推理能力强的比例,不高于数学推理能力弱的比例,那么,一般地左撇子并不擅长数学推理(充其量只比更不擅长数学推理的右撇子较强),这显然有悖于题干的断定。   -在2000年,世界范围的造纸业所用的鲜纸浆(即直接从植物纤维制成的纸浆)是回收纸浆(从废纸制成的纸浆)的2倍。造纸业的分析人员指出,到2010年,世界造纸业所用的回收纸浆将不少于鲜纸浆,而鲜纸浆的使用量也将比2000年有持续上升。   如果上面提供的信息均为真,并且分析人员的预测也是正确的,那么可以得出以下哪个结论?   I.在2010年,造纸业所用的回收纸浆至少是2000年的2倍。   II.在2010年,造纸业所用的总的纸浆至少是2000年的2倍。   III.造纸业在2010年造的只含鲜纸浆的纸将会比2000年少。   A.仅I。   B.仅II。   C.仅III。   D.仅I和II。   E.I、II和III。 [解题正确答案:A。 复选项Ⅰ一定是真的。因为由题干:   2000年鲜纸浆是回收纸浆的2倍;   到2010年,回收纸浆将不少于鲜纸浆;   到2010年,鲜纸浆的使用量多于2000年的使用量。   由此显然可以推出:在2010年,造纸业所用的回收纸浆至少是2000年的2倍。 H:\fanwen caiji two\委托技术开发合同1.doc  选项Ⅱ不一定是真的。例如,假设2000年鲜纸浆的用量是1个单位,回收纸浆是05个单位;到2010年,鲜纸浆的用量是11个单位,回收纸浆是12个单位。这一假设符合题干的所有条件,但2010年纸浆总用量少于2000年的2倍。   选项Ⅲ不一定是真的,造纸业在2010年造的只含鲜纸浆的纸将会比2000年较少。   -如果二氧化碳气体超量产生,就会在大气层中聚集,使全球气候出现令人讨厌的温室效应。在绿色植被覆盖的地方,特别是在森林中,通过光合作用,绿色植被吸收空气中的二氧化碳,放出氧气。因此,从这个意义上,绿色植被特别是森林的破坏,就意味着在"生产"二氧化碳。工厂中对由植物生成的燃料的耗用产生了大量的二氧化碳气体。这些燃料包括木材,煤和石油。   上述断定最能支持以下哪项结论?   A.如果地球上的绿色植被特别是森林受到严重破坏,将使全球气候不可避免地出现温室效应。   B.只要有效地保护好地球上的绿色植被特别是森林,那么,即便工厂超量耗用由植物生成的燃料,也不会使全球的气候出现温室效应。   C.如果各国工厂耗用的由植物生成的燃料超过了一定的限度,那就不可避免地使全球气候出现温室效应,除非全球的绿色植被特别是森林得到足够良好的保护。   D.只要各国工厂耗用的由植物生成的燃料控制在一定的限度内,就可使全球气候的温室效应避免出现。   E.如果全球气候出现了温室效应,则说明或者是全球的绿色植被没有得到有效的保护,或者各国的工厂耗用了超量的由植物生成的燃料。   [解题分析]正确答案:C。   题干中做出了三个断定:   工厂产生二氧化碳;   绿色植被特别是森林吸收二氧化碳;   如果二氧化碳超量产生,则会出现全球气候的温室效应。   从上述三个断定可以得出结论:如果工厂产生的二氧化碳超过了一定的限度,并且没有足够的绿色植被特别是森林吸收二氧化碳,那么就会不可避免地使全球出现温室效应。这正是C项所断定的。   其余各项均不能从题干推出。   -某公司在一次招聘中,对所有申请者进行了一次书面测试,其中包括这样一个问题:"你是否是一个诚实的人?"有五分之二的申请者的回答是:"我至少有一点不诚实。"该公司在这次测试中,很可能低度估申请者中不诚实的人所占的比例,因为   以下哪项作为上文的后续最为恰当?   A.在这次测试中,有些非常诚实的申请者可能作了不诚实的回答。   B.在这次测试中,那些回答"我至少有一点不诚实"的申请者可能是非常不诚实的。   C.在这次测试中,那些回答自己是不诚实的申请者,他所作的这一回答可能是诚实的。   D.在这次测试中,有些不诚实的申请者可能宣称自己是诚实的。   E.在这次测试中,其余五分之三的申请者中,可能很多人的回答是"我非常不诚实"。   [解题分析]正确答案:D。   当一个申请者回答自己不诚实(不管何种程度上),总可以判定他是不诚实的。因为如果他的回答是假的,则他不诚实(例如,如果“我至少有一点不诚实”是句假话,那么事实上他可能非常不诚实);如果他的回答是真的,则他自然是不诚实的。因此,当一个申请者回答自己不诚实时,是不可能使该公司低估申请者中不诚实的人所占的比例的。所以,B、C和E作为题干的后续均不恰当。A项使该公司可能高估申请者中不诚实的人所占的比例,作为题干的后续自然不恰当。   D项作为题干的后续恰当。   -在大型游乐公园里,现场表演是刻意用来引导人群流动的。午餐时间的表演是为了减轻公园餐馆的压力;傍晚时间的表演则有一个完全不同的目的:鼓励参观者留下来吃晚餐。表面上不同时间的表演有不同的目的,但这背后,却有一个统一的潜在目标,即   以下哪一选项作为本段短文的结束语最为恰当?   A.尽可能地减少各游览点的排队人数。   B.吸引更多的人来看现场表演,以增加利润。   C.最大限度地避免由于游客出入公园而引起交通阻塞。   D.在尽可能多的时间里最大限度地发挥餐馆的作用。   E.尽可能地招徕顾客,希望他们再次来公园游览。   [解题分析]正确答案:D。由题干知,大型游乐公园里的有两个经营项目:现场表演与公园餐馆。从题干的陈述不难发现,第一个项目是为第二个项目服务的,即现场表演的目的,是通过对人群流动的引导,在尽可能多的时间里最大限度地发挥餐馆的作用。因此,D项恰当。其余各项所断定的都可能是现场表演的,作为结束语言均不恰当。   -血液中的高浓度脂肪蛋白含量的增多,会增加人体阻止吸收过多的胆固醇的能力,从而降低血液中的胆固醇。有些人通过有规律的体育锻炼和减肥,能明显地增加血液中高浓度脂肪蛋白的含量。   以下哪项,作为结论从上述题干中推出最为恰当?   A.有些人通过有规律的体育锻炼降低了血液中的胆固醇,则这些人一定是胖子。   B.不经常经进体育锻炼的人,特别是胖子,随着年龄的增大,血液中出现高胆固醇的风险越来越大。   C.体育锻炼和减肥是降低血液中高胆固醇的最有效的方法。   D.有些人可以通过有规律的体育锻炼和减肥来降低血液中的胆固醇。   E.标准体重的人只需要通过有规律的体育锻炼就能降低血液中的胆固醇。 [解题分析]正确答案是D。   题干断定:有些人通过体育锻炼和减肥,能增加血液中的高浓度脂肪蛋白;同时,血液中的高浓度脂肪蛋白含量的增多,会降低血液中的胆固醇。由此可以推出,有些人可以通过体育锻炼和减肥来降低血液中的胆固醇。因此,D项作为题干的推论是恰当的。C项和D项类似,但其所做的断定过强,作为从题干推出的结论不恰当。其余各项均不恰当 第四章 逻辑解题套路精析(六) http://www.sina.com.cn 2003/10/27 14:33 新浪教育   六、解释   解释题型的特征是,给出一段关于某些事实或现象的客观描述,要求你对这些事实、现象、结果或矛盾做出合理的解释。其主要表现形式是,在题干中给出某种需要说明、解释的现象,再问什么样的理由、根据、原因能够最好地解释该现象,或最不能解释该现象,即与该现象的发生不相干。我们可以根据解释的侧重点把考题分为解释结论或现象、解释差异 或缓解矛盾。解这类题型有时需要一些相关的背景知识,但这些知识都属于语言常识和一般性常识,并且已经在题干或选项中给出,只是要求从中做一些选择和判断而已。   (一)解释结论或现象   本类考题是指给出一段关于某些事实或现象的客观描述,让我们从5个选项中寻求一个选项来解释事实或现象发生的原因,找到一个能直接说明结论能够成立或现象为什么发生的选项即可。因此在解题时,应抓住要解释的对象,一般首先要明确解释的关键概念,并用之来定位选项。   -第一个事实:电视广告的效果越来越差。一项跟踪调查显示,在电视广告所推出的各种商品中,观众能够记住其品牌名称的商品的百分比逐年降低。   第二个事实:在一段连续插播的电视广告中,观众印象较深的是第一个和最后一个,而中间播出的广告留给观众的印象,一般地说要浅得多。以下哪项,如果为真,最能使得第二个事实成为对第一个事实的一个合理解释?   A.在从电视广告里见过的商品中,一般电视观众能记住其品牌名称的大约还不到一半。   B.近年来,被允许在电视节目中连续插播广告的平均时间逐渐缩短。   C.近年来,人们花在看电视上的平均时间逐渐缩短。   D.近年来,一段连续播出的电视广告所占用的平均时间逐渐增加。   E.近年来,一段连续播出的电视广告中所出现的广告的平均数量逐渐增加。   [解题分析]正确答案:E。   题干的事实2断定,在一段连续插播的电视广告中,观众印象较深的是第一个和最后一个,其余的则印象较浅;而E项断定,一个广告段中所包含的电视广告的平均数目增加了。由这两个条件可推知,近年来,在观众所看到的电视广告中,印象较深的所占的比例逐渐减少,这就从一个角度合理地解释了,为什么在电视广告所推出的各种商品中,观众能够记住其品牌名称的商品的比重在下降。   其余各项都不能起到上述作用。其中,B和C项有利于说明,近年来人们看到的电视广告的数量逐渐减少,但不能说明,在人们所看过的电视广告中,为什么能记住的百分比逐年降低。D项断定,近年来,一段连续播出的电视广告所占用的平均时间逐渐增加,由此不能推出,一段连续播出的电视广告中所出现的广告的平均数量逐渐增加,因为完全可能少数几个广告所占的时间增加了,而人们在所看过的广告中能记住的百分比并不会降低。   -据一项有几个大城市的统计显示,餐饮业的发展和瘦身健身业的发展呈密切正相关。从1985年到1990年,餐饮业的网点增加了18%,同期在健身房正式注册参加瘦身健身的人数增加了17.5%;从1990年到1995年,餐饮业的网点增加了25%,同期参加瘦身健身的人数增加了25.6%;从1995年到2000年,餐饮业的网点增加了20%,同期参加瘦身健身的人数也正好增加了20%。   如果上述统计真实无误,则以下哪项对上述统计事实的解释最可能成立?   A.餐饮业的发展,扩大了肥胖人群体,从而刺激了瘦身健身业的发展。   B.瘦身健身运动,刺激了参加者的食欲,从而刺激了餐饮业的发展。   C.在上述几个大城市中,最近15年来,主要从事低收入重体力工作的外来人口的逐年上升,刺激了各消费行业的发展。   D.在上述几个大城市中,最近15年来,城市人口的收入的逐年提高,刺激了包括餐饮业和健身业在内的各消费行业的发展。   E.高收入阶层中,相当一批人既是餐桌上的常客,又是健身房内的常客。   [解题分析]正确答案:D。   选项A和B说明,餐饮业和瘦身健身业的发展互相促进,但是不大容易说明,二者的增长百分比何以如此接近。   选项C说明,外来人口的上升刺激了各消费行业的发展,这对题干是一种解释,但解释的力度不大。因为外来人口主要从事低收入重体力工作,因此有理由认为,他们对餐饮业特别是瘦身健身业发展的刺激,是非常有限的;特别难以说明,二者的增长百分比何以如此接近。   选项D说明,城市人口收入的逐年提高,是造成餐饮业和健身业以接近的增长百分比同步发展的原因。这是各选项中对题干的统计事实最合理的解释。   -烟草业仍然是有利可图的。在中国,尽管今年吸烟者中成人的人数减少,烟草生产商销售的烟草总量还是增加了。   以下哪项不能用来解释烟草销售量的增长和吸烟者中成人人数的减少?   A.今年中,开始吸烟的妇女数量多于戒烟的男子数量。   B.今年中,开始吸烟的少年数量多于同期于戒烟的成人数量。   C.今年,非吸烟者中咀嚼烟草及嗅鼻烟的人多于戒烟者。   D.今年和往年相比,那些有长年吸烟史的人平均消费了更多的烟草。   E.今年中国生产的香烟中用于出口的数量高于往年。   [解题分析]正确答案:A。   需要解释的是:为什么今年成人吸烟者人数减少了,但烟草的销售量却增加了。   B、C、D和E项显然都能对此作出解释。例如,据B项,烟草销售量的增加可能是由于少年吸烟量的增加;据E项,烟草销售量的增加可能是由于外销量的增加。   A项不能对此作出解释。因为虽然今年开始吸烟的归女数量多于戒烟的男子数量,但是由于成人吸烟者(包括男子和妇女)的数量总体上减少了,因此,不能得出结论:是这些新吸烟的妇女造成了烟草销售量的增加。 -在美国与西班牙作战期间,美国海军曾经广为散发海报,招募兵员。当时最有名的一个海军广告是这样说的:美国海军的死亡率比纽约市民还要低。海军的官员具体就这个广告解释说:“根据统计,现在纽约市民的死亡率是每千人有16人,而尽管是战时,美国海军士兵的死亡率也不过每千人只有9人。” 如果以上资料为真,则以下哪项最能解释上述这种看起来很让人怀疑的结论?   A.在战争期间,海军士兵的死亡率要低于陆军士兵。   B.在纽约市民中包括生存能力较差的婴儿和老人。   C.敌军打击美国海军的手段和途径没有打击普通市民的手段和途径来的多。   D.美国海军的这种宣传主要是为了鼓动入伍,所以,要考虑其中夸张的成分。   E.尽管是战时,纽约的犯罪仍然很猖獗,报纸的头条不时地有暴力和色情的报道。   [解题分析]正确答案:B。   题干广告中隐含的结论是,到海军服役不比在后方城市中生活危险。这个结论是建立在将两个具有不同内容的数字进行不恰当比较的基础上的。海军士兵正处于生存能力最佳状态的年龄段,造成他们死亡的几乎唯一的原因,是直接死于战争。如果处于后方的纽约市民具有和海军士兵相同的生存能力状态,前者的死亡率无疑要低得多;B项断定,在纽约市民中包括生存能力较差的婴儿和老人,这就抓住了题干进行不恰当比较的实质,并为统计数据所显示的纽约市民死亡率高于海军士兵的现象提供了一个合理的解释。   C项和E项断定的也是对纽约市民构成威胁的因素,但没有理由认为这些因素造成的威胁会大于直接的战争,因此如果不首先断定纽约市民和海军士兵处于不同的生存能力状态,C项和E项都不能对题干的统计数据提供解释。因为条件已假设题干提供的资料为真,所以,D项不成立。A项的不成立是显然的。   -几年来,我国许多餐厅使用一次性筷子。这种现象受到越来越多的批评,理由是我森林资源不足,把大好的木材用来做一次性筷子,实在是莫大的浪费。但奇怪的是,至今一次性筷子的使用还没有被禁止。   以下除哪项外,都能对上文的疑问从某一方面给以解释?   A.有些一次性筷子不是木制的,有些一次性木制筷子并没有使用森林中的木材。   B.已经证明了,一次性筷子的使用能有效的避免一些疾病的交叉感染。   C.一次性筷子的使用与餐厅之间相互攀比有关,要禁必须大家一起禁才行。   D.一次筷子并不如想像的那样卫生,有些病菌或病毒也会借助一次性筷子传播。   E.保护森林不能只保不用。合理地使用,适量地采伐,有利于森林的保护。   [解题分析]正确答案:D。   要想对上文疑问给以某一方面的解释,就一定要调和题干中的矛盾,而选项D非但没有这样做,反而加剧了题干的矛盾,故选D。   选项A、B、E都提供了一些额外信息,说明一次性筷子的好处;选项C说明了一次性筷子没有被禁止的一个现实原因。   -一项对东华大学企业管理系94届毕业生的调查的结果看来有些问题,当被调查毕业生被问及其在校时学习成绩的名次时,统计资料表明:有60%的回答者说他们的成绩位居班级的前20%。   如果我们已经排除了回答者说假话的可能,那么下面哪一项能够对上述现象给出更合适一些的解释?   A.未回答者中也并不是所有的人的成绩名次都在班级的前20%以外。   B.虽然回答者没有错报成绩,但不排除个别人对于学习成绩的排名有不同的理解。   C.东华大学对学生学习成绩的名次排列方式与其他大多数学校不同。   D.成绩较差的毕业生在被访问时一般没有回答这个有关学习成绩名次的问题。   E.在校学习成绩名次是一个敏感的问题,几乎所有的毕业生都进行略微的美化。   [解题分析]正确答案:D。   如果所有的上述毕业生都受到了调查,并且所有的被调查者都回答了所有的问题,那么,题干的断定就包含了明显的矛盾。D项断定,成绩较差的毕业生在被访问时一般没有回答这个有关学习成绩名次的问题,因此,产生题干矛盾的条件不成立,也就是说,回答问题者的成绩都较好,因此,其中的60%的成绩居前20%就不奇怪。这就对题干做出了一个恰当的解释。其余各项对题干都不能做出有说服力的解释。   (二)解释差异或缓解矛盾   对解释矛盾或差异型考题,解题思路的关键是抓住原文差异双方的差别,正是这一差别导致了现象的矛盾。   -胡萝卜、西红柿和其他一些蔬菜含有较丰富的β-胡萝卜素,β-胡萝卜素具有防止细胞癌变的作用。近年来提炼出的β-胡萝卜素被制成片剂并建议吸烟者服用,以防止吸烟引起的癌症。然而,意大利博洛尼亚大学和美国德克萨斯大学的科学家发现,经常服用β-胡萝卜素片剂的吸烟者反而比不常服用β-胡萝卜素片剂的吸烟者更易于患癌症。   以下哪项如果为真,最能够解释上述矛盾?   A.有些β-胡萝卜素片剂含有不洁物质,其中有致癌物质。   B.意大利博洛尼亚大学和美国德克萨斯大学地区的居民吸烟者中癌症患者的比例都较其他地区高。   C.经常服用β-胡萝卜素片剂的吸烟者有其它许多易于患癌症的不良习   惯。   D.β-胡萝卜素片剂不稳定,易于分解变性,从而与身体发生不良反应,易于致癌。而自然β-胡萝卜素片剂性质稳定,不会致癌。   E.吸烟者吸入体内烟雾中的尼古丁与β-胡萝卜素发生作用,生成一种比尼古丁致癌作用更强的有害物质。   [解题分析]正确答案:E。   E项如果为真,说明β-胡萝卜素的抗癌作用,不是无条件的。吸烟者吸入体内烟雾中的尼古丁与β-胡萝卜素发生作用,生成一种比尼古丁致癌作用更强的有害物质,这就造成经常服用β-胡萝卜素片剂的吸烟者反而比不常服用β-胡萝卜素片剂的吸烟者更易于患癌症。这有说明力地解释了题干陈述的现象。   -近期的一项调查显示:日本产“星愿”、德国产“心动”和美国产的“EXAP”三种轿车最受女性买主的青睐。调查指出,在中国汽车市场上,按照女性买主所占的百分比计算,这三种轿车名列前三名。星愿、心动和EXAP三种车的买主,分别有58%、55%和54%是妇女。但是,最近连续6个月的女性购车量排行榜,却都是国产的富康轿车排在首位。   以下哪项如果为真,最有助于解释上述矛盾?   A.每种轿车的女性买主占各种轿车买主总数的百分比,与某种轿车的买主之中女性所占的百分比是不同的。   B.排行榜的设立,目的之一就是引导消费者的购车方向。而发展国产汽车业,排行榜的作用不可忽视。 C.国产的富康轿车也曾经在女性买主所占的百分比的排列中名列前茅,只是最近才落到了第四名的位置。 D.最受女性买主的青睐和女性买主真正花钱去购买是两回事,一个是购买欲望,一个是购买行为,不可混为一谈。   E.女性买主并不意味着就是女性来驾驶,轿车登记的主人与轿车实际的使用者经常是不同的。而且,单位购车在国内占到了很重要的比例,不能忽略不计。   [解题分析]正确答案:A。   在近期中国汽车的市场上,按照女性买主所占的百分比,题干一方面断定,星愿、心动和EXAP名列前三,另一方面又断定富康位居榜首,这看来自相矛盾,其实并不矛盾。因为前者排名的依据,是某种轿车的买主之中女性所占的百分比,例如,星愿车的买主中,58%是女性;后者排名的依据,是富康车的女性买主占各种轿车买主总数的百分比。因此,这样情况完全是可能的:尽管富康车的女性买主在各种轿车买主总数中所占的百分比居第一,但是,富康车的买主中,女性的比例却低于54%。这样,题干的断定就不存在任何矛盾。A项正是指出了这一点,因此有助于解释题干中似乎存在的矛盾。其余各项都无助于做到这一点。   -尽管是航空业萧条的时期,各家航空公司也没有节省广告宣传的开支。翻开许多城市的晚报,最近一直都在连续刊登如下广告:飞机远比汽车安全!你不要被空难的夸张报道吓破了胆,根据航空业协会的统计,飞机每飞行1亿公里死1人,而汽车每走5000万公里死1人。   汽车工业协会对这个广告大为恼火,他们通过电视公布了另外一个数字:飞机每20万飞行小时死1人,而汽车每200万行驶小时死1人。   如果以上资料均为真,则以下哪项最能解释上述这种看起来矛盾的结论?   A.安全性只是人们在进行交通工具选择时所考虑问题的一个方面,便利性、舒适感以及某种特殊的体验都会影响消费者的选择。   B.尽管飞机的驾驶员所受的专业训练远远超过汽车司机,但是,因为飞行高速的原因,飞机失事的生还率低于车祸。   C.飞机的确比汽车安全,但是,空难事故所造成的新闻轰动要远远超过车祸,所以,给人们留下的印象也格外深刻。   D.两种速度完全不同的交通工具,用运行的距离做单位来比较安全性是不全面的,用运行的时间来比较也会出偏差。   E.媒体只关心能否提高收视率和发行量,根本不尊重事情的本来面目。   [解题分析]正确答案:D。   题干中的第一个统计数字似乎说明飞机比汽车安全,第二个统计数字似乎说明汽车比飞机安全,而题干又断定这两个统计数字都正确,这似乎存在矛盾,其实并不矛盾。因为飞机和汽车的速度明显不同。在不知道二者的速度或速度比的情况下,只以运行距离为单位,或者只以运行时间为单位   无法比较二者的安全性。D项正确地指明了这一点。其余各项作为对题干的解释均不得要领。   -经济学家与考古学家就货币的问题展开了争论。   经济学家:在所有使用货币的文明中,无论货币以何种形式存在,它都是因为其稀缺性而产生其价值的。   考古学家:在索罗斯岛上,人们用贝壳作货币,可是该岛上贝壳遍布海滩,随手就能拾到呵。   下面哪项能对二位专家论述之间的矛盾做出解释?   A.索罗斯岛上居民节日期间在亲密的朋友之间互换货币,以示庆祝。   B.索罗斯岛上的居民认为鲸牙很珍贵,他们把鲸牙串起来当作首饰。   C.索罗斯岛上的男女居民使用不同种类的贝壳作货币,交换各自喜爱的商   品。   D.索罗斯岛上的居民只使用由专门工匠加工的有美丽花纹的贝壳作货币。   E.即使在西方人将贵金属货币带上索罗斯岛之后,贝壳仍然是商品交换的媒介物。   [解题分析]正确答案:D。   D项断定,索罗斯岛上的居民只使用由专门工匠加工的有美丽花纹的贝壳作货币,这说明,虽然作为货币原料的贝壳遍布海滩,但作为货币本身的加工过的贝壳仍可能因其稀缺性而产生价值。这样,经济学家和考古家的观点并不存在矛盾。其余各项均不能对题干做出解释。 第四章 逻辑解题套路精析(七) http://www.sina.com.cn 2003/10/29 08:28 新浪教育   七、逻辑相关   MBA逻辑考试中有些题型似乎不太象逻辑猓颐前颜庑┛继夤槲奥呒喙亍崩啵饕ㄈ郑阂皇牵镆謇斫庑停庵痔庑陀械闫蛴谟镂闹械脑亩晾斫猓欢牵窒葳逵爰扑阈停庵痔庑突蚱蛴谒闶踔械募扑阌胧壑叮蚩疾於愿髦质质抵室庖宓睦斫猓蝗牵硕曰盎虮缏坌停庵痔庑透龅男问绞橇礁鋈说亩曰埃梢愿鞲鼋嵌确⑽剩?!--NEWSZW_HZH_BEGIN--> 因此其推理思路是对前面六大类逻辑推理题型的综合运用,很难把它具体归结为哪一类,这里也把其视为“逻辑相关”型的一种。   (一)语义理解   在我们平时的语言表达中也往往存在逻辑问题。对于日常语言由于所处环境的不同以及受话人个体的差异,往往有不同的理解。但在特定的语境下,一句话的含义应该是确定的。在需要确定一句话或一段话的真实含义时,有必要进行一定的语义分析。具体说,我们通常进行推理时,前提和结论之间总是存在着某种共同的意义内容,使得我们可以由前提推出结论。形式逻辑通常不理会推理内容的相关性,但批判性思维和以它为基础的MBA逻辑考试却要顾及前提和结论之间的这种内容相关性,并为此设计了许多要考虑题干和备选答案之间的语义关联的考题。   因此,解这类题的基本思路:一是要阅读仔细,通过对选项和题干的内容逐一对照,从迅速发现找到答案的线索;二是,充分运用自己平时积累起来的语感,力求准确理解、分析和推断题干给出的日常语言表达的句子或内容的复杂含义和深层意义。   -如果你的笔记本计算机是1999年以后制造的,那么它就带有调制解调器。   上述断定可由以下哪个选项得出?   A.只有1999年以后制造的笔记本计算机才带有调制解调器。   B.所有1999年以后制造的笔记本计算机都带有调制解调器。   C.有些1999年以前制造的笔记本计算机也带有调制解调器。   D.所有1999年以前制造的笔记本计算机都不带有调制解调器。   E.笔记本的调制解调器技术是在 1999年以后才发展起来的。 [解题分析]正确答案:B。   B项可由题干的断定推出,因为如果B项不成立,则题干的断定也不成立。假设B项不成立,则事实上有些1999年以后制造的笔记本计算机不带有调制解调器;因此不能由你的笔记本计算机是1999年以后制造的,就断定它带有强制解调器。   题干断定:对于笔记本计算机来说,1999年以后制造,是它就带有调制解调器的充分条件,而A项断定,1999年以后制造,是笔记本计算机带有调制解调器的必要条件。对于任意两种事物情况p和q,断定p和q的充分条件,推不出断定p是q的必要条件。因此,A项不能由题干推出。   其余各项显然不能由题干推出。   -任何方法都是有缺陷的。在母语非英语的外国学生中,如何公正合理地选拔合格的考生?对于美国这样一个每年要吸收大量外国留学生的国家来说,目前实行的托福考试恐怕是所有带缺陷的方法中最好的方法了。以下各项关于托福考试及其考生的断定都符合上述议论的含义,除了   A.大多数考生的实际水平与他们的考分是基本相符的。   B.存在低考分的考生,他们有较高的实际水平。   C.高分低能或低分高能现象的产生,是实施考试中操作失误所致。   D.存在高分的考生,他们并无相应的实际水平。   E.对美国来说,目前恐怕没有比托福考试更能使人满意的方法来测试外国考生的英语能力。   [解题分析]正确答案:C。   选项中赞成托福考试的都应当排除,这样选项A、E就去掉了;题干中也突出承认了托福考试的缺陷,选项B和D从两个角度说明了这一点,与题干含义相符,因此也被排除。   选项C强调托福考试的缺陷并非方法设计上的问题,而是实际操作中的失误,与题干含义存在一定的矛盾之处,当然也并不那么直接,但是相对其他选项,C显然是最合适的。   -中国青少年发展基金会在成功地推行希望工程八年之后,又面向社会隆重推出“中华古诗文经典诵读”工程。为此,国家科学技术部研究中心组成了专题评估小组,进行了抽样调查,得到有效样本1342个。调查结果显示:多数家长和教师认为古诗文诵读应从小抓起,作为孩子启蒙教育的一部分,这既能修身养性,又能促进学习。   以下哪项结论最不符合以上题干所表达的思想?   A.国家科学技术部研究中心组成的专题评估小组采取了科学的研究方法,进行了细致的调查研究,其结论很有说服力。   B.古诗文诵读不仅能让孩子学习语文知识,对加强精神文明建设也具有重要意义。   C.有了推行希望工程的经验,中华古诗文经典诵读工程成功的可能性更大了。   D.由于孩子们上中学以后,数理化课程加重,需要把古诗文诵读的任务放在幼儿园和小学阶段,在中学和大学阶段集中力量学习科学技术。   E.少数老师和学生家长认为不应该过分强调古诗文诵读,因为在孩子小的时候,过分强调背诵,会影响独立思考能力的培养。   [解题分析]正确答案:D。   从选项D的叙述可以看出,古诗文诵读虽然有益,但最好放在幼儿园和小学阶段,到中学和大学以后,课程加重,需要集中力量学习科学技术。言外之意,古诗文诵读会影响学习。这与题干中说的“既能修身养性,又能促进学习”的论点不符。   选项A、B支持题干的观点,选项C也不违背题干的意思。选项D说“少数老师和学生家长认为不应该过分强调古诗文诵读”,而题干讲“多数家长和教师认为古诗文诵读应从小抓起”,一个是“少数”,一个是“多数”,并不矛盾。   -某计算机销售部向顾客承诺:“本部销售的计算机在一个月内包换、一年内免费包修、三年内上门服务免收劳务费,因使用不当造成的故障除外。”   以下哪项所讲的是该销售部应该提供的服务?   A.某人购买了一台计算机,三个月后软驱出现问题,要求销售部修理,销售部给免费更换了软驱。   B.计算机实验室从该销售部购买了30台计算机,50天后才拆箱安装。在安装时发现有一台显示器不能显示彩色,要求更换。   C.某学校购买了10台计算机。没到一个月,计算机的鼠标丢失了三个,要求销售部无偿补齐。   D.李明买了一台计算机,不小心感染了计算机病毒,造成存储的文件丢失,要求销售部赔偿损失。   E.某人购买了一台计算机,一年后键盘出现故障,要求销售部按半价更换一个新键盘。   [解题分析]正确答案:A。   选项A所论述的情况属于一年内免费包修的服务项目,在软驱不能修时,销售部给免费更换软驱是应该的。   选项B中的情况是购买后50天的事,虽然没拆箱,但已过了包换的期限,计算机销售部可以担保其免费修理服务,但不一定包换。C中提到的鼠标丢失是保管不当造成的。D中的计算机病毒破坏是由于防范不当或使用不当引起的,公司没有责任。E中情况超过了免费包修期,三年内虽然可以免服务费,零件费还是要照全价付的。   -科学家的平均收入与他们作出的贡献比起来是太低了。最杰出的科学家的收入不应该和普通的名演员、歌星、体育明星、大饭店经理相比,应该和他们之中的最杰出者相比。   除了以下哪项,其余各项都可能是上述议论所表达的意思?   A.有的科学家的收入和他们作出的贡献比起来不算太低。   B.最杰出的科学家的收入并不比普通的名演员、歌星、体育明星和大饭店经理低。   C.最杰出的名演员、歌星、体育明星、大饭店经理的收入一般地要高于最杰出的科学家。   D.最杰出的科学家的收入一般地还不如普通的名演员、歌星、体育明星和大饭店经理。   E.最杰出的科学家的收入不应该低于最杰出的名演员、歌星、体育明星和大饭店经理。   [解题分析]正确答案:D。   题干说“最杰出的科学家的收入不应该和普通的名演员、歌星、体育明星、大饭店经理相比,应该和他们之中的最杰出者相比”,就是隐含说尽管现在这些最杰出的科学家的收入比普通的名演员等等要高,但却无法与他们之中的最杰出者相比(就是选项B和选项C的结合)。所以只有选项D与此内容相反,故选D。   (二)数字陷阱与计算   数字陷阱与计算型考题主要包括两类:一是考察对数字的理解;二是有一些非常接近算术题,需要进行一定的计算或对数论知识的运用。 在当代社会,各种数字、数据、报表可以说铺天盖地,频频出现在大众传媒之中,我们常常会想这些数字、数据准确、可靠吗?对“精确”数字保持必要的警惕,应该说是一种明智的、理性的态度。下面揭示一些隐藏在“精确”数字背后的陷阱:一是平均数陷阱,在对平均数的模糊理解做文章;二是百分比陷阱,一般题干仅提供两种事物的某种比率就比较出两种事物的结果,其实其陷阱就在于该百分比所赖以计算出来的基数是不同的;三是错误比较,或者不设定供比较的对象,不设定比较的根据或基础,因此,表面上在进行比较,实际上根本就不能比较。 数字在逻辑题中扮演着非常重要的角色,段落中出现的数字应引起我们的高度警觉。命题人员一般不考绝对的数值,而喜欢在相对的数字上故弄玄虚。一般来讲,数量与比例相结合才能说明问题,而仅仅有比例或数量大多不能说明问题,尽管比例要比数字重要。对于这一类题目,可转化为数学去思维(比例与数量作为“题眼”也在反对或支持题型中大量出现)。   数字陷阱型考题很多也涉及统计问题,因此,有必要介绍一下有关的统计常识。统计结论的可靠性主要取决于样本的代表性。只有从能够代表总体的样本出发,才能得到关于总体的可靠结论。一般从抽样的规模、抽样的广度和抽样的随机性三个方面去保证样本的代表性。对于任何一个抽样统计结果,我们都可以从这些角度去质疑它的可靠性。同时,我们还应该懂一些计量经济学研究的基本观念:不能只看到两组数据之间的正比或反比关系,关键要分析其背后的社会经济联系。比如两组数据的相关性很强(存在正比或反比关系),但仅此而已相互间并没有任何因果关系,有时也可能两组数据都是由第三种数据决定的。   -如果一个用电单位的日均耗电量超过所在地区80%用电单位的水平,则称其为该地区的用电超标单位。近三年来,湖州地区的用电超标单位的数量逐年明显增加。   如果以上断定为真,并且湖州地区的非单位用电忽略不计,则以下哪项断定也必定为真?   I.近三年来,湖州地区不超标的用电单位的数量逐年明显增加。   II.近三年来,湖州地区日均耗电量逐年明显增加。   III.今年湖州地区任一用点超标单位的日均耗电量都高于全地区的日均耗电量。   A.只有I   B.只有II   C.只有III   D.只有II和III   E. I、II和III   [解题分析]正确答案:A。   由题干,湖州地区用电单位中,超标单位占20%,不超标单位占80%。又近三年来,湖州地区的用电超标单位的数量逐年明显增加,因此,显然可以得出结论:近三年来,湖州地区不超标的用电单位的数量逐年明显增加。否则,超标单位的比例就会超过20%,有悖于题干中关于超标单位的定义。所以复选项Ⅰ一定为真。   复选项Ⅱ不一定为真。因为由题干,一个单位是否为用电超标单位,不取决于自己的绝对用电量,而取决于和其他单位比较的相对用电量。因此,用电超标单位的数量的增加,并不一定导致实际用电量的增加。   复选项Ⅲ不一定为真。例如,假设该地区共有10个用电单位,其中8个不超标单位分别日均耗电1个单位,2个超标单位中,一个日均耗电2个单位,另一个日均耗电30个单位。这个假设完全符合题干的条件,但日均耗电2个单位的超标单位,其日均耗电量并不高于全地区的日均耗电量。   -1998年度的统计显示,对中国人的健康威胁最大的三种慢性病,按其在总人口中的发病率排列,依次是乙型肝炎、关节炎和高血压。其中,关节炎和高血压的发病率随着年龄的增长而增加,而乙型肝炎在各个年龄段的发病率没有明显的不同。中国人口的平均年龄,在1998年至2010年之间,将呈明显上升态势而逐步进入老人社会。   依据题干提供的信息,推出以下哪项结论最为恰当?   A.到2010年,发病率最高的将是关节炎。   B.到2010年,发病率最高的将仍是乙型肝炎。   C.在1998年至2010年之间,乙型肝炎患者的平均年龄将增大。   D.到2010年,乙型肝炎患者的数量将少于1998年。   E.到2010年,乙型肝炎的老年患者将多于非老年患者。   [解题分析]正确答案:C。   由题干,乙型肝炎在各个年龄段的发病率没有明显的不同;又,中国人口的平均年龄,在1998年至2010年之间,将呈明显上升态势。因此,一个显然恰当的推论是:在1998年至2010年之间,乙型肝炎患者的平均年龄将增大。这正是C项所断定的。其余各项均不从题干中恰当地推出。   -某研究所对该所上年度研究成果的统计显示:在该所所有的研究人员中,没有两个人发表的论文的数量完全相同;没有人恰好发表了10篇论文;没有人发表的论文的数量等于或超过全所研究人员的数量。   如果上述统计是真实的,则以下哪项断定也一定是真实的?   Ⅰ.该所研究人员中,有人上年度没有发表1篇论文。   Ⅱ.该所研究人员的数量,不少于3人。   Ⅲ.该所研究人员的数量,不多于10人。   A.只有Ⅰ和Ⅱ。   B.只有Ⅰ和Ⅲ。   C.只有Ⅰ。   D.Ⅰ、Ⅱ和Ⅲ。   E.Ⅰ、Ⅱ和Ⅲ都不一定是真实的。   [解题分析]正确答案:B。   题干的统计结论有三个:   结论一:没有两个人发表的论文的数量完全相同;   结论二:没有人恰好发表了10篇论文;   结论三:没有人发表的论文的数量等于或超过全所研究人员的数量。   设全所人员的数量为n,则由结论一和结论三,可推出:全所人员发表论文的数量必定分别为0,1,2,…,n-1。因此,选项Ⅰ成立。   又由结论二,可推出:该所研究人员的数量,不多于10人。否则,如果该所研究人员的数量多于10人,则有人发表的论文多于或等于10篇,则有人恰好发表了10篇论文,和结论二矛盾。因此,选项Ⅲ成立。   选项Ⅱ不成立。例如,如果研究人员的数量是2,其中一人未发表论文,另一个发表了一篇论文,题干的三个结论可同时满足。 -在国庆50周年仪仗队的训练营地,某连队一百多个战士在练习不同队形的转换。如果他们排成五列人数相等的横队,只剩下连长在队伍前面喊口令;如果他们排成七列这样的横队,只有连长仍然可以在前面领队;如果他们排成八例,就可以有两人作为领队了。在全营排练时,营长要求他们排成三列横队。 以下哪项是最可能出现的情况?   A.该连队官兵正好排成三列横队。   B.除了连长外,正好排成三列横队。   C.排成了整齐的三列横队,另有两人作为全营的领队。   D.排成了整齐的三列横队,其中有一人是其他连队的。   E.排成了三列横队,连长在队外喊口令,但营长临时排在队中。   [解题分析]正确答案:B。   设连队的人数是x。由题干,显然100   条件一:x除以5,余数是1。条件二:x除以7,余数是1。条件三:x除以8,余数是2。   5和7的公倍数,满足大于100且小于200的,有105,140和175。因此,同时满足条件一和条件二的x的取值,可以是106,141或176,在这三个数字中,可以满足条件三的只有x取值106。因此,同时满足三个条件的x的唯一取值是106。   这样,A项不能成立,因为106不能被3整除。B项能成立,因为106除以3,余数是1。C项不成立,因为106除以3,余数不是2。D项不成立,因为(106+1),不能被3整除。E项不成立,因为(106-1+1),不能被3整除。   -据世界卫生组织1995年的调查报告显示,70%的肺癌患者有吸烟史,其中有80%的人吸烟的历史多于10年。这说明吸烟会增加人们患肺癌的危险。   以下哪项最能支持上述论断?   A. 1950年至1970年期间男性吸烟者人数增加较快,女性吸烟者也有增加。   B.虽然各国对吸烟有害进行大力宣传,但自50年代以来,吸烟者所占的比例还是呈明显的逐年上升的趋势。到90年代,成人吸烟者达到成人数的50%。   C.没有吸烟史或戒烟时间超过五年的人数在1995年超过了人口总数的40%。   D. 1995年未成年吸烟者的人数也在增加,成为一个令人挠头的社会问题。   E.医学科研工作者已经用动物实验发现了尼古丁的致癌作用,并从事开发预防药物的研究。   [解题分析]正确答案:B。   自50年代以来,吸烟者所占的比例还是呈明显的逐年上升趋势。到90年代,成人吸烟者达到成人数的50%,说明在50到90年代中,成人吸烟者少于成人数的50%。1995年的调查报告中提到有吸烟史的肺癌患者中有80%的人吸烟的历史多于10年,那时是1985年以前。当时吸烟者不足成人的50%,而后来发现的有吸烟史的肺癌患者占肺癌患者总数的56%(70%×80%),明显高于50%。说明吸烟会增加人们患肺癌的危险。   选项A不妥,它只说吸烟者人数增加快,没有讲占成人总数的比例,难以成为有力的论据。选项C讲的是在1990年时不吸烟的人数超过了40%,吸烟人数少于60%,但对1985年时的情况没有涉及,对题干中关于10年吸烟史的人员的判断不能提供任何支持。选项D不对。D只是揭示了一个令人挠头的社会问题,但与题干的结论无关。应该注意到吸烟者比例越高,越不能支持题干的论点。比如,80%的成人吸烟,患肺癌的人中,吸烟者占80%,说明吸烟不增加患肺癌的危险。选E看来有一定道理,但用动物(最通常的理解是除人以外的动物)实验毕竟只能作为参考,不能直接将结论推广到人。我们可以与选项B比较,B是建立在统计推断的理论之上的,更能支持题干的论断。   -东升商城公关部职工的平均工资是营业部职工的2倍,因此,公关部职工比营业部职工普遍有较高的收入。   以下哪项如果是真的,将最能削弱上述论证?   A.公关部职工的人均周实际工作时数要超过营业部职工的50%。   B.按可比因素计算,公关部职工为商城创造的人均价值是营业部职工的近10倍。   C.公关部职工中最高工资与最低工资间的差别要远大于营业部职工。   D.公关部职工的人数只是营业部职工的10%。   E.公关部职工中有20%享受商城的特殊津贴,营业部职工中则有25%享受此种津贴。   [解题分析]正确答案:C。   这道题有个小陷阱。公关部职工的平均工资是营业部职工的2倍,能不能得出,公关部职工比营业部职工普遍有较高的收入呢?   选项A、B似乎都是在讨论这种分配差异合理不合理的问题,不选。选项D讲人数,这影响不了平均工资和普遍收入水平的关系,不选。其实选项E是削弱题干的,而且是一种很好的思路,那就是告诉你:工资只是收入中的一部分。如果选项说“工资只是收入中很小的一部分”(比如公关部职工灰色收入特别多之类的),那就应该选。这也是经常考察的一个考点,但这次出题的人放过了它,还设了20%和25%这么小的差距来含混它。   选项C是个算术题。一种极端的情况是公关部有几个职工工资特别高,剩下的职工的工资普遍比营业部绝大部分职工的工资还低,但是平均下来,公关部的平均工资还是营业部的2倍。这是可能的,我们觉得也是出题人设下的埋伏。所以选C。   (三)二人对话与辩论   二人对话与辩论题型是对前面所述的假设、支持、削弱、归纳等逻辑推理题型的综合运用,因此是相对较难的题。解题的关键是:一要抓住对话双方意思的差异,二要注意对话或论辩双方的语气,从而明确问题的方向。为有助于对二人对话或辩论类题型的理解,这里补充介绍一下对话中所涉及的“预设”方面的知识。预设有语义预设和语用预设之分,语义预设是一个命题及其否定都要假定的东西,是一个命题能够为真或为假的前提条件;语用预设则可以表述为,如果话语A只有当命题B为交谈双方所共知时才是恰当的,则A在语用上预设B。由于日常交际中的推理都是具有某些共同背景知识的人在特定的语境中进行的,不必列出所有必需的前提,但要注意的是这种省略本身可能不是真的或这种省略推理中可能暗含着逻辑错误。因此,在批判性思维中,常常需要把这些被省略的前提、假定、预设补充到推理过程中来,以便考察被省略的前提是否真实,推理过程是否正确,即对推理者的推理进行评价。同时,由于这种省略形式的前提对结论提供了不充分的支持,有时候需要加强前提以便对结论提供更强的支持,或对该论证提供更好的辩护。被补充到前提中去的,可以是某个一般原理如因果关系陈述,也可以是某个假设、假定或事实性断言。对前提的加强可以到使该推理成为形式有效的推理的地步,但更多的时候只是提高了推理中前提对结论的证据支持度。   -史密斯:根据《国际珍稀动物保护条例》的规定,杂种动物不属于该条例的保护对象。《国际珍稀动物保护条例》的保护对象中,包括赤狼。而最新的基因研究技术发现,一直被认为是存种物种的赤狼实际上是山狗与灰狼的杂交来重新获得它。由于赤狼明显需要保护,所以条例应当修改,使其也保护杂种动物。   张大中:您的观点不能成立。因为,如果赤狼确实是山狗与灰狼的杂交种的话,那么,即使现有的赤狼灭绝了,仍然可以通过山狗与灰狼的杂交来重新获得。   以下哪项最为确切地概括了张大忠与史密斯争论的焦点?   A.赤狼是否为山狗与灰狼的杂交种。   B.国际珍稀动物保护条例的保护对象中,是否应当包括赤狼。 C.国际珍稀动物保护条例的保护对象中,是否应当包括杂种动物。 D.山狗与灰狼是否都是纯种物种。   E.目前赤狼是否有灭绝的危险。   [解题分析]正确答案:C。   史密斯的观点是:《国际珍稀动物保护条例》的保护对象中,应当包括杂种动物。其根据是:《国际珍稀动物保护条例》的保护对象中,包括赤狼。赤狼是杂种动物。既然赤狼明显需要保护,所以一般地,杂种动物需要保护。   张大中的观点是:《国际珍稀动物保护条例》的保护对象中,不应当包括杂种动物。其根据是:如果某种杂交动物物种灭绝的话,可以通过动物的杂交来重新获得它。   因此,两人争论的焦点是:《国际珍稀动物保护条件》的保护对象中,是否应当包括杂种动物。   -本题题干同上题   以下哪项最可能是张大中的反驳所假设的?   A.目前用于鉴别某种动物是否为杂种的技术是可靠的。   B.所有现存杂种动物都是现存纯种动物杂交的后代。   C.山狗与灰狼都是纯种物种。   D.国际珍稀动物保护条例执行效果良好。   E.赤狼并不是山狗与灰狼的杂交种。   [解题分析]正确答案:B。   B项是张大中的反驳所需要假设的,否则,如果有的杂交动物不是现存纯种动物杂交的后代,那么,此种杂交动物物种一旦灭绝,就不能通过杂交来重新获得它,张大中反驳的根据就不能成立。   其余各项均不是需要假设的。   -张教授:智人是一种早期人种。最近在百万前的智人遗址发现了烧焦的羚羊骨头碎片的化石。这说明人类在自己进化的早期就已经知道用火来烧肉了。   李研究员:但是在同样的地方也同时发现了被烧焦的智人骨头碎片的化石。   以下哪项最可能是李研究员所要说明的?   A.百万年前森林大火的发生概率要远高于现代。   B.百万年前的智人不可能掌握取火用火的技能。   C.上述被发现的智人骨头不是被人控制的火烧焦的。   D.羚羊并不是智人所喜欢的食物。   E.研究智人的正确依据,是考古学的发现,而不是后人的推测。   [解题分析]正确答案:C。   张教授的结论是:人类在自己进化的早期就已经知道用火来烧肉了,其根据是:在百万年前的智人遗址发现了烧焦的羚羊骨头碎片的化石。   李研究员对此质疑的根据是:在同样的地方也同时发现了被烧焦的智人骨头碎片的化石,据此显然是要说明:上述羚羊的骨头不是被人控制的火烧焦的。   -本题题干同上题   以下哪项最可能是李研究员的议论所假设的?   A.包括人在内的所有动物,一般不以自己的同类为食。   B.即使在发展的早期,人类也不会以自己的同类为食。   C.上述被发现的智人骨头碎片的化石不少于羚羊骨头碎片的化石。   D.张教授并没有掌握关于智人研究的所有考古资料。   E.智人的主要食物是动物而不是植物。   [解题分析]正确答案:B。   B项是李研究员的议论所必须假设的,否则,如果在发展的早期,人类以自己的同类为食,那么,上述被烧焦的智人骨头碎片完全可能是人为火烧的结果,这样,李研究员的质疑就失去了根据。   A项的断定中包含对B项的断定,但过强了,不是李研究员的议论所必须假设的。   其余各项显然不是需要假设的。   -李工程师:一项权威性的调查数据显示,在医疗技术和设施最先进的美国,婴儿最低死亡率在世界上只占17位。这使我得出结论,先进的医疗技术和设施,对于人类生命和健康所起的保护作用,对成人要比对婴儿显著得多。   张研究员:我不能同意您的论证。事实上,一个国家所具有的先进的医疗技术和设施,并不是每个人都能均等地享受的。较之医疗技术和设施而言,较高的婴儿死亡率更可能是低收入的结果。   以下哪项最为恰当地概括了张研究员反驳李工程师的使用的方法?   A.对他的论据的真实性提出质疑。   B.对他的结论的真实性提出质疑。   C.对他援引的数据提出另一种解释。   D.暗指他的数据会导致产生一个相反的结论。   E.指出他偷换了一个关键的概念。   [解题分析]正确答案:C。   张研究员的反驳是基于李工程师援引的数据之上的,这说明他并没有对李工程师的论据提出质疑。因此,A不成立。   李工程师的结论是:先进的医疗技术和设施,对于人类生命和健康所起的保护作用,对成人要比对婴儿显著得多。张研究员在反驳中并没有对这一结论提出质疑,而只是指出,一个国家所具有的先进的医疗技术和设施,并不是每个人都能均等地享受的。因此,B不成立。 与李工程师所论证的相反的结论是,先进的医疗技术和设施,对于人类生命和健康所起的保护作用,对婴儿要比对成人显著得多。张研究员显然并不认为根据李工程师援引的数据会导致这一结论。因此,D不成立。   张研究员的反驳没有涉及李工程师的论证中是否存在偷换概念的问题,因此,E不成立。   张研究员的反驳是对李工程师援引的数据提出了另一种解释,即低收入使得一些美国的穷人难以让他们的婴儿享受先进的医疗而导致了较高的婴儿死亡率。因此,B成立。   -本题题干同上题   张研究员的反驳基于以一哪项假设?   Ⅰ在美国,享受先进的医疗技术和设施,需要一定的经济条件。   Ⅱ在美国,存在着明显的贫富差别。   Ⅲ在美国,先进的医疗技术和设施,主要用于成人的保健和治疗。   A.只有Ⅰ。   B.只有Ⅱ。   C.只有Ⅲ。   D.只有Ⅰ和Ⅱ。   E.Ⅰ、Ⅱ和Ⅲ。   [解题分析]正确答案:D。   Ⅰ和Ⅱ是必须假设的。否则,如果美国不存在明显的贫富差别,或者虽然存在贫富差别,但享受先进的医疗作为一种社会福利是免费或基本免费提供的,那么,张研究员关于低收入使得一些美国的穷人难以让他们的婴儿享受先进的医疗技术和设施而导致较高的婴儿死亡率的解释就不能成立。   Ⅲ不但不是必须假设的,而且是不能假设的,否则,如果事实上美国先进的医疗技术和设施,主要用于成人而不是婴儿的保健和治疗,那么,这本身就成为美国婴儿死亡率相对较高的一个有说服力的解释,将对张研究员的解释构成有力的质疑。   -本题题干同上题   以一哪项如果为真,能最有力地削弱张研究员的反驳?   A.美国的人均寿命占世界第二。   B.全世界的百岁老人中,美国人占了30%。   C.美国的婴儿死亡率呈下降趋势。   D.美国用于医疗新技术开发的投资,占世界之最。   E.一般地说,拯救婴儿免于死亡的医疗要求高于成人。   [解题分析]正确答案:A。   如果张研究员的解释是成立的,那么,被迫无法享受先进医疗的,就会不仅是贫穷的婴儿,而且包括贫穷的成人。这样,不仅婴儿死亡率会相对较高,成人死亡率也会同样如此,这又不可避免地会使得美国人均寿命在世界上变得相对较低。而A断定美国的人均寿命居世界第二,这就对张研究员的解释提出了有力的质疑。   选项B如果为真,能支持李工程师的结论,但是否能削弱张研究员的反驳,还得取决于美国百岁老人中低收入者所占的比例,如果这种比例非常低,那么,这不但没有削弱,反而加强了张研究员的反驳。   选项C、D和E不能削弱张研究员的反驳,其中,E能加强这种反驳。   -李工程师:在日本,肺癌病人的平均生存年限(即从确诊至死亡的年限)是9年,而在亚洲的其他国家,肺癌病人的平均生存年限只有4年。因此,日本在延长肺癌病人生命方面的医疗水平要高于亚洲的其他国家。   张研究员:你的论证缺乏充分的说服力。因为日本人的自我保健意识总体上高于其他的亚洲人,因此,日本肺癌患者的早斯确诊率要高于亚洲其他国家。   张研究员的反驳,基于以下哪项假设?   Ⅰ.肺癌患者的自我保健意识对于其疾病的早期确诊起到重要作用。   Ⅱ.肺癌的早期确诊对延长患者的生存年限起到重要作用。   Ⅲ.对肺癌的早期确诊技术是衡量防治肺癌医疗水平的一个重要方面。   A.只有Ⅰ。   B.只有Ⅱ。   C.只有Ⅲ。   D.只有Ⅰ和Ⅱ。   E.Ⅰ、Ⅱ和Ⅲ。   [解题分析]正确答案:D。   由题干,李工程师认为:日本肺癌病人平均生存年限较长的原因,是日本的医疗水平较高。张研究员则认为:日本肺癌病人平均生存年限较长的原因,是日本肺癌患者的早期确诊率较高;而日本肺癌患者的早期确诊率较高的原因,是日本人的自我保健意识较高。显然,为使张研究员的反驳成立,选项Ⅰ和选项Ⅱ是必须假设的,否则,张研究员断定的两个因果关系就不能成立。选项Ⅲ不是必须假设的。   -本题题干同上题   以下哪项如果为真,能最为有力地指出李工程师论证中的漏洞?   A.亚洲一些发展中国家的肺癌患者是死于由肺癌引起的并发症。   B.日本人的平均寿命不仅居亚洲之首,而且居世界之首。   C.日本的胰腺癌病人的平均生存年限是5年,接近于亚洲的平均水平。   D.日本医疗技术的发展,很大程度上得益于对中医的研究和引进。   E.一个数大大高于某些数的平均数,不意味着这个数高于这些数中的每个数。   [解题分 析]正确答案:E。 李工程师的论证实际上包含了两个推理。第一个推理是根据肺癌病人在日本的平均生存年限高于在亚洲其它国家的平均生存年限,推出肺癌病人在日本的平均生存年限高于在亚洲其他(任一)国家的平均生存年限;第二个推理是根据肺癌病人在日本的平均生存年限高于在亚洲其他(任一)国家的平均生存年限,推出日本在延长肺癌病人生命方面的医疗水平要高于亚洲的其他(任一)国家。   E项指出了第一个推理中存在的漏洞。事实上,正如E项所指出的,虽然肺癌病人在日本的平均生存年限高于在亚洲其他国家的平均生存年限,但完全可能有某个或某些亚洲国家,它的肺癌病人的平均生存年限高于日本。因此,李工程师的第一个推理是不成立的,因而它的结论也是不可靠的。其余各项均不能说明李工程师的论证中存在漏洞。 第四章 逻辑解题套路精析(八) http://www.sina.com.cn 2003/10/29 08:35 新浪教育   八、逻辑分析   MBA联考逻辑考试的主流题型与国外考试的逻辑推理题(Logical Reasoning)类似,然而也有少量考题与国外考试的逻辑分析题(Analytical Reasoning)在分析思路上有些相似,但题型却有较大差别,因为国外的逻辑分析题通常所说的是组题、串题,同一题干一般有3—4个题,而我们这里所说的MBA联考的逻辑分析题基本上都是单题,即一个题干一般只有1个 题,而且整个解题思路一般也没有国外考试那么复杂。   对于推理步骤较多,花费时间较长的逻辑分析难题,虽然对训练思维也很有好处,但由于正式考试的时间限制,在MB联考逻辑考试中则不会出现。不象逻辑推理题,目前MBA的逻辑分析题均不是来源于国外考题,而是国内自行命制的。逻辑分析题偏重于考察考生的缜密严峻的推理以及对具体事物的抽象能力,解分析题从宏观角度是对大局的认识,框架的认识;从微观角度是对每个条件的具体使用方法的灵活运用。   逻辑分析题从推理思路上也属于归纳型,即“自上而下推理”,其解题关键是要“把条件用尽”,即对于题目所给出的规则,必须边读题边把题目所给出的条件一条条在草稿纸上逐一列出,同时要善于分析隐含条件。解这类考题最好能借助于一些技巧,比如列个表或画个图,有时需要借助于归谬法。   (一)排序   排序是最简单的逻辑分析题,该题型一般在题干部分给出不同对象之间的若干个两两对比的结果,要求从中推出具体的排序。解这类题型的主要思路是要把所给条件抽象成最简单的排序形式。   -甘蓝比菠菜更有营养。但是,因为绿芥兰比莴苣更有营养,所以甘蓝比莴苣更有营养。   以下各项,作为新的前提分别加入到题干的前提中,都能使题干的推理成立,除了   A.甘蓝与绿芥兰同样有营养。   B.菠菜比莴苣更有营养。   C.菠菜比绿芥兰更有营养。   D.菠菜与绿芥兰同样有营养。   E.绿芥兰比甘蓝更有营养。   [解题分析]正确答案:E。   E项断定:绿芥兰比甘蓝更有营养。题干断定:绿芥兰比莴苣更有营养。由这两个断定显然不能推出“甘蓝比莴苣更有营养”。   其余各项,作为新的前提分别加入到题干的前提中,都能使题干的推理成立。   -去年MBA入学考试的五门课程中,王海天和李素云只有数学成绩相同,其他科的成绩互有高低,但所有课程的分数都在60分以上。在录取时只能比较他们的总成绩了。   下列哪项如果为真,能够使你判断出王海天的总成绩高于李素云?   A.王海天的最低分是数学,而李素云的最低分是英语。   B.王海天的最高分比李素云的最高分要高。   C.王海天的最低分比李素云的最低分高。   D.王海天的最低分比李素云的两门课分别的成绩高。   E.王海天的最低分比李素云的平均成绩高。   [解题分析]正确答案:E。   因为王海天总成绩不会小于最低分的五倍,而李素云的总成绩正好是平均成绩的五倍,所以“王海天的最低分比李素云的平均成绩高”意味着“王海天的总成绩比李素云的总成绩高”。答案A不能提供有用的信息,答案B、C、D提供的信息不充分。   -有四个外表看起来没有分别的小球,它们的重量可能有所不同。取一个天平,将甲、乙归为一组,丙、丁归为另一组分别放在天平的两边,天平是基本平衡的。将乙和丁对调一下,甲、丁一边明显地要比乙、丙一边重得多。可奇怪的是,我们在天平一边放上甲、丙,而另一边刚放上乙,还没有来得及放上丁时,天平就压向了乙一边。   请你判断,这四个球中由重到轻的顺序是什么?   A.丁、乙、甲、丙。   B.丁、乙、丙、甲。   C.乙、丙、丁、甲。   D.乙、甲、丁、丙。   E.乙、丁、甲、丙。   [解题分析]正确答案:A。   确立传递关系。可用不等式推导,令甲为a,乙为b,丙为c,丁为d。根据题意可知:   a + b=c + d (1) b + c (2)>  a + d a + c (3)>  b c>c + d + b + c即a>  由(1)+(2)可得:a + b + a + d b>a + b + b + c即d>  同时(1)+(2)还可得:c + d + a + d a>a + c可得:b>  又由(3)b c即四个球由重到轻的顺序是丁、乙、甲、丙>a>b>  综合推出d   (二)集合   集合题型的一般特点是:在题目中出现“所有”、“有些”、“某个”、“每一个”、“没有一个”等集合型的叙述或题干提供的概念间的范围有重合的部分。可以根据基本的集合概念和逻辑常识解决该类题型,解这种题型的重点放在集合的“部分与全体”上,同时要善于分辨可能重合的部分和绝不会重合的部分。最直观的办法是根据题干提供的条件画个小图,题目即可迎刃而解。   -所有切实关心教员福利的校长,都被证明是管理得法的校长;而切实关心教员福利的校长,都首先把注意力放在解决中青年教员的住房上。因此,那些不首先把注意力放在解决中青年教员住房上的校长,都不是管理得法的校长。   为使上述论证成立,以下哪项必须为真?   A.中青年教员的住房问题,是教员的福利中最为突出的问题。   B.所有管理得法的校长,都是关心教员福利的校长。   C.中青年教员的比例,近年来普遍有了大的增长。   D.所有首先把注意力放在解决中青年教员住房上的校长,都是管理得法的校长。   E.老年教员普遍对自己的住房善比较满意。   [解题分析]正确答案:B。   题干只断定:所有关心教员福利的校长,都是管理得法的校长。由此推不出:所有管理得法的校长,都是关心教员福利的校长。为使题干的论证成长,B项必须为真,否则,如果有校长管理得法,但是却不关心教员福利,那么,他完全可能不首先把注意力放在解决中青年教员住房上。这样,题干的结论就不能成立。   -所有持有当代商厦购物优惠卡的顾客,同时持有双安商厦的购物优惠卡。今年国庆,当代商厦和双安商厦同时给持有本商厦的购物优惠卡的顾客的半数,赠送了价值100元的购物奖卷。结果,上述同时持有两个商厦的购物优惠的顾客,都收到了这样的购物奖卷。   如果上述断定是真的,则以下哪项断定也一定为真?   Ⅰ.所有持有双安商厦的购物优惠卡的顾客,也同时持有当代商厦的购物优惠卡。   Ⅱ.今年国庆,没有一个持有上述购物优惠卡的顾客分别收到两个商厦的购物奖卷。   Ⅲ.持有双安商厦的购物优惠卡的顾客中,至多有一半收到当代商厦的购物奖卷。   A.只有Ⅰ。   B.只有Ⅱ。   C.只有Ⅲ。   D.只有Ⅰ和Ⅱ。   E.Ⅰ、Ⅱ和Ⅲ。   [解题分析]正确答案:C。   由题干,所有持有当代商厦购物优惠卡的顾客,同时持有双安商厦的购物优惠卡。这说明,持有双安优惠卡的顾客人数不会少于持有当代优惠卡的顾客人数。如果持有双安优惠卡的顾客中,有超过一半的人收到当代的购物奖卷,这说明收到当代购物奖卷的人数,超过了持有当代优惠卡顾客人数的半数,这和题干的条件矛盾,因此,Ⅲ的断定一定为真。Ⅰ和Ⅱ都不一定是真的。这二者的关系是,如果Ⅰ是真的,则Ⅱ是真的;因为Ⅰ不一定是真的,所以Ⅱ不一定是真的,即二者都不一定是真的。   -某大学一寝室中住着若干个学生。其中,一个是哈尔滨,两个是北方人,一个是广东人,两个在法律系,三个是进修生。该寝室中恰好住了8个人。   如果题干中关于身份的介绍涉及了寝室中所有的人,则以下各项关于该寝室的断定都不与题干矛盾,除了   A.该校法律系每年都招收进修生。   B.该校法律系从未招收过进修生。   C.来自广东的室友在法律系就读。   D.来自哈尔滨的室友在财政金融系就读。   E.该室的三个进修生都是南方人。 [解题分析]正确答案:C。 这类题目就是考察你对概念的外延是否交叉和包含的判断。要做对本题,就要理解,哈尔滨人是北方人;所以,假设其他概念不交叉,那么一共是8个人,因此,其他概念必须不能“交叉”。选项C中,广东人和法律系交叉了,这样,总人数就少于8人了,因此,与题干矛盾。   选项A、B、D、E都没有构成新的概念交叉,因此不与题干矛盾。   -本问题发生在一所学校内。   学校的教授们中有一些是足球迷。   学校的预算委员会的成员们一致要把学校的足球场改建为一个科贸写字楼,以改善学校收入状况。   所有的足球迷都反对将学校的足球场改建成科贸写字楼。   如果以上各句陈述均为真,则下列哪项也必为真?   A.学校所有的教授都是学校预算委员会的成员。   B.学校有的教授不是学校预算委员会的成员。   C.学校预算委员会有的成员是足球迷。   D.并不是所有的学校预算委员会的成员都是学校的教授。   E.有的足球迷是学校预算委员会的成员。   [解题分析]正确答案:B。   B项必为真。因为所有的足球迷都反对改建足球场,而所有的预算委员会的成员都主张改建足球场,因此,所有的预算委员会的成员都不是足球迷。又有的教授是足球迷。因此有的教授不是预算委员会的成员。其余各项均不必定为真。   -如果作为上题陈述的补充,明确以下条件:所有的学校教授都是足球迷,那么下列哪项一定不可能是真的?   A.有的学校教授不是学校预算委员会的成员。   B.学校预算委员会的成员中有的是学校教授。   C.并不是所有的足球迷都是学校教授。   D.所有的学校教授都反对将学校的足球场改建为科贸写字楼。   E.有的足球迷不是学校预算委员会的成员。   [解题分析]正确答案:B。   B项不可能真。因为所有的教授都是足球迷,因此所有的教授都反对改建足球场;而所有的预算委员会的成员都主张改建足球场。因此不可能有预算委员会的成员是教授。   (三)匹配   匹配题型的一般特点是:题干一般提供几类因素,每类因素又有几种不同情况,同时题干还给出属于不同类因素之间不同情况的判断,要求推出确定的结论。有的考生特别害怕这种匹配类型的题目,其实只要细心得法,这类题目并不难。解这类考题时,所要使用的推理形式和推理步骤较多,推理过程显得相对复杂。解题基本思路是,通过对题干给出的多种因素间的关系进行分析推理和排列组合,弄清题干中所给条件的内在关系,从一个一个条件出发,通过逻辑运算与推理,直至推出正确答案。   匹配型考题的主要解题方法有:一是,假设反证法,耐心点推是个笨办法,但绝对是个好办法;二是,表格法,把已知条件划在一个表格上,再进一步推理。   -三位高中生赵、钱、孙和三位初中生张、王、李参加一个课外学习小组。可选修的课程有:文学、经济、历史和物理。   赵选修的是文学或经济。   王选修物理。   如果一门课程没有任何一个高中生选修,那么任何一个初中生也不能选修该课程;如果一门课程没有任何初中生选修,那么任何一个高中生也不能选修该课程;一个学生只能选修一门课程。   如果上述断定为真,且钱选修历史,以下哪项一定为真?   A.孙选修物理。   B.赵选修文学。   C.张选修经济。   D.李选修历史。   E.赵选修经济。   [解题分析]正确答案:A。   由题干,如果有一个初中生选修某门课程,那么就有一个高中生也选修该课程;反之亦然。   已知初中生王选修物理,所以有一个高中生也选修物理,即越、钱或孙选修物理。   又因为一个学生只能修一门课程,已知钱选修历史,所以钱不选修物理;赵选修文学或经济,所以赵不选修物理。因此,可推出孙选修物理。   -如果上题题干的断定为真,且有人选修经济,则选修经济的学生中不可能同时包含   A.赵和钱。   B.钱和孙。   C.孙和张。   D.孙和李。   E.张和李。   [解题分析]正确答案:B。   因为赵、钱或孙中至少有一人选修物理,又已知赵选修文学或经济所以钱或孙中至少有一人选修物理,不可能同时选修经济。   -方宁、王宜和余涌,一个是江西人,一个是安徽人,一个是上海人,余涌的年龄比上海人大,方宁和安徽人不同岁,安徽人比王宜年龄小。   根据上述断定,以下结论都不可能推出,除了 方宁是江西人,王宜是安徽人,余涌是上海人。  B.方宁是安徽人,王宜是江西人,余涌是上海人。   C.方宁是安徽人,王宜是上海人,余涌是江西人。   D.方宁是上海人,王宜是江西人,余涌是安徽人。   E.方宁是江西人,王宜是上海人,余涌是安徽人。   [解题分析]正确答案:D。   容易检验当D为真时,与题干的论述不矛盾。同时,我们也可以从题干推出D。由“余涌的年龄比上海人大”与“安徽人比王宜年龄小”,可以得出江西人最大;由“方宁和安徽人不同岁,安徽人比王宜年龄小”可知余涌是安徽人;由“余涌的年龄比上海人大”可知上海人最小;由“安徽人比王宜年龄小”可知王宜是江西人。剩下没有判定的方宁只能是上海人。与选项D完全一致。既然选项D是可以从题干严格推理得出的,其他选项与D又不同,所以其他选择都是错的。   列个表更直观些:王宜>余涌> (方宁)(江西)安徽上海   -某宿舍住着四个研究生,分别是四川人、安徽人、河北人和北京人。他们分别在中文、国政和法律三个系就学。其中:   Ⅰ.北京籍研究生单独在国政系。   Ⅱ.河北籍研究生不在中文系。   Ⅲ.四川籍研究生和另外某个研究生同在一个系。   Ⅳ.安徽籍研究生不和四川籍研究生同在一个系。   以上条件可以推出四川籍研究生所在的系为哪个系?   A.中文系。   B.国政系。   C.法律系。   D.中文系或法律系。   E.无法确定。   [解题分析]正确答案:C。   我们来使用假设反证法。要推四川籍研究生所在的系,我们来依次假设。设四川=中文。则还有一个研究生也在中文,是谁呢?不是河北籍(河北籍研究生不在中文系),也不是安徽籍(安徽籍研究生不和四川籍研究生同在一个系),那就是北京籍。而这与“北京籍研究生单独在国政系”矛盾。设四川=国政。则与“北京籍研究生单独在国政系”矛盾。   设四川=法律。则还有一个研究生也在法律。是谁呢?不是安徽籍,不是北京籍,那就是河北籍。安徽籍在中文系,北京籍在国政系。   (四)综合   综合型试题是相对较难的逻辑题,通常在题干中给出若干条件,要求考生从这些条件中合乎逻辑推出某种结论。解这类题一般一下子看不出答案,需进行深入分析和推理。其解题基本思路是,做题时首先要把所有条件列出,再从条件的逻辑关系或事物的内部联系出发,逐步综合进行推理,注意一定要把所有条件彻底用完。   -甲乙丙三人一起参加了物理和化学两门考试。三个人中,只有一个在考试中发挥正常。   考试前,甲说:   如果我在考试中发挥不正常,我将不能通过物理考试。   如果我在考试中发挥正常,我将能通过化学考试。   乙说:   如果我在考试中发挥不正常,我将不能通过化学考试。   如果我在考试中发挥正常,我将能通过物理考试。   丙说:   如果我在考试中发挥不正常,我将不能通过物理考试。   如果我在考试中发挥正常,我将能通过物理考试。   考试结束后,证明这三个人说的都是真话,并且:   发挥正常的人是三人中惟一的一个通过这两门科目中某门考试的人;   发挥正常的人也是三人中惟一的一个没有通过另一门考试的人。   从上述断定能推出以下哪项结论?   A.甲是发挥正常的人。   B.乙是发挥正常的人。   C.丙是发挥正常的人。   D.题干中缺乏足够的条件来确定谁是发挥正常的人。   E.题干中包含互相矛盾的信息。   [解题分析]正确答案:B。   如果甲是发挥正常的人,则根据甲说的话,可知甲通过了化学考试,但没有能过物理考试;(注意,甲发挥正常但没有通过物理考试,和他所说的“如果我在考试中发挥不正常,我将不能通过物理考试”的话并不矛盾,因为从这句话不能得出“如果我在考试中发挥正常,我将能通过物理考试”)。又由条件,可知甲是三人中惟一的没有通过物理考试的人;又根据丙说的话,可知丙因为发挥不正常,也没有通过物理考试。这样,就有两个人没有通过物理考试。矛盾!因此,甲不是发挥正常的人。   如果丙是发挥正常的人,则根据丙说的话,可知丙通过了物理考试,又由条件,可知丙没有通过化学考试,并且是三人中惟一的没有通过化学考试的人;又根据乙说的话,可知乙因为发挥不正常,也没有通过化学考试。这样,就有两个人没有通过化学考试。矛盾!因此,丙不是发挥正常的人。如果乙是发挥正常的人,则可知乙通过了物理考试,但没有通过化学考试。又根据甲和丙说的话,可知甲和丙都没有通过物理考试。又由条件,可推出甲和丙都通过了化学考试。这里没有任何矛盾。因此,答案是B。 -全国运动会举行女子5000米比赛,辽宁、山东、河北各派了三名运动员参加。比赛前,四名体育爱好者在一起预测比赛结果。甲说:“辽宁队训练就是有一套,这次的前三名非他们莫属。”乙说:“今年与去年可不同了,金银铜牌辽宁队顶多拿一个。”丙说:“据我估计,山东队或者河北队会拿牌的。”丁说:“第一名如果不是辽宁队的,就该是山东队的了。”比赛结束后,发现以上四人只有一人言中。 以下哪项最可能是该项比赛的结果?   A.第一名辽宁队,第二名辽宁队,第三名辽宁队。   B.第一名辽宁队,第二名河北队,第三名山东队。   C.第一名山东队,第二名辽宁队,第三名河北队。   D.第一名河北队,第二名辽宁队,第三名辽宁队。   E.第一名河北队,第二名辽宁队,第三名山东队。   [解题分析]正确答案:D   用假设反证法。   若A成立,则甲真,乙假,丙假,丁真;   若B成立,则甲假,乙真,丙真,丁真;   若C成立,则甲真,乙真,丙真,丁真;   若D成立,则甲假,乙假,丙真,丁假;   若E成立,则甲假,乙真,丙真,丁假。   由于四人中只有一人言中,故只能选D项。   (注意本题不能用矛盾法做,甲、丙的话并不矛盾,因为这两句话可以同假。即甲真能推出丙假,但甲假推不出丙真。因为甲假说明有别的队拿牌,而题干并没讲只有辽宁、山东、河北三个队参加女子5000米比赛,因此,并不一定能推出丙真。   同时用假设反证法时,应优先对选项进行假设反证,因为只要能确定其他选项都与题干的条件矛盾,省下一项没有矛盾,该选项就是正确答案。做题的目的是选出答案,并不一定要推出具体结果,因为有时根据题干条件不一定能推出唯一结果的,当然没有唯一结果并不等于没有正确答案) 第四章 逻辑解题套路精析(九) http://www.sina.com.cn 2003/10/30 08:30 新浪教育   九、逻辑应用   逻辑应用类题型具有明显的形式逻辑学的特征,需要直接或间接用到一些逻辑知识。大纲中所述的“熟悉一些逻辑学的基础知识,掌握一些逻辑学的基本方法,有助于考生迅速准确地解题”对这类逻辑应用型考题最适用。   (一)对当关系   直言判断及模态命题的对当关系是最基本的一个逻辑知识点。这类试题的表现形式可以多种多样,但近几年在MBA逻辑考试中出现的次数有减少的趋势。解这类题型,关键是要从题干给出的内容出发,从中抽象出同属于对当关系的逻辑形式,根据对当关系来分析判断。   -通过调查得知,并非所有的个体商贩都有偷税、逃税行为。   如果上述调查的结论是真实的,则以下哪项一定为真:   A.所有的个体商贩都没有偷税、逃税行为。   B.多数个体商贩都有偷税、逃税行为。   C.并非有的个体商贩没有偷税、逃税行为。   D.并非有的个体商贩有偷税、逃税行为。   E.有的个体商贩确实没有偷税、逃税行为。   [解题分析]正确答案:E。   题干的意思是所有的个体商贩都有偷税、逃税行为是假命题,而不是说所有的个体商贩都没有偷税、逃税行为。不能选A。题干的意思是有的个体商贩有偷税、逃税行为,多数还是少数并不知道,B不一定为真。C和D它们对两个真命题进行了否定。不能选C和D。只有E,肯定了有的个体商贩没有偷税、逃税行为,与题干相符,一定为真。   -不可能所有的错误都能避免。   以下哪项最接近上述断定的含义?   A.所有的错误必然都不能避免。   B.所有的错误可能都不能避免。   C.有的错误可能不能避免。   D.有的错误必然能避免。   E.有的错误必然不能避免。   [解题分析]正确答案:E。   不可能所有的错误都能避免。不意味着所有的错误都不能避免,只是强调一定会有反例存在,即有的错误必然不能避免。   (二)复合命题推理   逻辑的复合命题推理主要涉及对假言、联言和选言及负命题推理的综合运用。特别是充分条件和必要条件的区分与运用以及命题间的推理关系,是逻辑考试中一个必考也是常考的知识点。这里,再归纳一下这两个知识点,要求考生能够熟练理解与掌握。   1、充分条件与必要条件   (1)充分条件:所谓充分条件就是仅有这条件就足以带来结果,无需考虑别的条件了。它是谁成立,谁一定也成立,比如A→B,如果A成立,那么一定有B,则A是B的充分条件。 (2)必要条件:所谓必要条件就是没有这个条件,结论一定不对。比如B→A成立,那么A是B成立的必要条件。   2、命题间的推理关系   (1)若A→B为原命题,则:B→A为逆命题;非A→非B为否命题;非B→非A为逆否命题。原命题和逆否命题为等价命题,逆命题和否命题为等价命题。   (2)或命题和与命题   或命题:B1或B2表达为B1+B2   与命题:B1与B2表达为B1*B2   A→B1+B2的逆否命题为非B1*非B2→非A   A→B1*B2的逆否命题为非B1+非B2→非A   B1+B2→A的逆否命题为非A→非B1*非B2   B1*B2→A的逆否命题为非A→非B1+非B2   解这类题型要善于在读题中能迅速抽象出逻辑形式,同时要善于做关系的传递(比如:A→B,C→非B,可得出A→非C)。如果已知条件很多的问题时,要迅速找到答案有一定的难度,因此,要同时考虑已知条件和选项,在理解了已知条件的基础上迅速浏览选项,从两头推理,从而尽快找到答案。   -在微波炉清洁剂中加入漂白剂,就会释放出氯气;在浴盆清洁剂中加入漂白剂,也会释放出氯气;在排烟机清洁剂中加入漂白剂,没有释放出任何气体。现有一种未知类型的清洁剂,加入漂白剂后,没有释放出氯气。根据上述实验,以下哪项关于这种未知类型的清洁剂的断定一定为真?   I.它是排烟机清洁剂。   II.它既不是微波炉清洁剂,也不是浴盆清洁剂。   III.它要么是排烟机清洁剂,要么是微波炉清洁剂或浴盆清洁剂。   A.仅I。   B.仅II。   C.仅III。   D.仅I和II。   E. I、II和III。   [解题分析]正确答案:B。   题干断定了四个条件;   在微波炉清洁剂中加入漂白剂,会释放出氯气;   在浴盆清洁剂中加入漂白剂,会释放出氯气;   在排烟机清洁剂中加入漂白剂,没有释放出任何气体;   一种未知类型的清洁剂,加入漂白剂后,没有释放出氯气。   由(1)和(4),可推出该清洁剂不是微波炉清洁剂;   由(2)和(4),可推出该清洁剂不是浴贫清洁剂。   因此,由题干可推出:该清洁剂既不是微波炉清洁剂,也不是浴盆清洁剂。这正是复选项Ⅱ所断定的。其余复选项均不一定为真。   -一个社会是公正的,则以下两个条件必须满足:第一,有健全的法律;第二,贫富差异是允许的,但必须同时确保消灭绝对贫困和每个公民事实上都有公平竞争的机会。   根据题干的条件,最能够得出以下哪项结论?   A. S社会有健全的法律,同时又在消灭了绝对贫困的条件下,允许贫富差异的存在,并且绝大多数公民事实上都有公平竞争的机会。因此,S社会是公正的。   B. S社会有健全的法律,但这是以贫富差异为代价的,因此,S社会是不公正的。   C. S社会允许贫富差异,但所有人都由此获益,并且每个公民都事实上有公平竞争的权利。因此,S社会是公正的。   D. S社会虽然不存在贫富差异,但这是以法律不健全为代价的,因此,S社会是不公正的。   E. S社会法律健全,虽然存在贫富差异,但消灭了绝对贫困。因此,S社会是公正的。   [解题分析]正确答案:D。   题干断定,法律健全是社会公正的必要条件。D项断定,S社会法律不健全,因此,可推出S社会不公正。   其余各项均不能由题干的条件得出。例如,A项断定S社会满足题干所提及的一个公正社会的所有必要条件,但并不能依此就断定S社会是公正的。   -一个心理健康的人,必须保持自尊;一个人只有受到自己所尊敬的人的尊敬,才能保持自尊;而一个用“追星”方式来表达自己尊敬情感的人,不可能受到自己所尊敬的人的尊敬。   以下哪项结论可以从题干的断定中推出?   A.一个心理健康的人,不可能用“追星”的方式来表达自己的尊敬情感。   B.一个心理健康的人,不可能接受用“追星”的方式所表达的尊敬。   C.一个人如果受到了自己所尊敬的人的尊敬,他(她)一定是个心理健康的人。   D.没有一个保持自尊的人,会尊敬一个用“追星”方式表达尊敬情感的人。   E.一个用“追星”方式表达自己尊敬情感的人,完全可以同时保持自尊。 [解题分析]正确答案:A。 在以下的叙述中,“受到自己所尊敬的人的尊敬”缩写为“受到尊敬”;“用‘追星’方式来表达自己尊敬情感”缩写为“追星”。   题干的断定可整理为:   (1)如果“心理健康”,则“保持自尊”;   (2)如果“保持自尊”,则“受到尊敬”;(“只有A,才B”=“如果B,则A”)   (3)如果“追星”,则“不会‘受到尊敬’”。   从(3)可得:   (4)如果“受到尊敬”,则“不会追星”。(“如果A,则非B”=“如果B,则非A”)   从(1)、(2)、和(4)可得:   (5)如果“心理健康”,则“不会追星”。   这正是A项所断定的。因此,A项能从题干的断定中推出。   其余各项都不能从题干推出。   -林园小区有住户家中发现了白蚁。除非小区中有住户家中发现白蚁,否则任何小区都不能免费领取高效杀蚁灵。静园小区可以免费领取高效杀蚁灵。   如果上述断定都真,则以下哪项据此不能断定真假?   Ⅰ.林园小区有的住户家中没有发现白蚁。   Ⅱ.林园小区能免费领取高效杀蚁灵。   Ⅲ.静园小区的住户家中都发现了白蚁。   A.只有Ⅰ。   B.只有Ⅱ。   C.只有Ⅲ。   D.只有Ⅱ和Ⅲ。   E.Ⅰ、Ⅱ和Ⅲ。   [解题分析]正确答案:E。   题干作出了三个断定:   断定一:林园小区有住户家中发现了白蚁。   断定二:小区中有住户家中发现白蚁,是该小区免费领取高效杀蚁灵的必要条件。   断定三:静园小区可以免费领取高效杀蚁灵。   和选项Ⅰ相关的是断定一。但由断定一的真,不能断定选项Ⅰ的真假。   和选项Ⅱ相关的是断定一和断定二。但由这两个断定的真,不能断定选项Ⅱ的真假,因为断定二断定的是必要条件关系。   和选项Ⅲ相关的是断定二和断定三。由这两个断定的真,可推出“静园小区有住户家中发现白蚁”真,但由“静园小区有住户家中发现白蚁”真,不能确定选项Ⅲ的真假。   -如果飞行员严格遵守操作规程,并且飞机在起飞前经过严格的例行技术检验,那么,飞机就不会失事,除非出现例如劫机这样的特殊意外。这架波音747在金沙岛上空失事。   如果上述断定是真的,则以下哪项也一定是真的?   A.如果失事时无特殊意外发生,则飞行员一定没有严格遵守操作规程,并且飞机在起飞前没有经过严格的例行技术检验。   B.如果失事时有特殊意外发生,则飞行员一定严格遵守了操作规程,并且飞机在起飞前经过了严格的例行技术检验。   C.如果飞行员没有严格遵守操作规程,并且飞机起飞前没有经过严格的例行技术检验,则失事时一定没有特殊意外发生。   D.如果失事时没有特殊意外发生,则可得出结论:只要飞机失事的原因是飞行员没有严格遵守操作规程,那么飞机在起飞前一定经过了严格的例行技术检验。   E.如果失事时没有特殊意外发生,则可得出结论:只要飞机失事的原因不是飞机在起飞前没能经过严格的例行技术检验,那么一定是飞行员没有严格遵守操作规程。   [解题分析]正确答案:E。   题干实际断定,如果飞行员严格遵守操作规程,并且飞机在起飞前经过严格的例行技术检验,并且不出现特殊意外,那么,飞机就不会失事。现在飞机失事了。由此可以得出结论:飞机失事的原因是:飞行员没有严格遵守操作规程,或者飞机在起飞前没有经过严格的例行技术检验,或者出现了特殊意外。这三个原因,依次简记为P,Q和R。从题干的条件可知,这三个原因中,至少有一个存在,也可能都存在。   A项不成立。因为由R不是原因,不能断定P和Q同时都是原因,而只能断定其中至少有一个是原因。   B项不成立。因为由R是原因,不能断定P和Q都不是原因。   C项不成立。因为由P和Q都是原因,不能断定R一定不是原因。   D项不成立。因为由R不是原因,不能断定:只要P是原因,Q就一定不是原因。   E项成立。因为R不是原因,可以断定:只要Q不是原因,P就一定是原因。否则,三者都不是原因,就会和题干的推断矛盾。   -只有在广江市的人才能够不理睬通货膨胀的影响;住在广江市的每一个人都要付税;每一个付税的人都发牢骚。根据上面的这些句子,判断下列各项哪项一定是真的? Ⅰ.每一个不理睬通货膨胀影响的人都要付税。 Ⅱ.不发牢骚的人中没有一个能够不理睬通货膨胀的影响。   Ⅲ.每一个发牢骚的人都能够不理睬通货膨胀的影响。   A.仅Ⅰ。   B.仅Ⅰ和Ⅱ。   C.仅Ⅱ。   D.仅Ⅱ和Ⅲ。   E.Ⅰ、Ⅱ和Ⅲ。   [解题分析]正确答案:B。   类似的题目曾多次出现。依次判断Ⅰ、Ⅱ和Ⅲ。   Ⅰ每一个不理睬通货膨胀影响的人,根据题干,那么肯定是住在广江市的人;而住在广江市的人都要付税,所以为真。   Ⅱ不发牢骚的人,那么肯定是不付税的人,因为题干说付税的人都发牢骚;不付税的人肯定没有住在广江市,因为广江市的每一个人都要付税;没有住在广江市,那么根据题干,一定不能够不理睬通货膨胀的影响。所以为真。   Ⅲ每一个发牢骚的人,是一个比付税的人可能更大的范围;而每一个不理睬通货膨胀影响的人,根据题干,肯定是住在广江市的人;这就是说,每一个发牢骚的人都肯定住在广江市;广江市的人是一个比付税的人可能小的范围。因此,可能存在矛盾,不一定为真。   (三)真话假话   把这类考题根据题目的表现形式归结为真话假话型,这是一种通俗的说法,其本质是涉及了逻辑基本规律(同一律、矛盾律、排中律)。解决这类问题的突破口往往是运用对当关系及复合命题推理等逻辑知识在所有叙述中找出有互相矛盾的判断,从而必知其一真一假。   下列两个性质命题是互相矛盾的:   1.“所有S是P”与“有些S不是P”   2.“所有S不是P”与“有些S是P”   下列两个模态命题是互相矛盾的:   1.“必然p”与“可能非p”   2.“不可能p”与“可能p”   要注意的是:有时两个命题虽然不是矛盾的,但互相反对(或下反对),即不能同真(或不能同假),那就可以推出两个判断中至少有一个是假的(或者至少有一个是真的),这也同样可以成为解题的关键。   复合命题的矛盾关系要复杂些,考生要注意下列两个复合命题是互相矛盾的:   1.“p并且q”与“非p或者非q”。   2.“p或者q”与“非p并且非q”。   3.“要么p,要么q”与“p并且q”或者“非p并且非q”。   4.“如果p,那么q”与“p并且非q”。   5.“只有p,才q”与“非p并且q”。   6.“当且仅当p,才q”与“p并且非q”或者“非p并且q”。   真话假话类题型的解题基本思路主要有两种:一是用矛盾(或反对)法,具体做法是从题干提供的所有判断中,找到两个矛盾(或反对)的判断,从而知其真假关系,进一步可推理出答案;二是用假设反证法,有的貌似真话假话型题没有矛盾的判断,只能用假设反证法,这种方法虽然显得笨些,却很有实效。   值得注意的是,因为在最初的考试中多次出现此题型,但在近来的考试中出现的次数已明显减少。不过,考生仍应多加戒备,因为此类考题实在是容易命题。   -某矿山发生了一起严重的安全事故。关于事故原因,甲乙丙丁四位负责人有如下断定:   甲:如果造成事故的直接原因是设备故障,那么肯定有人违反操作规程。   乙:确实有人违反操作规程,但造成事故的直接原因不是设备故障。   丙:造成事故的直接原因确实是设备故障,但并没有人违反操作规章。   丁:造成事故的直接原因是设备故障。   如果上述断定中只有一个人的断定为真,则以下断定都不可能为真,除了   A.甲的断定为真,有人违反了操作规程。   B.甲的断定为真,但没有人违反操作规程。   C.乙的断定为真。   D.丙的断定为真。   E.丁的断定为真。   [解题分析]正确答案:B。   甲和丙的断定互相矛盾,其中必有一真一假。又只有一人的断定为真,因此,乙和丁的断定为假。由丁的断定假,可知:造成事故的直接原因不是设备故障。由乙的断定假,可推知:或者没有人违反操作规程,或者造成事故的直接原因是设备故障。因为已知造成事故的直接原因不是设备故障,所以,可推知:没有人违反操作规程。   这样,可得出结论:   第一,事实上造成事故的直接原因不是设备故障。   第二,事实上没有人违反操作规程。 丙的断定为假,因而甲的断定为真。 因此,除了B项为真外,其余各项均不可能真。   -学校在为失学儿童义捐活动中收到两笔没有署真名的捐款,经过多方查找,可以断定是周、吴、郑、王中的某两位捐的。经询问,周说:“不是我捐的”;吴说:“是王捐的”;郑说:“是吴捐的”;王说:“我肯定没有捐”。   最后经过详细调查证实四个人中只有两个人说的是真话。   根据已知条件,请你判断下列哪项可能为真?   A.是吴和王捐的。   B.是周和王捐的。   C.是郑和王捐的。   D.是郑和吴捐的。   E.是郑和周捐的。   [解题分析]正确答案:C。   吴和王的断定是互相矛盾的,因此,其中必有一真,且只有一真。又由题干,只有两人说的是真话,因此,周和郑两人中有且只有一个人说真话。假设郑说真话,周说假话,则可得出:吴和周捐的款;假设周说真话,郑说假话,则可得出:周和吴都没捐,因而是郑和王捐的。这两种假设都没导致矛盾。因此,根据题干的条件,有关四人中哪两个捐款,有且只有两种情况可能为真:第一,吴和周捐的款,第二,郑和王捐的款。其余的情况一定为假。因此,选项A、B、D和E不可能为真;C项可能为真。   -甲、乙、丙、丁四人在一起议论本班同学申请建行学生贷款的情况。   甲说:“我班所有同学都已申请了贷款。”   乙说:“如果班长申请了贷款,那么学习委员就没申请。”   丙说:“班长申请了贷款。”   丁说:“我班有人没有申请贷款。”   已知四人中只有一人说假话,则可推出以下哪项结论?   A.甲说假话,班长没申请。   B.乙说假话,学习委员没申请。   C.丙说假话,班长没申请。   D.丁说假话,学习委员申请了。   E.甲说假话,学习委员没申请。   [解题分析]正确答案:E。   经典题目。只有一个说假话的限定,给出了解此类题目的一条捷径,就是去寻找两个互相矛盾的命题,这样其中就必然有一真一假。本题中,甲和丁就是矛盾的说法。这时,可以假设甲为真,再看乙和丙的话,乙显然与甲矛盾,则乙也为假,这时假话就不唯一了,因此甲所说必然为假。在“甲说假话”的两个选项中,如果是A,则丙所说为假,矛盾,故选E。   -某县领导参加全县的乡计划生育干部会,临时被邀请上台讲话。由于事先没有做调查研究,也不熟悉县里计划生育的具体情况,只能说些模棱两可、无关痛痒的话。他讲道:“在我们县14个乡中,有的乡完成了计划生育指标;有的乡没有完成计划生育指标;李家集乡就没有完成嘛。”在领导讲话时,县计划生育委员会主任手里捏了一把汗,因为领导讲的三句话中有两句不符合实际,真后悔临时拉领导来讲话。   以下哪项正确表示了该县计划生育工作的实际情况?   A.在14个乡中至少有一个乡没有完成计划生育指标。   B.在14个乡中除李家集乡外还有别的乡没有完成计划生育指标。   C.在14个乡中没有一个乡没有完成计划生育指标。   D.在14个乡中只有一个乡没有完成计划生育指标。   E.在14个乡中只有李家集乡完成了计划生育指标。   [解题分析]正确答案:C。   根据题意,知“有的乡完成”“有的乡没完成”“李家集乡没完成”三种情况一真二假。如果“有的乡完成”为假,则所有乡都完成,而后两句话必有一真,故不可能。从中推出“有的乡完成”必为真,则“有的乡没完成”“李家集乡没完成”均为假。进一步从“有的乡没完成”为假推出所有乡都完成了计划生育指标。   -学校的抗洪赈灾义捐活动收到一大笔没有署真名的捐款,经过多方查找,可以断定是周、吴、郑、王中的某一位捐的。经询问,周说:“不是我捐的”;吴说:“是王捐的”;郑说:“是吴捐的”;王说:“我肯定没有捐”。   最后经过详细调查证实四个人中只有一个人说的是真话。   根据已知条件,请你判断下列哪项为真?   A.周说的是真话,是吴捐的。   B.周说的是假话,是周捐的。   C.吴说的是真话,是王捐的。   D.郑说的是假话,是郑捐的。   E.王说的是真话,是郑捐的。   [解题分析]正确答案:B。   吴和王的话互相矛盾,因此,其中必有一真。又只有一人说真话,可推出周和郑均说假话,即事实上是周捐的款。所以B项为真。   (四)逻辑错误或推理评价   逻辑错误型考题较多地出现在早期的逻辑考试中,近来已不再考类似的考题。因为在大纲中已规定“不考察逻辑学的专门知识”,所以,直接判断逻辑错误的考题今后应该不会再出现。今后要考对逻辑错误的辨析,也只能考逻辑错误的类比,比如问你“题干中所犯逻辑错误与下列备选项中的哪一项最为类似?”也就是让考生比较题干和选项中所犯逻辑错误的相同或不同。这类逻辑错误类比型考题本书已把其归到下一节“类比与推理结构比较”。 另外,要求考生评价推理过程或指出推理缺陷的考题是今后这类考题的方向,这要引起一定的重视。 -在产品检验中,误检包括两种情况:一是把不合格产品下为合格:二是把合格产品定为不合格。有甲乙两个产品检验系统,它们依据的是不同的原理,但共同之处在于:第一,它们都能检测出所有送检的不合格产品;第二,都仍有恰好3%的误检率:第三,不存在一个产品,会被两个系统都误检。现在把这两个系统合并为一个系统,使得被该系统测定为不合格的产品,包括且只包括两个系统分别工作时都测定的不合格产品。可以得出结论:这样的产品检验系统的误检率为0。   以下哪项最为恰当地评价了上述推理?   A.上述推理是必然性的,即如果前提真,则结论一定真。   B.上述推理很强,但不是必然性的,即如果前提真,则为结论提供了很强的证据,但附加的信息仍可能削弱该论证。   C.上述推理很弱,前提尽管与结论相关,但最多只为结论提供了不充分的根据。   D.上述推理的前提中包含矛盾。   E.该推理不能成立,因为它把某事件发生的必要条件的根据,当作充分条件的根据。   [解题分析]正确答案:A。   不妨把上述甲乙两个系统所合并成的系统称为系统丙。   由题干,对于甲乙两个系统中的任一系统:   测定为合格的产品实际上都是合格产品;   测定为不合格的产品中,实际上有3%是合格产品,属误检;   甲系统误检为不合格的产品,若经乙系统检验,则被测定为合格(同样,乙系统误检为不合格的产品,若经甲系统检验,则被测定为合格)。   因此,任意一批产品,如果分别经过甲乙两个系统的检验,那么,其中的不合格产品,包括并且只包括两个系统都测定为不合格的产品。   由题干,丙系统测定为不合格的产品,包括且只包括两个系统分别工作时都测定的不合格产品,因此,其误检率为0。   -有一种观点认为,到21世纪,和发达国家相比,发展中国家将有更多的人死于艾滋病。其根据是:据统计,艾滋病毒感染者人数在发达国家趋于稳定或略有下降,在发展中国家却持续快速发展;到21世纪,估计全球的艾滋病毒感染者将达到4000万至1亿1千万人,其中,60%将集中在发展中国家。这一观点缺乏充分的说服力。因为,同样权威的统计数据表明,发达国家的艾滋病感染者从感染到发病的平均时间要大大短于发展中国家,而从发病到死亡的平均时间只有发展中国家的二分之一。   以下哪项最为恰当地概括了上述反驳所使用的方法?   A.对“论敌”的立论动机提出质疑。   B.指出“论敌”把两个相近的概念当作同一概念来使用。   C.对“论敌”的论据的真实性提出质疑。   D.提出一个反例来否定“论敌”的一般性结论。   E.指出“论敌”在论证中没有明确具体的时间范围。   [解题分析]正确答案:B。   本题属于指出推理缺陷。   题干所反驳的观点的结论是:到21世纪,和发达国家相比,发展中国家将有更多的人死于艾滋病;其根据是:艾滋病毒感染者人数在发达国家趋于稳定或略有下降,在发展中国家却持续快速上升。   题干对此所作的反驳实际上指出:上述观点把“死于艾滋病的人数”和“感染艾滋病毒的人数”这两个相近的概念错误地当作同一概念使用;艾滋病毒感染者人数在发达国家虽低于发展中国家,但由于发达国家的艾滋病感染者从感染到发病,以及从发病到死亡的平均时间要大大短于发展中国家,因此,其实际死于艾滋病的人数仍可能多于发展中国家。因此,B项恰当地概括了题干中的反驳所使用的方法。其余各项均不恰当。   -东方日出,西方日落,社会是发展的,生物是进化的,都反映了不依人的意志为转移的客观规律。小王对此不以为然。他说,有的规律是可以改造的。人能改造一切,当然也能改造某些客观规律。比如价值规律不是乖乖地为精明的经营者服务了吗?人不是把肆虐的洪水制住而变害为利了吗?   试问,以下哪项最为确切地提示了小王上述议论中的错误?   A.他过高地估计了人的力量。   B.他认为“人能改造一切”是武断的。   C.他混淆了“运用”与“改造”这两个概念。   D.洪水并没有都被彻底制服。   E.价值规律若被改造就不叫价值规律了。   [解题分析]正确答案:C。   且不说洪水是不是被人类彻底制服,这不是关键,关键是小王认为肆虐的洪水被人们制住而变害为利是改造规律的体现,其实这是人类运用规律的体现。精明的经营者也只是掌握了价值规律然后为我所用,怎么可能改造价值规律呢?价值规律若能被人随意改造那就不叫“规律”了(注意:这与选项E并不同)。所以,选C。   (五)类比与推理结构比较   类比是根据两个对象在一系列属性上是相同的,而且知道其中的一个对象还具有另一种属性,由此推出另一个对象也具有这一属性的推理。类比推理的客观依据是,现实中的事物的各个属性并不是孤立存在的,而是互相联系互相制约的,如两个对象在一系列属性上是相似的,那它们完全可能在另外的属性上也是相似的。   推理结构比较是主要从形式结构上比较题干和五个选项之间的相同或不同,即比较几个不同推理在结构上的相同或者不同。其解题基本思路是,着重考虑从具体的、有内容的思维过程的论述中抽象出一般形式结构,即用命题变项表示其中的单个命题,或用词项变项表示直言命题中的词项,每一个推理中相同的命题或词项用相同的变项表示,不同的命题或词项用不同的变项表示。做这类题型只考虑抽象出推理结构和形式,而不考虑其内容的对错,例如:地球上一切事物都是生物,长城是地球上的事物,所以,长城是生物。该三段论虽然大前提有误,但推理结构是正确的。有时,甚至题干本身的推理结构就是错误的,由于要求我们从中找出一个推理结构与题干类似的选项,因此我们同样不要在乎题干推理结构是否正确,只要找到一个类似结构的选项就是正确答案。   -要选修数理逻辑课,必须已修普通逻辑课,并对数学感兴趣。有些学生虽然对数学感兴趣,但并没修过普通逻辑课,因此,有些对数学感兴趣的学生不能选修数理逻辑课。   以下哪项的逻辑结构与题干的最为类似?   A.据学校规定,要获得本年度的特设奖学金,必须来自贫困地区,并且成绩优秀。有些本年度特设奖学金的获得者成绩优秀,但并非来自贫困地区,因此,学校评选本年度奖学金的规定并没有得到很好的执行。   B.一本书要畅销,必须既有可读性,又经过精心的包装。有些畅销书可读性并不大,因此,有些畅销书主要是靠包装。   C.任何缺乏经常保养的汽车使用了几年之后都需要维修,有些汽车用了很长时间以后还不需要维修,因此,有些汽车经常得到保养。   D.高级写字楼要值得投资,必须设计新颖,或者能提供大量办公用地。有些新写字楼虽然设计新颖,但不能提供大量的办公用地,因此,有些新写字楼不值得投资。 E.为初学的骑士训练的马必须强健而且温驯,有些马强健但并不温驯,因此,有些强健的马并不适合于初学的骑手。 [解题分析]正确答案:E。   题干的结构是:   如果p,则(q并且r);r并且非q;因此,r并且非p   各选项中,E项的结构和题干的最为类似。   -农科院最近研制了一高效杀虫剂,通过飞机喷撒,能够大面积地杀死农田中的害虫。这种杀虫剂的特殊配方虽然能保护鸟类免受其害,但却无法保护有益昆虫。因此,这种杀虫剂在杀死害虫的同时,也杀死了农田中的各种益虫。   以下哪项产品的特点,和题干中的杀虫剂最为类似?   A.一种新型战斗机,它所装有的特殊电子仪器使得飞机员能对视野之外的目标发起有效攻击。这种电子仪器能区分客机和战斗机,但不能同样准确地区分不同的战斗机。因此,当它在对视野之外的目标发起有效攻击时,有可能误击友机。   B.一种带有特殊回音强立体声效果的组合音响,它能使其主人在欣赏它的时候倍感兴奋和刺激,但往往同时使左邻右舍不得安宁。   C.一部经典的中国文学名著,它真实地再现了中晚期中国封建社会的历史,但是,不同立场的读者从中得出不同的见解和结论。   D.一种新投入市场的感冒药,它能迅速消除患者的感冒症状,但也会使服药者在一段时间中昏昏欲睡。   E.一种新推出的电脑杀毒软件,它能随时监视并杀除入侵病毒,并在必要时会自动提醒使用者升级,但是,它同时减低了电脑的运作速度。[解题分析]正确答案:A。   题干中的杀虫剂的特点是能区分鸟类和昆虫,但不能区分昆虫中的益虫与害虫,因此,在杀死害虫时虽然高效,但同时也杀死了益虫。   A项中的战斗机的特点是能区分客机和战斗机,但不能区分战斗机中的敌机与友机,因此,攻击敌机虽然有效,但也可能误击友机。这和题干中杀虫剂的特点类似。其余各项产品都不具有类似于题干中杀虫剂的上述特点。   -对同一事物,有的人说“好”,有的人说“不好”,这两种人之间没有共同语言。可见,不存在全民族通用的共同语言。   以下除哪项外,都与题干推理所犯的逻辑错误近似?   A.甲:“厂里规定,工作时禁止吸烟。”乙:“当然,可我吸烟时从不工作。”   B.有的写作教材上讲,写作中应当讲究语言形式的美,我的看法不同。我认为语言就应该朴实,不应该追求那些形式主义的东西。   C.有意杀人者应处死刑,行刑者是有意杀人者,所以行刑者应处死刑。   D.象是动物,所以小象是小动物。   E.这种观点既不属于唯物主义,又不属于唯心主义,我看两者都有点像。   [解题分析]正确答案:E。   逻辑错误题目首先要搞清是什么逻辑错误。本题题干的逻辑错误是偷换概念,前后出现的两个“共同语言”是两个概念。   与题干推理错误最近似的是A和B,工作既是一个时段概念,也可以是一个动作的概今,A偷换了二者,B中的语言也有这个问题;选项C严格地讲并没有逻辑错误,只是前提“有意杀人者应处死刑”本身有问题;选项D的逻辑错误是“小动物”已经成了另外一个概念,与“小的动物”是有不同的。之所以选择E,是因为E的逻辑错误是自相矛盾,与题干的逻辑错误离得最远。   -如果学校的财务部门没有人上班,我们的支票就不能入账;我们的支票不能入账。因此,学校的财务部门没有人上班。   请在下列各项中选出与上句推理结构最为相似的一句。   A.如果太阳神队主场是在雨中与对手激战,就一定会赢。现在太阳神队主场输了,看来一定不是在雨中进行的比赛。   B.如果太阳晒得厉害,李明就不会去游泳。今天太阳晒得果然厉害,因此可以断定,李明一定没有去游泳。   C.所有的学生都可以参加这一次的决赛,除非没有通过资格赛的测试。这个学生不能参加决赛,因此他一定没有通过资格赛的测试。   D.倘若是妈妈做的菜,菜里面就一定会放红辣椒。菜里面果然有红辣椒,看来,是妈妈做的菜。   E.如果没有特别的原因,公司一般不批准职员们的事假申请。公司批准了职员陈小鹏的事假申请,看来其中一定有一些特别的原因。   [解题分析]正确答案:D。   题干的推理结构是:如果P,那么Q;Q。因此P。   A项的结构是:如果P,那么Q;非Q。因此非P。   B项的结构是:如果P,那么Q;P。因此Q。   C项的结构是:P,除非Q;非P。因此非Q。   D项的结构是:如果P,那么Q;Q。因此P。   E项的结构是:如果P,那么Q;非Q。因此非P。   显然,D项和题干具有相同的结构。
本文档为【11道经典逻辑推理题】,请使用软件OFFICE或WPS软件打开。作品中的文字与图均可以修改和编辑, 图片更改请在作品中右键图片并更换,文字修改请直接点击文字进行修改,也可以新增和删除文档中的内容。
该文档来自用户分享,如有侵权行为请发邮件ishare@vip.sina.com联系网站客服,我们会及时删除。
[版权声明] 本站所有资料为用户分享产生,若发现您的权利被侵害,请联系客服邮件isharekefu@iask.cn,我们尽快处理。
本作品所展示的图片、画像、字体、音乐的版权可能需版权方额外授权,请谨慎使用。
网站提供的党政主题相关内容(国旗、国徽、党徽..)目的在于配合国家政策宣传,仅限个人学习分享使用,禁止用于任何广告和商用目的。
下载需要: 免费 已有0 人下载
最新资料
资料动态
专题动态
个人认证用户
豆浆
暂无简介~
格式:doc
大小:371KB
软件:Word
页数:106
分类:工学
上传时间:2019-05-17
浏览量:209